You are on page 1of 315

Welcome to SRM Gateway

Dear Student,

Welcome to SRM Gateway. We thank you very much for joining SRM Gateway. We are
all committed to your satisfaction and fulfill your expectations in training, quality in our
services, which is our primary goal.
We are sure that you will become very skilled and will be positioning yourself well in the IT
industry.
We shall always ensure that you are guided well during the course.
We wish you all the best for the complete learning of the technology Stay ahead in the
technology. Way ahead of others.

Warm Regards,
Management
SRM Gateway

Document Number : SRMGW - IIT-JEE - MATH-01


Course Code : IIT-JEE

SRM Gateway
This document is the property of SRM Gateway. This
document has been prepared exclusively for the use of the
students. No part of this document shall be copied or
transferred in any form or by any means which would
initiate legal proceedings, if found.

SRM Gateway
No. 9, 3rd Avenue, Ashok Nagar, Chennai 600 083

Preface
About the book

This book will be an effective training supplement for students to master the subject. This
material has in-depth coverage and review questions to give you a better understanding.
A panel of experts have complied the material, which has been rigorously reviewed by industry experts and we hope that this material will be a value addition.
We also would value your feedback, which will be useful for us to fine-tune it better.

Wishing you all success.

SRM Gateway

IIT - MATHS
SET - 1

INDEX

1.

BASIC MATHEMATICS ...................................................................................

2.

ALGEBRA PROGRESSION .............................................................................

54

3.

BASIC TRIGONOMETRY ..............................................................................

132

4.

TRIGONOMETRY EQUATIONS ...................................................................

164

5.

INVERSE TRIGONOMETRY FUNCTION ...................................................

186

6.

PROPERTIES OF TRIANGLE .......................................................................

210

7.

CO - ORDINATE GEOMENTRY ...................................................................

232

IIT- MATHS

BASIC MATHEMATICS

BASIC TRIGONOMETRY

Number System
(i) Natural numbers

: N = {1, 2, 3, 4, . . . . . }

(ii) Whole numbers

: W = {0, 1, 2, 3, 4, . . . . . }

(iii) Integers

: Z or I = {. . . . . 3, 2, 1, 0, 1, 2, 3, . . . . .}

Natural numbers are also called positive integers (denoted by Z+ or I+)


Whole numbers are also called nonnegative integers.
The set of negative integers, Z or I = {. . .. . 3, 2, 1}.
The set of non positive integers is {, 3, 2, 1, 0}.
Zero is neither positive nor negative but it is nonpositive as well as nonnegative.
(iv)Rational numbers: Numbers of the form p/q where p, q Z and q 0 (because division by
zero is not defined). Q represents their set. All integers are rational numbers with q = 1
When q 1 and p, q have no common factor except 1, the rational numbers are called
fractions.
Rational numbers when represented in decimal form are either terminating or non
terminating but repeating.
e.g., 5/4 = 1.25 (terminating)
5/3 = 1.6666 . . . . . (non terminating but repeating)
p

(v) Irrational numbers: Numbers, which cannot be represented in q form.


In decimal representation, they are neither terminating nor repeating all surds fall into this
category
e.g.,

2 , 151 / 3 , p, etc.

Note : p 22/7, 22/7 is only an approximate value of p in terms of rational numbers,


taken for convenience Actually p = 3.14159 . . . . .
(vi)Real numbers: All rational and irrational numbers taken together form the set of real
numbers, represented by R. This is the largest set in the real world of numbers.
Also note that
Integers which give an integer on division by 2 are called even integers otherwise they are
called odd integers.
Zero is considered as even number.
The set of natural numbers can be divided in two ways.
(i) Odd and even natural numbers.
(ii) Prime numbers (which are not divisible by any number except 1 and themselves) and
composite numbers (which have some other factor apart from 1 and themselves).

IIT- MATHS
1 is neither prime nor composite
2 is the only even number which is prime

Set Theory
Basic Concept
Set: A set is a welldefined collection of objects or elements. Each element in a set is unique.
Usually but not necessarily a set is denoted by a capital letter e.g. A, B, . . . . . U, V etc. and the elements
are enclosed between brackets {}, denoted by small letters a, b, . . . . . x, y etc. For example:
A = Set of all small English alphabets
= {a, b, c, . . . . . x, y, z}
B = Set of all positive integers less than or equal to 10
= {1, 2, 3, 4, 5, 6, 7, 8, 9, 10}
R = Set of real numbers
= {x : < x < }
The elements of a set can be discrete (e.g. set of all English alphabets) or continuous (e.g., set of
real numbers). The set may contain finite or infinite number of elements. A set may contain no elements
and such a set is called Void set or Null set or empty set and is denoted by (phi). The number of
elements of a set A is denoted as n(A) and hence n () = 0 as it contains no element.

Union of sets
Union of two or more sets is the set of all elements that belong to any of these sets. The symbol used
for union of sets is
i.e., AB = Union of set A and set B = {x : x A or x B(or both)}
e.g. If A = {1, 2, 3, 4} and B = {2, 4, 5, 6} and C {1, 2, 6, 8}, then ABC = {1, 2, 3, 4, 5, 6, 8}

Intersection of sets
It is the set of all the elements, which are common to all the sets. The symbol used for intersection
of sets is i.e., AB = {x : xA and xB} e.g. If A = {1, 2, 3, 4} & B = {2, 4, 5, 6}and C = {1, 2, 6, 8},
then ABC = {2}.
Remember that n(AB) = n(A) + n(B) n (AB)

Difference of two sets


The difference of set A to B denoted as A B is the set of those elements that are in the set
A but not in the set B i.e., A B = {x : x A and xB}
Similarly B A = {x : x B and x A}. In general A B B A
e.g.

If A = {a, b, c, d} and B = {b, c, e, f}, then A B = {a, d} and B A = {e. f}

BASIC TRIGONOMETRY

LOGARITHM
If a is a positive real number other then 1 and ab = c, then we write logac = b obviously c is positive.
For example log3 81 = 4 34 = 81
Note
The expression logb a is meaningful for a > 0 and for either 0 < b < 1. or b > 1
a = b log b a
log c b

logab = log a
c
a1 a 2 0
if b 1
logba1 logba2 0 a a if 0 b 1
1
2
Brain Teaser 1 : log x2 = 2logx, is it true or false?
Formulae
(i) loga|mn| = loga|m| + loga |n|
(ii) loga m = loga|m| loga|n|
n

(iii) loga|mn| = n log a|m|


(iv) logab = logcb logac
(v) log a k N =

1
logaN
k

Modulus Function
Let x R, then the magnitude of x is called its absolute value and in general, denoted by |x| and
x , x 0
x 0

defined as |x| = x,

Note that x = 0 can be included either with positive values of x or with negative values of x. As we
know all real numbers can be plotted on the real number line, |x| in fact represents the distance of number
x from the origin, measured along the numberline. Thus |x| 0 secondly, any point x lying on the real
number line will have its coordinate as (x, 0). Thus its distance from the origin is x 2
Hence |x| =

x 2 . Thus we can define |x| as |x| =

x2

e.g if x = 2.5 then |x| = 2.5, if x = 3.8 then |x| = 3.8


Brain Teaser 2 : If x R then

x
4x

is

1
1
1
, , or not defined.
2
2
2

Basic Properties :

||x|| = |x|
|x| > a x > a or x < a if a R+ and x R if a R
|x| < a a < x < a if a R+ and no solution if a R{0}
|x + y| |x| + |y|
|xy| = |x||y|

IIT- MATHS
x |x|

,y0
y | y|

Intervals

Intervals are basically subsets of R and are of very much importance in calculus as you will get to
know shortly. If there are two numbers a, b R such that a < b, we can define four types of intervals as
follows:
Open interval : (a, b) = {x : a < x < b} i.e., end points are not included.
Closed interval: [a, b] = {x : a x b} i.e., end points are also included. This is possible only when
both a and b are finite.
Openclosed Interval: (a, b] = {x : a < x b}
Closedopen interval:[a, b) = {x : a x < b}
The infinite intervals are defined as follows
(a, ) = {x : x > a}
[a, ) = {x : x a}
(, b] = {x : x b}
intervals are particularly important in solving inequalities or in finding domains etc.

Inequalities
The following are some very useful points to remember
a b either a < b or a = b.
a < b and b < c a < c.
a < b a + c < b + c cR
a < b a > b i.e., inequality sign reverses if both sides are multiplied by a negative number.
a < b and c < d a + c < b + d and a d < b c.
a < b ma < mb if m > 0 and ma > mb if m < 0.
0 < a < b ar < br if r > 0 and ar > br if r < 0.

a 2 a > 0 and equality holds for a = 1.


a

a 2 a < 0 and equality holds for a = 1.


a

If a1 > b1, a2 > b2, a3 > b3 . . . . . , where ai > 0, bi > 0, i = 1, 2,


Then a1 + a2 + a3 + . . . > b1 + b2 + b3 + . . . and a1a2a3 . . . > b1b2b3 . . .
If a > b, p and q are some positive integers, then following results are evident.

BASIC TRIGONOMETRY
a > b an > bn an < bn where n N
a > b al/q > bl/q ap/q > bp/q

Wavy Curve Method


In order to solve inequalities of the form
Px
Px
0,
0 , where P(x) and Q(x) are polynomials, we use the following method:
Qx
Qx

If x1 and x2 (x1 < x2) are two consecutive distinct roots of a polynomial equation, then within this
interval the polynomial itself takes on values having the same sign. Now find all the roots of the polynomial equations P(x) = 0 and Q(x) = 0. Ignore the common roots and write

x 1 x 2 x 3 . . . . . x n
Px
f x
x 1 x 2 x 3 . . . . . x m ,
Qx
Where a1, a2, . . . . . an, b1, b2, . . . . . , bm are distinct real numbers. Then f(x) = 0 for x = a1,
a2, . . . . . , an and f(x) is not defined for x = b1, b2, . . . . . , bm apart from these (m + n) real numbers
f(x) is either positive or negative. Now arrange a1, a2, . . . . . , an, b1, b2, . . . . . , bm in an increasing order say
c1, c2, c3, c4, c5, . . . . . , cm+n. Plot them on the real line. And draw a curve starting from right of cm+n along
the real line which alternately changes its position at these points. This curve is known as the wavy curve.

The intervals in which the curve is above the real line will be the intervals for which f(x) is positive
and intervals in which the curve is below the real line will be the intervals in which f(x) is negative.

Factor Theorem
Let p(x) be a polynomial of degree greater than or equal to 1 and a be a real number such that p(a)
= 0, then (x a) is a factor of p(x). Conversely, if (xa) is a factor of p(x), then p(a) = 0.

Remainder Theorem
Let p(x) be any polynomial of degree greater than or equal to one and a be any real number. If p(x)
is divided by (xa), then the remainder is equal to p(a).

DETERMINANTS
Consider the equations a1x + b1y = 0 and a2x + b2y = 0. These give
a1 y a 2
a
a

1 2 ab ab =0
1 2
2 1
b1
x
b2
b1 b 2
a1

b1

We express this eliminant as a b = 0.


2
2

IIT- MATHS
a1

b1

The expression a b is called a determinant of order two, and equals a1b2 a2b1.
2
2
A determinant of order three consisting of 3 rows and 3 columns is written as
a1

b1

c1

a2

b2

c2

a3

b3

b 2c 2

a 2c 2

a 2b2

and is equal to a1 b c b1 a c + c1 a b
3 3
3 3
3 3
c3

= a1 (b2c3 c2b3) b1 (a2c3 c2a3) + c1(a2b3 b2a3)


The numbers ai, bi, ci (i = 1, 2, 3) are called the elements of the determinant.

Function
In the study of natural phenomena and the solution of technical and mathematical problems, it is
necessary to consider the variation of one quantity as dependent on the variation of another. For example,
in studies of motion, the path traversed is regarded as a variable, which varies with time. Here we say that
the distance traversed is a function of time. The area of a circle, in terms of its radius R, is pR2. If R takes
on various numerical values, the area assumes different numerical values. So the variation of one variable
brings about a variation in the other. Hence area of the circle is a function of the radius R.
If to each value of variable x (within a certain range) there corresponds a unique value of another
variable y, then we say that y is a function of x, or, in functional notation y = f(x). The variable x is called
the independent variable or argument. And the variable y is called the dependent variable. The relation
between the variable x and y is called a functional relation. The letter f in the functional notation y = f(x)
indicates that some kind of operation must be performed on the value of x in order to obtain the values of y.
f(x)

f(x)
y = f(x)

L
y = f(x)

C
y3

y2

y2
y1

y1

x2
x3

x1
Fig (a)

x0

Fig (b)

These figures show the graph of two arbitrary curves. In the figure any line drawn parallel to yaxis would meet the curve at only one point. That means each element of X would have one and only one
image. Thus the figure (a) would represent the graph of a function.
In the figure (b) certain line (e.g. line L) would meet the curve in more than one points (A, B and
C). Thus element x0 of X would have three distinct images. Thus this curve will not represent a function.
The set of all possible values which the independent variable (here x) is permitted to take for a
given functional dependence to be defined is called the domain of definition or simply the domain of the
8

BASIC TRIGONOMETRY
function.
e.g. The function y = sin x is defined for all values of x. Therefore its domain of definition is the
infinite interval < x < .
The function y =

1
x 1

is defined for all x > 1 its domain is (1, ).

Elementary Functions:
(i) Constant function: y = c where c is a constant, defined for all real x.
(ii) Power function: y x
(a) a is positive integer. The function is defined in the infinite interval < x < .
(b) a is negative integer. The function is defined for all values of x except for x = 0.
(iii) General exponential function: y = ax, where a is positive not equal to unity. This function
is defined for all values of x.
(iv)Logarithmic function: y = logax, a > 0 but a 1. This function is defined for all x > 0.
(v) Trigonometric function: y = sinx, y = cosx defined for all real x
y = tanx, y = secx, defined for R (2n + 1)

.
2

y = cotx, y = cosecx, defined for R n , where nl


It must be noted that in all these function the variable x is expressed in radians. All these
function have a very important property that is Periodicity.
`

Is sec2 tan2 = 1 valid for all R (real) ?


(vi) Algebraic function:
(a) Polynomial function: y = a0xn + a1xn1 + + an, where a0, a1 an are real constants
(a0 0) and n is a positive integer, called the polynomial of degree n.
e.g. y = ax + b, a 0 (a linear function)
y = ax2 + bx + c, a 0 (a quadratic function)
A polynomial function is defined for all real values of x.
a 0 x n a 1 x n 1 ... a n

(b) Rational Function y = b x m b x m 1 ... b


0
1
m
e.g:y = a/x (inverse variation)
The rational function is defined for all values of x except for those where the denominator
becomes zero.
(c) Irrational function e.g. y =

2x 2 x
1 5x 2

IIT- MATHS

Differential Calculus
Let y = f(x) be a function. Putting the values of x in this relation, we obtain the corresponding
values of y. Suppose we start putting some values of x in increasing order. The respective values of y
that we obtain may turn out to be in increasing order, or in decreasing order, or they may remain constant,
or they may even have a mixed trend, depending upon the type of function.
Let us take two values of x: x1 and x2(x1 < x2). So, y1 = f(x1) and y2 = f(x2)
y y

2
1
Then, the quantity x x will tell us the average rate of change of y w.r.t. x in the interval [x1, x2] .
2
1

y 2 y1

Let y2 > y1 x x is positive Function is increasing on an average.


2
1
y 2 y1

if y2 < y1 x x is negative Function is decreasing on an average.


2
1
y 2 y1

If y2 = y1 x x is zero Function is constant on an average.


2
1
y y

2
1
As you can see, if x1 and x2 are sufficiently far apart, the quantity x x can not give the exact
2
1

idea of the variation of y w.r.t. x in the interval [x1, x2]. it just provides an overall information. For
example if y2 = y1 it does not necessarily mean that y is same for all x in the interval [x1, x2]. Thus, to
obtain a sufficiently accurate information, we have to choose x1 and x2 sufficiently close to each other.
This sufficiently close is the key word here. To know the rate of change of y w.r.t. x at x = x1, we take x2
y 2 y1

very near to x1 (as much as possible), i.e., x 2 tends to x1 and then calculate x x . In the limiting case,
2
1
dy

we say that x2 nearly coincides with x1 and represent it as x2 x1. We use the notation dx

y 2 y1

x x1

for x x
2
1

as x2 x1. dx means small change in x (near x = x1) and dy means the corresponding change in y. We call
dy
the derivative or the differential coefficient of y w.r.t. x.
dx

y
y2 y1
x2 x 1

x1

x2

(You can understand it physically by taking x as time and y as displacement of a body,


10

BASIC TRIGONOMETRY
Then

dy
denotes the magnitude of velocity).
dx

dy
df ( x )
is also represented as f (x) or
dx
dx
dy

Graphically, dx

x x1

(i.e.,

dy
computed at x = x1) denotes the slope of the tangent to the curve y =
dx

f(x) at x = x1
We will not here derive the formulae for (x) of various functions, but we give the results of the
derivations here,

Basic Differentiation Formulae


y = constant

dy
0
dx

y = tan1x

dy
1

dx 1 x 2

y = xn

dy
nx n 1
dx

y = cot1x

dy
1

dx 1 x 2

y = sinx

dy
cos x
dx

y = cosec1x

dx
| x | x 2 1

y = cosx

dy
sin x
dx

y = sec1 x

dx
| x | x 2 1

y = tanx

dy
sec 2 x
dx

y = ax

dy
a x ln a
dx

y = cotx

dy
cos ec 2 x
dx

y = ex

dy x
e
dx

y = sin1x

dx
1 x2

y = logax

dx x ln a

y = cos1x

dx
1 x2

y = ln x

dy

dy

dy

dy

dy

dy 1

dx x

Some Important Theorems


The following are very important theorems, which can be applied directly.
Theorem 1:
If a function is of the form y = k f(x), where k is a constant, then

dy
df ( x )
k
dx
dx

Theorem 2:
The derivative of the sum or difference of a finite number of differentiable functions is equal
to the sum or difference of the derivatives of these functions.

11

IIT- MATHS
i.e., if y = u (x) + v (x) + w(x) then y = u(x) + v(x) + w(x).
Theorem 3
The derivative of the product of two differentiable functions is equal to the product of the
derivative of the first function with the second function plus the product of the first function
with the derivative of the second function: i.e., if y = uv, then y = uv + uv.
This formula can be extended for the derivatives of the product of any (finite) number of
functions.
Theorem 4
u(x)

dy

If y = v( x ) , then y =
dx
Theorem 5

u v uv
v2

If y = uv, where u and v are functions of x, then y = vuv-1 u + uv ln u.v


Derivative of a Composite Function
Given a composite function y = f(x), i.e., a function represented by
y = F(u), u = f (x) or y = F[f(x)],
then y =

dy dF du

dx du dx

This is called the chain rule. The rule can be extended to any number of composite function;
e.g. if
y = f(u(v)), then y =

dy df du dv

.
dx du dv dx

Parametric Representation of a Function and its Derivatives


We find the trajectory of a load dropped from an aeroplane moving horizontally with uniform
velocity v0 at an altitude y0. We take the co-ordinate system as shown and assume that the load is dropped
at the instant the aeroplane cuts the y-axis. Since the horizontal translation is uniform, the position of the
load at any time t, is given x = v0t, y = y0

gt 2
2

Y
v0
(x, y)
X

Those two equations are called the parametric equations of the trajectory because the two variables x and y have been expressed in terms of the third variable t (parameter) i.e. two equations x = (t),
y = (t)

12

BASIC TRIGONOMETRY
where t assumes values that lie in a given interval (t1, t2)
dy dy / dt

(d / dt ) ( t )

Then dx dx / dt (d / dt ) ( t )

Second Derivative of a Function


The second derivative of y w.r.t. x is the function obtained by differentiating

It is represented as

So,

dy
w.r.t. x.
dx

d2 y
dy
5
= 5x4
2 or y or f (x). e.g. If y = x then
dx
dx

d 2 y d dy
d
=
(5x 4 ) = 5.4 x3 = 20x3
2
dx dx
dx
dx

The acceleration a of a particle is the second derivative of the distance s (given as a function of
time).
i.e. if s = f(t) then v =

ds
d 2s
dv
= f (t) and a = 2 =
= f (t)
dt
dt
dt

THE BEHAVIOUR OF FUNCTIONS


The following behaviours of a function are important to study

Increasing and Decreasing Functions


(i) Increasing Functions
If y = f(x) and x2 > x1 implies y2 > y1 for any x belonging to the interval [a, b], then y is said
to be an increasing function of x. (x) increases in [a, b] f (x) > 0 x in (a, b).
(ii) Decreasing Function
If x2 > x1 y2 < y1 for any x belonging to [a, b], then y is said to be a decreasing function
of x.
f(x) decreases in [a, b] f (x) < 0 x in (a, b)
For a constant function, f (x) = 0
For a non-decreasing function f (x) 0
For a non increasing function, f (x) 0

Maxima and Minima of Functions


A function f is said to have a maxima at x = x0 if f(x) < f(x0), x in the immediate neighbourhood
of x0.
Similarly, a function f is said to have a minima at x = x0 if f(x) > f(x0), x in the immediate
neighbourhood of x0
We have used the word immediate here because a given function may have any number of high
13

IIT- MATHS
and low points. It is just like moving on an uneven surface (which has many bumps and depressions).
Mathematically, these bumps are called the points of local maxima and the depressions are called the
points of local minima. The highest of all the bumps is the global maxima and the lowest of all the
depressions is the global minima. We state here the preliminary methods only to find the maxima and
minima of functions.
dy
or f (x)
dx

(a)

Find

(b)

Find the points at which it becomes zero. These points are called critical points.

To find the points of maxima and minima we resort to either of the following tests.
(a) First Derivative Test
Suppose x = x0 is a critical point i.e., f (x0) = 0.
If f (x) changes sign from positive to negative in the neighbourhood of x = x0

Maxima at x0

If f (x) changes sign from negative to positive in the neighbourhood of x = x0

Minima at x0

(b) Second Derivative Test


(i)

d2y
Find 2 or f (x)
dx

(ii)

Compute the value of f (x) at the critical points

If it is positive Minima at those values of x.


If it is negative maxima at those values of x.
If the function is defined in an interval [a, b], then to find the maxima and minimum
values i.e., global maxima and global minimum of the function in that interval we com
pare the values of the function (i.e., y) at all the critical points and also the end points (i.e.,
y = f(a) and f(b)). Then the largest among them gives the global maximum values and
smallest gives the global minimum values.

Integral Calculus: The Antiderivative of Function


A function F(x) is called the antiderivative of the function f(x) on the interval [a, b] if, at all points
of the interval f(x) = F(x).
x3 '
x3
For example, the antiderivative of the function f(x) = x is
, as 3 = x2. The function
3

3
x3
x3
x
2 and
1 are also antiderivatives of f(x) = x2. Infact,
C , where C is an arbitrary constant, is
3
3
3
2

the antiderivative of x2. So if a function f(x) possesses an anti-derivative F(x), then it possesses infinitely
many antiderivatives, all of them being contained in the expression F(x) + C, where C is a constant.

14

BASIC TRIGONOMETRY
If the function F(x) is an antiderivative of f(x), then the expression F(x) + C is called the indefinite
integral of the function f(x) and is denoted by the symbol f(x) dx. Thus, by definition f(x) dx = F(x) +
C, if F(x) = f(x). If a function f(x) is continuous on an interval [a, b], then this function has an antiderivative. The process of finding the antiderivative of a function f(x) is called integration. Two different integrals of a function differ by a constant.

Standard Elementary Integrals


In the following integrals, C stands for an arbitrary constant.
n
x dx

x n 1
c, ( n 1)
n 1

(f ( x )) n f ( x ) dx

(f ( x )) n 1
C (n 1 )
n 1

sec x dx = ln |sec x + tan x| + c


dx

a2 x2
dx

1
x
tan1 c
a
a

x dx ln | x | c

1 x 2 tan

x
x
e dx e c

sin x dx = cosx + c

cos x dx = sinx + c

dx
a2 x2

x c

sin 1

x
x
c or cos1 + C
a
a

dx
1
1 x 2 = sin x + c

dx
x 2 1

= sec1x + c or cosec1 x + c

sec2 x dx = tanx + c
cosec2 x dx = cotx + c
tan x dx = ln|cosx| + c = ln |secx| + c
cot x dx = ln |sin x| + c = ln |cosecx| + c
1

e.g.

(i)

1
x2

x2
2
dx
+ c = . x3/2 + c
1
3
1
2

(ii)

1
x 21
1
2
dx

x
dx

c c
x2

2 1
x

The following points are to be noted:

x dx lnx + c if x is positive = ln (x) + c if x is negative because


1) =

1
1 x2

d
1
(ln (x)) =
(
dx
x

x dx ln | x | C

dx = sin1x or cos1x. It does not mean that sin1 x = cos1x.

The only legitimate conclusion is that they differ by some constant. In fact sin1x (cos
1
x)
= sin1x + cos1x = /2.

15

If a is a constant, then a f(x) dx = a f ( x ) dx

IIT- MATHS

[f(x) g(x)] dx, = f(x) dx g(x) dx

Methods of Integration
(i) Integration by Substitution
This method consists of expressing the integral f(x) dx, where x is the independent variable, in
terms of another integral where some other, say t, is the independent variable; x and t being connected
by the relation x =f (t). i.e., f(x)dx = f[j (t)] j (t) dt. This method is useful only when a relation x = (t)
can be so selected that the new integrand f(x)

dx
is of a form whose integral is known
dt

(ii) Integration by parts

f (x) (x) dx
(i )

( ii )

df

( x ) dx dx

dx

= f (x) ( x ) dx

Integral of the product of two function = first function integral of secondintegral of (derivative
of first integral of second).

Definite Integral
The difference in the values of an integral of a function f(x) for two assigned values of the independent variable x, say a, b, is called the definite integral of f(x) over the interval (a, b) and is denoted by
b

f ( x) dx. Thus

f ( x) dx F(b) F (a ),

where F(x) is the antiderivative of f(x). Or, we write

b
b

f (x ) dx | F( x) dx |a F(b) F(a ). a is called the lower limit and b the upper limit of integration.
a

Note:
b

f (x ) dx f (x) dx
a
b

b
c

f (x ) dx f ( x) dx f ( x) dx
a

where c is any point inside or outside the interval (a, b).

Geometrically definite integral represents area under curve.

16

BASIC TRIGONOMETRY

SOLVED SUBJECTIVE EXAMPLES


Example 1 :
In a DABC, the medians AD, BE and CF pass through G.
(a) If BG = 6, what is BE ?
(b) If FG = 4, what is GC ?
Solution:
(a) We have, BG =
6=

2
BE
3

2
BE BE = 9
3

(b) We have,
GC = 2 FG
GC = 2 4 = 8
Example 2 :
Triangles ABC and DBC are on the same base BC with A, D on opposite sides of lne BC, such
that ar. (DABC) = ar. (DDBC). Show that BC bisects AD.
Solution:
Since Ds ABC and DBC are equal in area and have a common side BC.
Therefore the altitudes corresponding to BC are equal
i.e.,AE = DF
Now, in Ds AEO and DFO,
We have
1 = 2 (vertically opposite angles)
AEO = DFO (90 each)
and AE = DF
A

1
B

DAEO @ DDFO (by A.A.S)


AO = OD
17

IIT- MATHS
BC bisects AD
Example 3 :
Solve |x2 3x 4| = x23x 4
Solution:
We know |x| = x when x 0
So, x2 3x 4 0
(x 4) (x + 1) 0 x 4 or x 1.
Example 4 :
Solve 184x3 = (54 2 )3x4
Solution:
Given equation is 184x3 = (54 2 )3x4
Taking log on both the sides, we get
(4x 3)log 18 = (3x 4) log(18.3

2 ) (since 3 2 =

18 )

or, (4x 3) log 18 = (3x 4) log (18)3/2 or, 4x 3 = (3x 4)

3
2

or, 8x 6 = 9x 12, or x = 6
Example 5 :
Solve for x if log3x + log9(x2) + log27 (x3) = 3
Solution:
log3x + log9(x2) + log27(x3) = 3
log x

2 log x

3 log x

3 log x

log 3 2 log 3 3 log 3 3 log 3 3 logx = log3


x=3

Example 6 :
If r be the ratio of the roots of the equation ax2 + bx + c = 0, show that

(r 1) 2 b 2

r
ac

Solution:
Let a and ra be the roots of the equation ax2 + bx + c = 0
So, a + ra =

b
b
b
a(1 + r) =
a = a (r 1)
a
a

Also, a ra =

r.

(1)

c
c
ra2 =
a
a

b2
c

[Using (1)]
2
2
a ( r 1) a

18

BASIC TRIGONOMETRY

(r 1) 2 b 2

r
ac

Example 7 :
If x is real, prove that 3x2 5x + 4 is always positive
Solution:

2
3x2 5x + 4 = 3 x 3 x 3

2
2

5 48 25
5 23
= 3 x
3 x 0 since square of real number is always non-negative.
6
36
6 36

Example 8 :
Find the area of the largest circle that can be inscribed in a square of side 14 c.m.
Solution:
BC = 14 c.m.
D
7

14
7 c.m.
radius of circle =
2

7
A

Now the area of circle = r 2 = (7) 2 = 49 c.m2


Example 9 :
x2
If f(x) =
, x R, find the range of f(x)
1 x2

Solution:
f(x) =

1
x2
x2 11
=
= 1
2
2
1 x2
1 x
1 x

Clearly f(x) [0, 1)


Example 10 :
If x = 2 ln cot t and y = tan t + cot t, find
Solution:
Since

19

dy dy / dt

dx dx / dt

dy
dx

IIT- MATHS
dx
cos ec 2 t
2
4

Now dt
cot t
cos t sin t sin 2t

dy
sin 2 t cos 2 t
cos 2 t
2
2

sec
t

cos
ec
t

4
Also
2
2
dt
sin t cos t
sin 2 2 t
dy 4 cos 2t sin 2 t cos 2 t

Hence dx
= cot 2t
2
sin 2t 4 sin 2 t

Example 11 :
sin 3 x cos 3 x
sin 2 x cos 2 x dx

Solution:
sin 3 x cos 3 x
sin 2 x cos 2 x dx =

sin 3 x
cos 3 x
dx

sin 2 x cos 2 x sin 2 x cos 2 x dx

= tanx secx dx + cotx cosecx dx = secx cosec x + c


Example 12 :
1/ 2

Evaluate:

sin 1 x
dx
(1 x 2 ) 3 / 2

Solution:
Let x = sinq dx = cos q dq and q = sin1x; when x = 0, sinq = 0
When x = 1 / 2 , sinq = p/4
1
2

sin x

1 x

2 3/ 2

dx

cos d
sec 2 d
cos 3
0

/ 4
/4
= tan 0 1. tan

d (Integrating by parts) =

1
ln 2
4 2

Example 13 :
Solve : log| x | | x | = 0
Solution:
We have
log| x | | x | = 0
|x| = 1

but | x | 1 (being in base of logarithm)


x

Example 14 :
20

BASIC TRIGONOMETRY
The maximum and minimum value of f (x) = 2x3 24x + 107 in the interval [1, 3].
Solution:
We have f(x) = 2x3 24x + 107
So, f (x) = 6x2 24
Now, f (x) = 0 6x2 24 = 0 x = 2
But x = 2 [1, 3]
So x = 2 is the only stationary point.
Now, (1) = 2 14 + 107 = 85, f(2) = 2(2)3 24 (2) + 107 = 75
And, (3) = 2 (3)3 24 3 + 107 = 89.
Hence, the maximum value of f (x) is 89 which attains at x = 3 and the minimum value is 75
which is attained at x = 2.
Example 15 :
Find area bounded by y = cosx, xcos , y-axis and x =
Solution:
/2

The represented area =

cos x dx
0

/2

sin x 0 1 0 = 1 sp. unit.

21

.
2

IIT- MATHS

SECTION - I
SUBJECTIVE
LEVEL - I
REVIEW YOUR CONCEPTS
1.

( x 3) 2 ( x 3) 5 ( x 1)
Find the solution to the inequality
>0
( x 5) ( x 4)

2.

Solve for x
(a) |x 4| > 7

3.

(b) |x| > x

(a)

Solve for x: log2x > 3

(b)

Which is greater: log23 or log1/25

4.

If the roots of (1 + m) x2 2 (1 + 3m) x + (1 + 8m) = 0 are equal, then find the value of m.

5.

For every x R, prove that 2x2 6x + 9 is always positive.

6.

Differentiate the following with respect to x.


(i)

sinx + cosx

(ii)

xlogx

(iii)

Differentiate sin2x + cos2x with respect to x

7.

d2 y
2
If y = acos t + sin t , then prove that 2 y 0
dt

8.

(a)

Find domain of the definition of the following:


(i)

(b)

(ii)

log e x

Draw the graph of the following:


(i) f(x) = x3

(ii) f(x) = logex

9.

Find the intervals of increase and decrease of (x 3) (x + 1).

10.

Evaluate : (i) (x 2 2x 1) dx .

(ii) (2xe x ) dx

/2

(ii)

(sin x cos x) dx
0

22

BASIC TRIGONOMETRY

LEVEL - II
BRUSH YOUR CONCEPTS
1.

3
2
5
Find the solution common to both the inequalities ( x 1) ( x2 3x 2)7 | x 4 | 0 & 1 < |x 3| < 5

2.

Solve for x

( x 4 x 4)

(i)

|x 3| + |x + 2| = 3

(ii)

(a) Solve for x,

x2
5
2x 7

1
(log a log b 4 log 2) .
2

3.

If a2 + 4b2 = 12 ab, then prove that log (a + 2b) =

4.

Find Domain of definition

5.

Differentiate cos (4x33x) with respect to x.

6.

If for the function h, given by h(x) = kx2 + 7x 4, h (5) = 97, find k.

7.

Find the intervals of increase and decrease of the function y = cosx, x

(i) sin

x3
x 1

(ii)

Integrate the following sin x . cos 2 x dx .


0

9.

In the figure given below, find the value of angle P


A
24
R

36
B

10.

54
C

In the figure given below, find the value of x


T
5
P

23

( x 2) ( x 4)

8.

loge ( x 1) ( x 3)

IIT- MATHS

LEVEL - III
CHECK YOUR SKILLS
1.

Solve: log0.3 (x 1) < log0.09 (x 1).

2.

Find the set of all solution of the equation 2|y| = 2 y1 1

3.

Evaluate: 7 log3 5 3log 5 7 5log 3 7 7 log 5 3

1
log 7 5

1
.
log10 0.1

4.

Evaluate:

5.

Find domain of definition (i) f(x) =

log 2 ( x 3)
x 2 3x 2

(ii) f(x) =

5x x 2

log
10
(iii)

5 cos x 3sin x cos

(iv) y

| x | 1
2 | x |
x2
1 x

x2
1 x

dx .
x

6.

Evaluate:

7.

Find the intervals of decrease and increase of (x + 2) ex .

8.

In the figure given below, ABCD is square and triangles BCX and DYC are equilateral triangle.
Find the value of y.

B
A

9.

(i) In the figure given below. Find QSR


Q

50

24

BASIC TRIGONOMETRY
(ii) In the given figure, O is the centre of the circle. If OCA = 26, then find ODB
C

O
A

10.

25

ABC is a triangle in which BC is produced to D. CA is produced to E, DCA = 108 and EAB


= 124. Then find ABC.

IIT- MATHS

SECTION - II
OBJECTIVE
LEVEL - I
1.

If log16x + log4x + log2x = 14, then x =


(a) 16
(c) 64

2.

If | 4 3x |

(b) 32
(d) none
1
then x is equal to
2

7 3

(a) ,
6 2
7 3

(c) ,
6 2
3.

(d) none of these

The product of all the roots of the equation x 2 - |x| 6 = 0 is

(a) 9
(c) 9
4.

7 3
6 2

(b) ,

(b ) 6
(d) 36

The value of p for which (x 1) is a factor of x3 + (p + 1)2 x2 10 is given by


(a) 4, 2
(c) 2, 4

(b) 2, 4
(d) none of these
1

5.

The domain of definition of the function f(x) =


(a) R
(c) (, 0)

6.

log x
x

(a) log x

(b)

(c) (xlogx)1

(d) xlogx

The function f(x) = tanx x,


(a) always increases
(c) never decreases

8.

(b) (0,)
(d) none of these

The differential coefficient of f(x) = log (logx) with respect to x is


x

7.

x x is

(b) always decreases


(d) some times increases and some times decreases

The function f(x) = x3 3x is


(a) increasing on (, 1] [1, ) and decreasing on (1, 1)
26

BASIC TRIGONOMETRY
(b) decreasing on (, 1] [1, ) and increasing on (1, 1)
(c) increasing on (0, ) and decreasing on (, 0)
(d) decreasing on (0, ) and increasing on ( , 0)
9.

The minimum value of 2( x 2 3)2 27 is


(a) 227
(c) 1

10.

(b) 2
(d) none of these

The maximum value of x3 3x in the interval [0, 2] is


(a) 2
(c) 2

(b) 0
(d) 1
x/4

11.

Evaluate

12.

sin x
dx
cos 3 x 3 cos x

(a)

1
2

(b)

1
6

(c)

1
8

(d)

1
12

If log10 3 = 0.477, the number of digits in 340 is


(a) 18
(c) 20

13.

(b) 19
(d) 21

The interior and its adjacent exterior angle of a triangle are in the ratio 1 : 2. What is the sum of
the other two angles of the triangle?
(a) 112
(c) 120

14.

(b) 110
(d) 90

In the figure PAQ is a tangent of the circle with centre O at a point A if OBA = 32. The value
of x and y is
P

C
y

O
x
B

27

IIT- MATHS
(a) 30, 50
(c) both 58
15.

(b) both 40
(d) 30, 60

The difference between the interior and exterior angles of a regular polygon is 132. The number
of sides of the polygon is
(a) 12
(c) 15

(b) 8
(d) 20

28

BASIC TRIGONOMETRY

LEVEL - II
1.

If A = log2log2log4256 + 2 log 2 2, then A equals to


(a) 2
(c) 5

2.

(b) 3
(d) 7

If logkA . log5k = 3, then A =


(a) 53
(c) 12

3.

(b) k3
(d) 243

If the product of the roots of the equation x 2 5 x 4 log 2 0 is 8, then l is


(a) 2 2
(c) 3

(b) 2 2
(d) none of these

4.

The number of real roots of the equation (x 1)2 + (x 2)2 + (x 3)2 = 0


(a) 3
(b) 2
(c) 1
(d) 0

5.

If a and b are roots of the equation x2 + x + 1 = 0 then a2 + b2 =


(a) 1
(b) 2
(c) 1
(d) 3

6.

x2 9
The domain of definition of the function f (x) =
log x

(b) (1, )
(d) [1, )

(a) R
(c) (0, 1) (1,

7.

dy
If x = a cos , y = a sin , 1+ =
dx
(a) tan2
(b) sec2
(c) sec
(d) |sec|
3

8.

The function y = x3 3x2 + 6x 17


(a) increases everywhere
(b) decreases everywhere
(c) increases for positive x and decreases for negative x
(d) increases for negative x and decreases for positive x

9.

The largest value of 2x3 3x2 12x + 5 for 2 x 4 occurs at x =


(a) 2
(b) 1
(c) 2
(d) 4

10.

The greatest value of f(x) = cos (xex + 7x2 3x), x [ 1, ) is


(a) 1
(b) 1
(c) 0
(d) none of these

29

IIT- MATHS

11.

ex
1 e 2 x dx

(a) cot1ex+c
(c) tanex + c

(b) tan1ex + c
(d) sin1ex + c

x/4

12.

Eva luate

tan 2 x dx

13.

(a) 3

(b) 2

(c) 1

(d)

5
2

If the area of parallelogram ABCD is 32 sq. cm. Area of DAMN is equal to


A

(a) 8 cm2
(c) 4 cm2

14.

(b) 2 cm2
(d) none of these

In the given figure if AX = 5 cm, XD = 7 cm, CX = 10 cm find BX


B

A
x
C

(a) 3 cm
(c) 4 cm

15.

(b) 3.5 cm
(d) 4.5 cm

If A, B, C are three consecutive points on the arc of a semicircle such that the angles subtended by
the chords AB and AC at the centre O is 90 and 100 respectively. Then the value of angle BAC
is equal to
(a) 5
(c) 20

(b) 10
(d) 30

30

BASIC TRIGONOMETRY

SUBJECTIVE SOLUTIONS
LEVEL - I

(CBSE Level)

CHECK YOUR SKILLS


1.

( x 3)2 ( x 3)5 ( x 1)
0
Find the solution to the inequality
( x 5)( x 4)

Solution:

(x 3)2 (x 3)5 (x 1)
0
(x 5) (x 4)

By wavy curve method,

x ( , 5) ( 3 1) (4 )

2.

Solve for x
(a) |x 4| > 7
Solution:
(a)

(b) |x| > x

|x 4| > 7
x 4 > 7 or x 4 < 7
x > 11 or x < 3
3

x ( , 3) (11, )
(b) |x| > x
Case I : x > 0,
x > x which is not possible.
Case II: x < 0
x < x 2x < 0 x < 0
x ( , 0) or R

3.

(a)
(b)
Solution:
(a)

Solve for x: log2x > 3


Which is greater: log23 or log1/25
log2x > 3

x > 23 [ base is greater than 1]


31

11

+
4

IIT- MATHS
(b)

x > 8 x (8, )
log23 or log1/2 5
log1/2 5 = log25 < 0. [ log1/a x = logax]

log23 is greater than log1/2 5.

4.
If the roots of (1 + m) x2 2 (1 + 3m) x + (1 + 8m) = 0 are equal then find the value of m.
Solution:
Given equation is
(1 + m) x2 2(1 + 3m) x + (1 + 8m) = 0
roots are equal , then D = 0

4 (1 + 3m)2 4 (1 + m) (1 + 8m) = 0

1 + 9m2 + 6m 1 9m 8m2 = 0

m2 3m = 0

m (m 3) = 0

m = 0, 3.

5.
For every x R, prove that 2x2 6x + 9 is always positive.
Solution:
Let f(x) = 2x2 6x + 9 = 2(x2 3x + 9/2)
= 2( x 3/2)2 + 9/4 > 0 [ square of real number is always nonnegative]
Hence f(x) is always positive.
6.

Differentiate with respect to x


(i)
sinx + cosx
(ii)
xlogx
Solution:
(i)

(ii)

Let y = sinx + cosx

dy
cos x sin x
dx

d
(sinx + cosx) = cosx sinx
dx

Let y = x logx
dy
d
d
1
x (log x) log x (x) = x log x1
dx
dx
dx
x
d
(x log x) = 1 + logx
dx

8.

(a)

Find domain
(i)

(b)

Draw graph
(i) f(x) = x3

(ii)

log e x

(ii) f(x) = logex


32

BASIC TRIGONOMETRY
Solution:
(a)

(i) Let f(x) =

x
f(x) is real for all x 0
Df = [0, ]
(ii) Let f(x) =

log e x

f(x) is real for all x 1


Df = [1, )

(b)
Let f(x) = x3
It is odd function so graph will be symmetric about the origin
(ii)
Df R
(iii) Rf R
(iv)

f (x) = 3x2
f (x) = 0 x = 0

(v)

f (x) = 6x

(vi)

ve, x 0
f (x) 6x
ve, x 0

(ii)

Let f(x) = logx

y = logx

(1, 0) x

1
1
(i) f (x) = , f (x) = 2
x
x

(ii) Df R {0}
(iii) Rf R
9.

Differentiate with respect to x


(i)
sin2x + cos2x
(ii)

Solution:
(i)

x = a (cost + log tan

t
), y = a sint.
2

Let y = sin2x + cos2x

dy d
d
d
d
(sin 2 x)
(cos 2 x) (cos x)
(sinx) +
dx dx
dx
dx
dx

= 2sinx cosx + 2cosx (sinx) = 2sinx cosx 2sinx cosx = 0

33

d
(sin 2 x cos 2 x) 0
dx

d
d
(1) 0 [
(constant) = 0]
dx
dx

(ii)

x = a (cost + log tan

t
),
2

y = asint

IIT- MATHS
Diff. w.r.to t
1
t 1

sin t
sec 2 dy
dx

t
2 2 ,
a
a cos t
tan

dt
dt

= a[ sint + cosect]
dy
cos t

dx cos ect sin t

cos t.sin t
cos t.sin t
=
= tant
2
1 sin t
cos 2 t

10.
Find the intervals of increase and decrease of (x 3) (x + 1).
Solution:
Let f(x) = (x 3) (x + 1) = x2 2x 3
f (x) 2x 2 = 2(x 1)

for increasing, f (x) 0

2(x 1) 0 x 1
x [1, )

for decreasing, f (x) 0

x ( ,1]

11.

Find the point of local maxima and minima of the following


(i) x2 + 3x
(ii) logex + x
Solution:
Let f(x) = x2 + 3x = x(x + 3)
f (x) = 2x + 3, for local maxima or local minima we must have, f (x) = 0

3
2

f (x) = 2 > 0

3/2

f(x) is local minima at x

(ii)

f(x) = logx + x
f (x)

3
2

1
1
x

for local maxima or local minima, we must have f (x) 0

1
1 0 x = 1
x
1
0
x2
f(x) is local maxima at x = 1.
f (x)

34

BASIC TRIGONOMETRY
12.
Integrate the following
(i)

(ii) 2 xe x2

x2 + 2x + 1
x/2

(iii)

(sin x cos x ) dx

(iv)

x dx

Solution:
(i)

Let I = (x 2 2x 1) dx
x 2 dx 2 x dx 1.dx

(ii)

x3
x2
2 x C
3
2

x3
x 2 x C, where C is integral constant.
3

Let I =

2x e

x2

dx

Put x2 = t
2xdx = dt
2

x
t
I e dt = et + C = e C, where C is integral constant.
/2

(iii)

(sin x cos x) dx
0

/2

/2

sin x dx

cos x dx
0

= cos x 0 / 2 [sin x]0 / 2

= cos cos 0 sin sin 0 = [0 1] + [1 0] = 2

2
2

(iv)

Let I | x |dx
1

x, x 0
Let f(x) = |x| =
x, x 0
0

I f (x)dx f (x) dx
1

2
2
xdx xdx x x
2
1
0
1 2 0

1
1
[0 1] (4 0)
2
2

13.
35

5
2

In the figure given below. Find QSR

IIT- MATHS
Q

50

Solution:
QSR + PRQ + QPR + QOR = 360
90 + 90 + 50 + QOR = 360
QOR = 130
1
QSR = QOR = 65
2
14.

In the given figure, O is the centre of the circle. If OCA = 26, then find ODB
C

O
A

Solution:
OCA = 26 = DBA
OB = OD

OBD = ODB = 26

B
26

26

O
A

ABC is a triangle in which BC is produced to D. CA is produced to E, DCA = 108 and


EAB
= 124. Then find ABC.
Solution:

DCA = 108
E

ACB = 72
124 A
56

EAB = 124
108
BAC
=
56
D
C

ABC = 180 (72 + 56) = 62


15.

36

BASIC TRIGONOMETRY

LEVEL - I
BRUSH UP YOUR CONCEPTS
1.

Find the solution common to both the inequalities


( x 1)3 ( x 2 3x 2) 5 | x 4 |
0 & 1 < |x 3| < 5
( x 2 4 x 4) 7

Solution:
Case I : x < 4

(x 1)3 (x 2)5 (x 1)5 (x 4)


0
(x 2)14

(x 1)3 (x 2)5 (x 1)5 (x 4)


0
(x 2)14

x ( , 4)
Case II : x > 4

+
4

Similarly, x (4 2) ( 1,1)
... (i)
x ( , 4) ( 4, 2) ( 1,1)
And 1 < | x 3| < 5
Case I : x < 3
1 < x + 3 < 5 2 < x < 2 2 < x < 2

x (2, 2)
Case II: x > 3
1<x3<5 4<x<8
x (4,8)

... (ii)

x ( 2, 2) (4,8)

from (i) & (ii)


4

2.

x ( 1,1)

Solve for x
(i)

|x 3| + |x + 2| = 3

Solution:
(i) |x 3| + |x + 2| = 3
Case I:
37

(ii)

x2
5
2x 7

IIT- MATHS
x<2
x + 3 x 2 = 3
2x = 2 x = 1, which is impossible
Case II:
2 x < 3
x + 3 + x + 2 = 3, which is not possible
Case III:
x3
x3+x+2=3
2x = 4
x = 2, which is impossible
x

(ii)

x2
5
2x 7

x 2 5(2x 7)
x 2 10x 35
0
0
2x 7
2x 7

9x 37
0 (9x + 37) (2x + 7) > 0
2x 7

+
37/9

3.

x ( ,

+
7/2
37
7
) ( , )
9
2

If a2 + 4b2 = 12 ab, then prove that log (a + 2b) =

1
(log a log b 4log 2)
2

Solution:
a2 + 4b2 = 12ab
a2 + 2a2b + 4b2 = 16 ab
(a + 2b)2 = 16ab
taking log on both the sides,
2 log (a + 2b) = log16 + log a + logb

4.

log (a + 2b) =

Find Domain

1
(loga + logb + 4log2)
2

(i) sin

x3
x 1

(ii)

( x 1)( x 3)
loge

( x 2)( x 4)

Solution:

x 3
x 1
f(x) is defined for

(i) Let f(x) = sin

x 3
0 (x 1) (x + 3) 0
x 1

38

BASIC TRIGONOMETRY
x

x ( ,3] (1, )

(ii)

Let f(x) = log e

(x 1) (x 3)
(x 2) (x 4)

f(x) is defined for


(x 1) (x 3)
0
(x 2) (x 4)
x ( , 3) ( 2,1) (4, )

5.

+
2

+
4

Differentiate with respect to x


(i)

(ii)
(iii)
Solution:
(i)

x sin x
+ log
(1 x 2 )

(1 x 2 )

(sinx)x
cos (4x33x)

Let y

x sin x
log 1 x 2
2
1 x

dy (1 x 2 ) (sin x x cos x) (2x)x sin x


1
(x 2x)

2 2
dx
(1 x )
1 x2 2 1 x2

(ii)

(1 x 2 ) x cos x (1 x 2 ) sin x 2x 2 sin x


1

2
(1 x )2
1 x2

(1 x 2 ) x cos x (1 x 2 ) sin x
1

2 2
(1 x )
1 x2
Let y = (sinx)x. Taking log on both the sides, we get
logy = x log sinx
1 dy
x cos x
. log sin x
= x cotx + log sinx
y dx
sin x
dy
= (sinx)x [x cotx + log sinx]
dx
d
{(sin x) x } (sin x) x[x cot x log sin x]
dx
Let y = cos(4x3 3x)

(iii)

dy
= sin (4x3 3x) [12x2 3]
dx

d
(cos 4x3 3x) = 3 (1 4x2) sin (4x3 3x)
dx

6.
If for the1 function h, given by h(x) = kx2 + 7x 4, h (5) = 97, find k.
Solution:
39

IIT- MATHS

7.

h(x) = kx2 + 7x 4
h(x) 2kx + 7

h(5) = 10k + 7
97 = 10k + 7 k = 9 [ h (5) = 97]

(i)

Find the intervals of increase and decrease of the function y = cosx,

x
2

(ii)

Find the point of local maxima & minima of the function, f (x) = x

1
.
x

Solution:
(i)

Let f(x) = cosx, x


2
f (x) = sinx
f(x) increases, if f (x) 0 sinx 0 sinx 0

x , 0
2

/2

f(x) decreases, if f (x) 0


sinx 0 sinx 0
x [0, ]
(ii)

1
f (x) x
x

f (x) 1

f (x)

/2

1
x2

2
x3

For local maximum or local minimum we must have f (x) 0

1
0 x = 1
x2
at x = 1, f (x) 1 0

Hence local minima at x = 1


at x = 1, f (x) 1 0
Hence local maxima at x = 1.

8.

Integrate the following (i)

sin x . cos
0

x dx

e x
(ii) x 2 / 3 dx .
1

Solution:
/ 2

(i)

Let I =

sin x.cos

x dx

put cosx = t = t 2 dt sinx dx = dt when x = 0, t = 1, when x =


1

, t 0
2

40

BASIC TRIGONOMETRY
1

t3 1
t
dt

=
3
3 0
0
2

(ii)

e x
1 1
Let I = 2 / 3 dx put x1/3 = t . 2 / 3 dx dt when x = 8, t = 2 ,
x
3 x
1
2
t
t 2
x = 1, t = 1 = 3 e dt 3(e ) , = 3(e2 e) = 3e (e 1)
1

9.

In the figure given below, find the value of angle P


A
24
R

36
B

54
C

Solution:
ACD = ABC + BAC = 36 + 24 = 60
P = RCD + RDC = 60 + 52 = 104.

10.

In the figure given below, find the value of x


T
5
P

Solution:
PT2 = PA.PB
5 5 25

4
4
AB = PA PB

PA =

x=

25
9
4 units
4
4

Hence x =

41

9
units.
4

IIT- MATHS

LEVEL - III
CHECK YOUR SKILLS
1.
Solve, log0.3 (x 1) < log0.09 (x 1)
Solution:
log0.3 (x 1) < log0.09 (x 1)

1
log0.3 (x 1) <
log0.3 (x 1)

1
log0.3 (x 1) < 0
2
x1>1 x>2
x (2, )

Find the set of all solution of the equation 2 y 2 y 1 1


Solution:
2|y| = 2y1 + 1

Case I: y < 0
2y = 2y1 + 1

2.

2 2y
2 y (2y)2 + 2.2y 2 = 0
2
a2 + 2a 2 = 0 , where a = 2y
1=

2 4 8 2 2 3

2
2

a = 1 3 , 1 +

a=1

a=

2y =

log22y = log2 ( 3 1 )

3 , which is not possible

3 1
3 1

y = log2 ( 3 1 )

Case II: y < 0


2y = 2y1 + 1
2.2y = 2y + 2

2y = 2

y=1

set of solution 1, log2 ( 3 1)

3.
Evaluate: 7 log 3 5 3log 5 7 5log 3 7 7 log 5 3
Solution:
5

Let y = 7log3 3log5 5log3 7 log5

= 7log3 5 3log5 7 7 log3 5 3log 5 7 [ alogb c clogb a ]


42

BASIC TRIGONOMETRY
=0
Hence the result.
1

4.

Evaluate: 3 2 log 75

1
log 100.1

Solution:
3

Let y 5

1
1

log 7 5
log10 0.1

1/ 3

1
log 7

5 5

log10 10

Hence the result.

5.

= (7 + 1)1/3 = (8)1/3 = 2

Find domain (i) f(x) =

(ii) f(x)

x 1
2 x

Solution:
log 2 (x 3)
x 2 3x 2
Let g(x) = log2 (x + 3) and h(x) = x2 + 3x + 2
g(x) is defined for
x+3>0 x>3
Dg = (3, )

And h(x) is defined for x2 + 3x + 2 0 (x + 2) (x + 1) 0


Dh = R {2, 1}

Df = Dg Dh = (3, ) R {2, 1} = (3, ) {2, 1}

(i) f(x) =

(ii)

| x | 1
f(x) is defined for
2 | x |

f(x) =

| x | 1
(| x | 1) (2 | x |)
0
0
2 | x |
(2 | x |) 2

6.

(2 |x|) (|x| 1) 0
2 |x| > 0 or |x| 1 0
|x| < 2 or |x| 1
2 < x < 2 or x 1 or x 1
Df = (2, 1] [1, 2)

Find the domain


5x x 2

(i) y log10
(iii) y

x2
1 x

x2
1 x

Solution:
(i)

43

y log10 5x x f (x)
4

(ii) y cos ecx (sin x )1 / 3

IIT- MATHS
f(x) is defined for
5x x 2
log10
0
4

(ii)

5x x 2
1 5x x 2 4 0
4
x2 5x + 4 0
(x 1) (x 4) 0
either x 1 or x 4
Df = [1, 4]
Let f(x) =

cos ecx + (sinx)1/3

Let g(x) = cos ecx and h(x) = (sinx)1/3


g(x) is defined for
cosecx > 0
Dg = (2n , (2n + 1) ).
And h(x) is defined for R
Df = Dg Dh = (2n , (2n + 1) )
(iii)

Let f (x)

x2
1 x

x2
1 x

x2
and h(x) =
x2
g(x) is defined for
x 2
0 either x 2 0
x2
or x + 2 < 0
x 2 or x + 2 < 0
Dg = ( , 2) [2, )
h(x) is defined for
Let g(x) =

1 x
1 x

1 x
x 1
0
0
1 x
x 1
either 1 x 0 or 1 + x > 0

x 1 or x > 1
Dn = (1, 1]
Df = Dg Dh =
7.

Integrate the following


(a)

q
5 cos x 3 sinx +
cos 2 x

(b)

2 3 sin x
dx
cos 2 x
0

Solution:
(a) Let I = (5 cos x 3sin x

a
)dx
cos 2 x

5 cos x dx 3 sin x dx 9 sec 2 x dx


44

BASIC TRIGONOMETRY
= 5 sinx + 3cosx + 9 tanx + C,
where C is integral constant.
/4

(b)

Let I

2 3sin x
dx
cos 2 x

/4
2
= 2 sec xdx 3 tan x sec x dx
0

= 2[tan x]0 / 4 3[sec x]0 / 4 = 2[1 0] + 3 [ 2 1] = 3 2 1


8.
Differentiate e
Solution:
Let y = e
dy
e
dx

9.

(a)
(b)

x 2 1

w.r.t. x.

x 2 1

x 2 1

2 x2 1

(2x 0)

x.e

x 2 1

x2 1

Find the intervals of decrease and increase of (x + 2) ex .


Find the greatest and least values of the following functions on the intervals
y = 3x4 + 6x2 1
(2 x 2)
y=

x3
2 x 2 3x 1
3

(1 x 5)

Solution:
(a) Let f(x) = (x + 2) ex
f (x) = ex (x + 2) + ex = ex (x + 1)
for decreasing f (x) 0
ex (x + 1) 0

ex (x + 1) 0 ( ex 0]

x+1>0

x > 1 x [ 1, )
for increasing , f (x) 0
ex (x + 1) 0

ex (x + 1) 0
x+1 0
[ ex 0]

x 1

x (, 1]

(b) (i) y = 3x4 + 6x2 1, 2 x 2


dy
12 x 3 12x

dx

d2 y
36x 2 12 12(x 2 1) 0
2
dx

45

dy
0 12x (x2 1) = 0 x = 0, 1 2
dx
least value y = 2, at x = 0

IIT- MATHS
greatest value y = 25 at x = 2
(ii)

x3
2x2 + 3x + 1 , 1 x 5
3

dy
d2 y
x 2 4x 3
2x 4 2(x 2)
dx
dx 2

dy
0 x2 4x + 3 = 0
dx
(x 1) (x 3) = 0 x = 1, 3, 5

d2 y
20
dx 2
maximum at x = 1

maximum value of y =

at x = 1,

7
3

d2 y
20
at x = 3,
dx 2
minimum at x = 5

10.

minimum value of y

23
3

In the figure given below, ABCD is square and triangles BCX and DYC are equilateral
triangle. Find the value of y.
B
A

Solution:

BCX and DYC are equilateral


AB = BC = CD = DA = CX = DY = CY = BX
CBX = 60
ABX = 150
BAX = 15
A [ AD DY]
DAY = 75 = DYA

46

BASIC TRIGONOMETRY
Illustration 1:
If A = {a, b, c} and B = {b, c, d} then evaluate AB, AB, A B and B A
Solution:
AB = {x : x A or x B} = {a, b, c, d}
AB = {x : x A and x B} = {b, c}
A B = {x : x A and x B} = {a}
B A = {x : x B and x A} = {d}
Illustration 2:
Find the logarithms of 0.0625 to the base 2.
Solution:
Suppose 2x = 0.0625 =
or

1
1
4
16 2

2x = 24 or x = 4

Illustration 3:
Solve the equation a2x b3x = c5, where a, b, c R+
Solution:
Equation is a2x.b3x = c5
Taking log on both sides, we have
2x log a + 3x logb = 5 logc or x (2loga + 3 logb) = 5logc

5 log c

x = 2 log a 3 log b

Illustration 4:
Solve for x, log1/2 (x 2) > 2.
Solution:
1
4

1
4

9
4

log1/2(x 2) > log1/2 0 < x 2 < 2 < x <


So

9
4

x 2,

Illustration 5:
Solve |2x 1| < 3.
Solution:
|2x 1| < 3 3 < 2x 1 < 3

2 < 2x < 4 1 < x < 2.


Illustration 6:

47

IIT- MATHS

x 3x 2 x 5
x 1x 7 then find x such that

If f(x) =

(i) f(x) > 0.


Solution:
Given f(x) =

(ii) f(x) < 0.

x 3x 2 x 5
x 1x 7

Illustration 7:
x

If x x = 24,, then find the value of x.


Solution:
x

Here, x x = 24 x2 5x = 24
x2 8x + 3x 24 = 0 (x 8) (x + 3) = 0 x = 3, 8
Illustration 8:
2

Find the value of x and y when y x = 4 and 2 1 2


Solution:
2 3

Since, y x 4 2x 3y 4
x

(1)

and 2 1 2 x 2 y 2

(2)

Solving (1) and (2), we get x =

5
, y = 3.
2

Illustration 9:
Find the derivative of y =

3x ( x )1 / 3

1
x

Solution:
Here we have y = u + v + w,
where u = 3x , v x 1 / 3 and w =

1
x

Hence we can use theorem 2


1

dy
1 1 1 1
1
3 x 2 x 3 1.x 11
dx
2
3
2

3 1 1
1

2
2/3
x 3 (x)
x

Illustration 10:
Find the derivative of y = (a + x) ex w.r.t. x .
Solution:
48

BASIC TRIGONOMETRY
Using theorem 3
dy
d
d
(a x ) e x e x
(a x )
dx
dx
dx

= (a x ) e x e x .1 e x (a x 1)
Illustration 11:
Find the derivative of y =

ax
w.r.t. x
ax

Solution:
Illustration 12:
Differentiate the following w.r.t x.
(i) y = x x

(ii) y = (sin x ) x
Solution:
(i) y = xx

dy
x.x x 1 .1 x x ln x.1 x x (1 ln x )
dx

(ii)

y (sin x ) x

2
2
dy
d
dx 2
x 2 (sin x ) x 1
(sin x ) (sin x ) x ln sin x
dx
dx
dx

= x2 (sin x )

x 2 1

cos x (sin x ) x ln sin x. 2x.

Illustration 13:
Find the derivative of the following functions w.r.t. x
(i)
y = sinx2
(ii)
y = (lnx)3
(iii)
y = sin (lnx)3
(iv)
y = cos1 (lnx)
Solution:
(i)
y = sin (x)2. Let u = x2 y = sin u

dy
d( u 3 ) du
1 3
3u 2 (ln x ) 2
=
dx
du dx
x x

(ii)

y = (ln x)3. Let u = ln x y = u3

(iii)

y = sin (lnx)3 Let u = ln x, v = u3 y = sin v

(iv)

dy dy dv du d(sin v) d( u ) 3 d
1 3(ln x ) 2

(ln x ) = cos v. 3u 2
cos [(ln x ) 3 ]
dx dv du dx
dv
du dx
x
x

Let u = ln x = cos1 u

49

dy d
d 2

(sin u )
( x ) cos u.2 x 2x cosx2
dx du
dx

dy
1 du
1
1
1

dx
1 u 2 dx
1 u 2 x x 1 (ln x ) 2

IIT- MATHS
Illustration 14:
The function y of x is given by, x = a cos t, y = a sin t. Find the derivative of y w.r.t. x.
Solution:
dy (a sin t ) cos t

cot t
dx (a cos t ) sin t

If we want to compute

dy
dy

cot 1
at a particular t, say t = , then dx
4
x / 4
dx
4

Illustration 15:
d2y
Find
, where y = sin2x
dx 2

Solution:
dy
= 2sinxcosx = sin2x
dx

Illustration 16:
Find the interval of increase and decrease of the function y = x4.
Solution:
y = x4 y = 4x3
For x > 0, y > 0 the function increases in (0, ).
For x < 0, y < 0 the function decreases in (, 0)
Illustration 17:
Separate the intervals in which f(x) = 2x3 15x2 + 36x + 1 is increasing or decreasing
Solution:
We have f (x) = 6x2 30x + 36 = 6 (x 2) (x 3)
Thus for x < 2, f (x) > 0
over 2 < x < 3, f (x) < 0 and for x > 3, f (x) > 0
Hence the given function is increasing in (, 2) and (3, ), and decreasing in (2, 3)
Is tanx always increasing x R ?
Illustration 18:
Test y = 1x4 for maximum and minimum
Solution:
Here y = 4x3 = 0 for x = 0

x = 0 is the critical point.


Now y = 12 x2 = 0 at x = 0
It is thus impossible to determine the character of the critical point by means of the sign of the
second derivative. Thus we investigate the character of the given function in an interval containing point
x = 0.
For x < 0, y > 0 the function is increasing for x < 0
For x 0, y < 0 the function is decreasing for x > 0.
Consequently, at x = 0, the function has a maximum i.e., yx = 0 = 1

50

BASIC TRIGONOMETRY
Illustration 19:
Determine the maximum and minimum of the function y = x3 3x + 3 on the interval [3,

3
].
2

Solution:
For the given function, y = 3x2 3
For the critical points, 3x2 3 = 0 x = 1
Then y = 6x > 0 at x = 1, y < 0 at x = 1
Hence there is maximum at x = 1 at which y = 1 + 3 + 3 = 5
Also there is minimum at x = 1 at which y = 13 + 3 = 1
Now at x = 3, y = 27 + 9 + 3 = 15
and at x = 3/2, y = 15/8
Hence the minimum value of the given function is 15 at x = 3 and the maximum value is 5 at
x = 1. It should be noted that the values are actually the largest and smallest values of the function
in the given interval.
Illustration 20:
Evaluate:

(i)

(a0 + a1x + a2x2 ) dx

(ii)

cos x x e

(iii)

x2
1 x 2 dx

(iv)

x4
x 2 1 dx

Solution:
(i)

dx

(a0 + a1x + a2x2) dx = a0 dx + a1 x dx + a2 x2 dx


x2
x3

a
c
= a0 x + a1
2
2
3

x
dx cos x dx 2 dx e dx = sin x + 2 log |x| ex + c
x

(ii)

cos x x e

(iii)

x2
x2 11
1
dx

dx

1 x 2 1 x 2 dx 1 1 x 2 dx dx 1 x 2 = x tan1 x + c

(iv)

x4
x4 11
1
2
dx

x 2 1 1 x 2 dx x 1 1 x 2 dx
2
= x dx dx

dx
x3

x tan 1 x c
1 x2 3

Illustration 21:
Integrate the following w.r.t. x.
(i)
sin2x cosx
(ii)
Solution:
(i)
51

x3
1 x8

Let sin x = t cos x dx = dt

IIT- MATHS
2
2
sin x cos x dx t dt

(ii)

t3
sin 3 x
c
c
3
3

Let x4 = t = 4 x3 dx = dt x3 dx = dt
x3
1
1
dt
1
1
dx
dt =
tan 1 t c tan 1 x 4 c

8
2
2
1 x
4(1 t )
4 1 t 4
4

Illustration 22:
1

Evaluate

a x2

dx

Solution:
Let x = a sin dx = a cos d a2 x2 = a2 cos2

Note:

dx
2

a x

a cos d
x
1. d sin 1 c
a cos
a

1
bx c
sin 1
a
a 2 ( bx c) 2 b
dx

Illustration 23:
Evaluate
(i)

xe

(ii)

dx

sin

x dx

Solution:
(i)

let f(x) = x, (x) = ex xe x dx x.e x 1.e x dx xe x e x c

(ii)

Let f(x) = sin1x. (x) = 1


1
1
1
sin x dx sin x. 1 dx sin x.x

1
1 x2

x dx

= x sin1 x x (1 x 2 ) 1/ 2 dx x sin 1 x 1 x 2 c
Note: e x (f ( x ) f ( x )) dx e x f ( x ) dx e x f ( x ) dx
= f(x) ex f ( x ) e x dx e x f ( x ) dx f ( x ) e x c
Illustration 24:
Evaluate :
/ 2

cos x

1 sin 2 x dx
0

Solution:
Let sin x = t cos x dx = dt
When x = 0, t = 0, x = /2, t = 1
/2

cos xdx
dt

(tan 1 t )10 = tan1 1 tan1 0 = 0 = /4


2
2
1 sin x 0 1 t
4

52

BASIC TRIGONOMETRY

53

IIT- MATHS

ARITHMETIC PROGESSION

54

ARTHMETIC PROGESSION

SQUENCE AND SERIES


A succession of numbers a1, a2, a3..., an, ... formed, according to some definite rule, is called a
sequence.

ARITHMETIC PROGESSION (A.P.)


A sequence of numbers {an} is called an arithmetic progression, if there is a number d, such that d
= an-an1 for all n. d is called the common difference (C.D.) of the A.P.

(i) Useful Formulae


If a = first term, d = common difference and n is the number of terms, then
(a) nth term is denoted by tn and is given by
tn = a + (n 1) d.
(b) Sum of first n terms is denoted by Sn and is given by
n
Sn [2a (n 1)d]
2
n
or S n (a l ) , where l = last term in the series i.e., l = tn = a + (n 1) d.
2
(c) Arithmetic mean A of any two numbers a and b
a b
A
.
2
(d) Sum of first n natural numebrs ( n )
n(n 1)
n 2 , where n N .
(e) Sum of squares of first n natural numbers ( n 2 )

n( n 1) (2n 1)
6

(f) Sum of cubes of first n natural numbers (n 3 )


n(n 1)
n 3

(g) Middle term: If the number of terms is n, and


th

n 1
n is odd, then
term is the middle terms
2
th

th

n
n
n is even, then and 1 terms are middle terms.
2
2

(h) If terms are given in A.P., and their sum is known, then the terms must be picked
up in following way
For three terms in A.P., we choose them as (a d), a, (a + d)
For four terms in A.P. , we choose them as (a 3d), (a d), (a + d), (a + 3d)
55

IIT- MATHS
For five terms in A.P., we choose them as (a 2d), (a d), a, (a + d), (a + 2d) etc.

(ii) Useful Properties


(a) If tn = an + b, then the series so formed is an A.P.
(b) If Sn = an2 + bn + c, then series so formed is an A.P.
`

(c) If every term of an A.P. is increased or decreased by some quantity, the resulting terms
will also be in A.P.
(d) If every term of an A.P. is multiplied or divided by some non-zero quantity, the result
ing terms will also be in A.P.
(e) In an A.P. the sum of terms equidistant from the beginning and end is constant and
equal to sum of first and last terms.
(f) Sum and difference of corresponding terms of two A.P.s will form an A.P.
(g) If terms a1, a2, ..., an, an+1, ..., a2n+1 are in A.P., then sum of these terms will be equal to
(2n + 1)an+1.
(h) If terms a1, a2, ..., a2n1, a2n are in A.P. The sum of these terms will be equal to
a n a n 1
(2n)
.
2

GEOMETRIC PROGRESSION (G.P.)


A sequence of the numbers {an}, in which a1 0 , is called a geometric progression, if there is a
an
number r 0 such that a r for all n.r is called the common ratio (C.R.) of the G.P..
n 1

(i) Useful Formulae


If a = first term, r = common ratio and n is the number of terms, then
(a) nth term, denoted by tn , is given by

tn = arn1

(b) Sum of first n terms denoted by Sn is given by


a rl
a(1 r n )
a(r n 1)
Sn
or
or
,
1 r
1 r
r 1
where l is the last term (the nth term) in the series, r 1 In case r = 1, Sn = na.
(c) Sum of infinite terms (S )
a
S
(for | r | 1)
1 r
Note: When |r| 1, the series is divergent and so its sum is not possible.
(d) Geometric mean (G.M.)
G ab is the geometric mean of two positive numbers a and b.

(e) If terms are given in G.P. and their product is known, then the terms must be picked up
in the following way.
56

ARTHMETIC PROGESSION
a
For three terms in G.P., we choose them as , a, ar
r
a a
3
For four terms in G.P., we choose them as 3 , , ar, ar
r r
a a
2
For five terms in G.P., we choose them as 2 , , a, ar, ar etc.
r r
(ii) Useful Properties
(a) The product of the terms equidistant from the beginning and end is constant, and it is
equal to the product of the first and the last term.
(b) If every term of a G.P. is multiplied or divided by the some non-zero quantity, the
resulting progression is a G.P.
(c) If a1, a2, a3 ... and b1, b2, b3, ... be two G.P.s of common ratio r1 and r2 respectively, then
a1b1 , a2b2 ... and

a1 a 2 a 3
r
,
,
... will also form a G.P. Common ratio will be r1r2 and 1
b1 b 2 b3
r2

respectively.
(d) If a1, a2, a3, ... be a G.P. of positive terms, then loga1, loga2, loga3, ... will be an A.P. and
conversely.

HARMONIC PROGRESSION (H.P.)


(a) A sequence is said to be in harmonic progression, if and only if the reciprocal of its
terms form an arithmetic progression.
For example
1 1 1
, , ... form an H.P., because 2, 4, 6, ... are in A.P..
2 4 6

(b) If a, b are first two terms of an H.P. then


tn

1
1
1 1
(n 1)
a
b a

(c) Harmonic mean H of any two numbers a and b


H

2
1 1

a b

2ab
, where a, b are two non-zero numbers.
ab

(d) If terms are given in H.P. then the terms could be picked up in the following way
(i) For three terms in H.P, we choose them as
1
1 1
, ,
a d a a d

(ii) For four terms in H.P, we choose them as

57

IIT- MATHS
1
1
1
1
,
,
,
a 3d a d a d a 3d

(iii) For five terms in H.P, we choose them as


1
1 1 1
1
,
, ,
,
a 2d a d a a d a 2d

Insertion of Means Between two numbers


Let a and b be two given numbers.
(i) Arithmetic Means
If a, A1, A2 , ... An, b are in A.P., then A1, A2, ... An are called n A.M.s between a and b. If d is
the common difference, then
b = a + (n + 2 1) d d =
Ai = a + id = a + i

ba
n 1

b a a(n 1 i) ib

, i = 1, 2, 3, ..., n
n 1
n 1

Note: The sum of n-A. M s, i.e., A1 + A2 + ... + An =

n
(a b)
2

(ii) Geometric means


If a, G1, G2 ... Gn, b are in G.P., then G1, G2 ... Gn are called n G.M.s between a and b. If r is the
common ratio, then
1

b = a.r

n+1

(n 1)
r = b
a

b n 1
Gi = ar = a a
a
i

n 1 i
n 1

.b

i
n 1

, i = 1, 2, ..., n

Note: The product of n-G. M s i.e., G1 G2 ... Gn =

ab

(iii)Harmonic Means
If a, H1, H2... Hn, b are in H.P., then H1, H2 ... Hn are called n H.M.s between a and b. If d is the
common difference of the corresponding A.P., then

1 1
a b
(n 2 1) d d
b a
ab(n 1)
1 1
1
ab
ab(n 1)
id i

, i 1, 2, 3, ..., n
Hi a
a ab(n 1)
b(n i 1) ia

58

ARTHMETIC PROGESSION

MEANS OF NUMBERS
Let a1, a2, ... an be n given numbers
(i) Arithmetic Means (A.M.) =

a1 a 2 ... a n
n

(ii) Geometric Means (G.M.) = (a1 a2 ... an)1/n


n
(iii) Harmonic Mean (H.M.) = 1/ a 1/ a ... 1/ a
1
2
n

If weights of a1, a2, ... , an are w1, w2, ..., wn respectively, then their weighted arithmetic mean,
weighted geometric mean and weighted harmonic mean are respectively defined by
1

a1w1 a 2 w 2 ... a n w n
(a1w1 .a 2 w 2 ...a n w n ) w1 w 2 ...w n
w1 w 2 ... w n
and

w1 w 2 ... w n
.
w1 w 2
wn

...
a1 a 2
an

RELATION BETWEEN A, G AND H


If A, G and H are A.M., G.M. and H.M. of positive numbers a1, a2 ... an ( a1 a 2 ... a n ) then
a1 H G A a n ...(1)
Note:
(i) The equality at any place in (1) holds if and only if the numbers a1, a2, ..., an are all equal
(ii) (1) is true for weighted means also
(iii) G2 = AH, if n = 2.

ARITHMETIC MEAN OF mth POWER


Let a 1 , a 2
a1m a m2 ... a mn
n

... , a n be n positive real numbers

and let m be a real number, then

a a ... a n
1 2
, if m R [0,1].
n

a1m a m2 ... a mn a1 a 2 ... a n

However if m (0, 1), then

n
n

a m a m2 ... a mn a1 a 2 ... a n

Obviously if m {0,1}, then 1

n
n

ARITHMETIC-GEOMETRIC SERIES
A series whose each term is formed, by multiplying corresponding terms of an A.P. and a G.P., is
called an Arithmetic-geometric series.
59

IIT- MATHS
e.g.

1 + 2x + 4x2 + 6x3 + ..... ; a + (a + d) r + (a + 2d)r2 + .....


(i) Summation of n terms of an Arithmetic-Geometric Series
Let Sn = a + (a + d) r + (a + 2d)r2 + ... + [a + (n 1)d] rn1, d 0 , r 1
Multiply by r and rewrite the series in the following way
rSn = ar + (a + d)r2 + (a + 2d)r3 + ... + [a + (n 2)d]rn1 + [a + (n 1)d ]rn on subtraction,
Sn (1 r) = a + d(r + r2 + ... + rn1) [a + (n 1)d]rn
or, Sn (1 r) a

or, Sn

dr(1 r n 1 )
[a (n 1)d].r n
1 r

a
dr(1 r n 1 ) [a (n 1)d] n

.r
1 r
(1 r)2
1 r

(ii) Summation of Infinite Series


If |r| < 1, then (n 1)rn, rn1 0, as n .
Thus S = S =

a
dr

1 r (1 r)2

SUM OF MISCLENIOUS SERIES


(i) Defference Method
Let T1, T2, T3 ... Tn be the trms of a sequence and let (T2 T1) = T1 , (T3 T2) = T2 ... ,
(Tn Tn1) = T n 1 .
Case I: If T1, T2 ,....Tn1 are in A.P. then Tn is a quadratic in n. If T1 T2 , T2 T3 , ... are in
A.P., then Tn is a cubic in n.
Case II: If T1, T2 ,....Tn1 are not in A.P., but if T1 , T2 ,..., Tn1 are in G.P., then Tn = arn + b,
where r is the C.R. of the G.P. T1 , T2 , T3 .....and a, b R.
Again if T1,T2,...Tn1 are not in G.P. but T2 T1, T3 T2 ,...Tn1 Tn2 are in G.P., then Tn is of the
form arn + bn + c; r is the C.R. of the G.P. T2 T1, T3 T2 T4 T3 ... and a, b, c R.
(ii) Vn Vn1 Method
Let T1, T2, T3 , ... be the terms of a sequence. If there exists a sequence V1, V2, V3 ... satisfying
Tk = Vk Vk1, k 1,
n

k 1

k 1

then S T (V V ) V V .
k k k 1 n 0
n

60

ARTHMETIC PROGESSION

OBJECTIVE
SECTION - I
LEVEL - I
Multiple Choice Questions with Single Answer:
1.

2.

If a1, a2, a3 ... are in AP, then ap, aq, ar are in AP, if p, q, r are in
(a) AP

(b) GP

(c) HP

(d) none of these

Let tr denote the rth term of an AP. If tm =

(a)

1
mn

(c) 1

3.

1
1
and t n = , then tmn equal to
m
n

(b)

1 1
1
m n

(d) 0

If p, q, r, s N and they are four consecutive terms of an AP, then the pth, qth, rth, sth terms of
a GP are in

4.

5.

6.

61

(a) AP

(b) GP

(c) HP

(d) none of these

Let a1, a2, a3, ... be in AP and ap, aq, ar be in GP. Then aq : ap is equal to
(a)

rp
qp

(b)

qp
rq

(c)

rq
qp

(d) none of these

If a, b, c are in G.P., then a + b, 2b, b + c are in


(a) A.P.

(b) G.P.

(c) H.P.

(d) none of these

If a, b, c, d are nonzero real numbers such that (a2 + b2 + c2) (b2 + c2 + d2) (ab + bc + cd)2, then
a, b, c, d are in
(a) AP

(b) GP

(c) HP

(d) none of these

IIT- MATHS
7.

9.

10.

11.

12.

13.

If 4a2 + 9b2 + 16c2 = 2(3ab + 6bc + 4ca), where a, b, c are nonzero numbers, then a, b, c are in
(a) AP

(b) GP

(c) HP

(d) none of these

If a, b, c are in H.P., then c, c a, c b are in :


(a) A.P.

(b) G.P.

(c) H.P.

(d) none of these

Let S be the sum, P be the product and R be the sum of the reciprocals of n terms of a GP. Then
P2Rn: Sn is equal to
(a) 1 : 1

(b) (common ratio)n : 1

(c) (first term)2 : (common ratio)n

(d) none of these

If a1, a2, a3 are in AP, a2, a3, a4 are in GP and a3, a4, a5 are in HP, then a1, a3, a5 are in
(a) AP

(b) GP

(c) HP

(d) none of these

1
1
1
If x > 1, y > 1, z > 1 are three numbers in GP then 1+ ln x , 1+ ln y , 1+ ln z are in

(a) AP

(b) HP

(c) GP

(d) none of these

If a, a1, a2, a3, ... a2n1, b are in AP, a, b1 , b2 , b3 ..., b2n1, b are in GP and a, c1, c2, c3, ... , c2n1, b are
in HP, where a, b are positive, then the equation anx2 bnx + cn = 0 has its roots
(a) real and unequal

(b) real and equal

(c) imaginary

(d) none of these

The product of n positive numbers is unity. Then their sum is


(a) a positive integer
(c) equal to n +

14.

1
n

(b) divisible by n
(d) never less than n

If x > 0 and a is known positive number, then the least values of ax +


(a) a2

(b) a

(c) 2a

(d) none of these

a
is
x

62

ARTHMETIC PROGESSION
15.

16.

17.

If p, q, r be three positive real numbers, then the value of (p + q) (q + r) (r + p) is


(a) > 8 pqr

(b) < 8 pqr

(c) 8 pqr

(d) none of these

a1
a2
an
If a1, a2, a3 .. an are in H.P., then a 2 a 3 ....an , a 2 a 3 ....an ,.... a 2 a 3 ....an 1 are in

(a) A.P.

(b) G.P.

(c) H.P.

(d) none of these

1 1 1
1
Sn = 1 + 2 3 4 ..... 2 n 1 , then

(a) S100 < 100

(b) S200 < 200

(c) S200 > 100

(d) S50 > 25

18.

Given Sn
r0

19.

1
1
1
,s

, then least value of n is


r
r If S Sn
2
1000
r 0 2

(a) 8

(b) 9

(c) 10

(d) 11

If x15 x13 + x11 x9 + x7 x5 + x3 x = 7,(x > 0), then


(a) x16 is equal to 15

(b) x16 is less than 15

(c) x16 greater than 15

(d) none of these

20.

(n 1) (n 2) (n 3) ......(n k)

is equal to

n 1

1
(a) (k 1) k 1

1
(b) k k

1
(c) (k 1) k

1
(d) k

21.

63

n
1

Maximum value of n for which n is


2
14
1

(a) 4

(b) 5

(c) 6

(d) 7

IIT- MATHS
22.

1
7
1 20
..... is
The nth term of the series 2 1 1
2 13 9 23
20
(5n 3)

(b)

20
(5n 3)

(b) 20 (5n + 3)

(d)

20
(5n 2 3)

(a)

23.

24.

If

(a) A.P.

(b) G.P.

(c) H.P.

(d) None of these

If 2p + 3q + 4r = 15, the maximum value of p3q5 r7 will be

(a) 2180

54.35
(b) 15
2

55.7 7
217.9

(d) 2285

(c)

25.

1
1
1
1
+
+ +
= 0 and a c b then a, b, c are in
a a-b c c-b

The minimum value of x4

1
is
x2

1/ 3

1
(a) 3
4

(b)

1
2

1/ 3

1
(c) 2
3

(d) 2

64

ARTHMETIC PROGESSION

LEVEL - II
Multiple Choice Questions with one or more than one correct Answers:
1.

Total number of positive real values of x such that x, [x], {x} are in H.P, where [.] denotes the
greatest integer function and {.} denotes fraction part, is equal to
(a) zero
(c) 2

2.

(b) 1
(d) none of these

In the sequence 1, 2, 2, 4, 4, 4, 4, 8, 8, 8, 8, 8, 8, 8, 8, ..... , where n consecutive terms have the


value n, the 1025th term is
(a) 210
(c) 29

3.

(b) 211
(d) 28

Sr denotes the sum of the first r terms of an AP. Then S3n : (S2n Sn) is
(a) n
(c) 3

4.

(b) 3n
(d) independent of n

a, b, c are distinct real numbers such that a, b, c are in A.P. and a2, b2, c2 are in H.P. then
(a) 2b2 = ac
(c) 2b2 = ac

(b) 4b2 = ac
(d) 4b2 = ac
n-1

5.

If x1, x2, ... xn are in H.P. then

x r+1 is equal to

r=1

(a) (n 1)x1xn
(c) (n + 1) x1 xn
6.

(b) nx1 xn
(d) none of these

If ax = by = cz and x, y, z are in GP then logc b is equal to


(a) logba
(c) z/y

(b) logab
(d) none of these
n

7.

The value of

r=1

8.

65

(a)

n
a a nx

(c)

n( a nx a)
x

1
a + rx + a + (r -1)x

is

(b)

a nx a
x

(d) none of these

5c
Sides a, b, c of a triangle are in G.P. If ln , ln
a
must be

3b
a
and ln are in A.P., then triangle
5c
3b

IIT- MATHS
(a) Isosceles
(c) Obtuse angled
9.

(b) Equilateral
(d) None of these

G13 + G 32
If A1 be the A.M. and G1, G2 be two G.Ms between two positive numbers a and b, then
G1G 2 A1
is equal to
(a) 1
(c) 3

10.

(b) 2
(d) none of these

{bi}, i = 1, 2, ..., n is an arithmetic sequence. If b1 + b5 + b10 + b15 + b20 + b24 = 255, then
24

is equal to

i=1

(a) 600
(c) 300
11.

(b) 900
(d) none of these

If 2. nP1, nP2, nP3 are three consecutive terms of an AP then they are
(a) in GP
(c) equal

12.

(b) in HP
(d) none of these

In a GP the product of the first four terms is 4 and the second term is the reciprocal of the fourth
term. The sum of the GP up to infinite terms is
(a) 8
(c) 8/3

13.

(b) -8
(d) 8/3

If a1, a2, ... an are positive real numbers whose product is a fixed number c, then the minimum
value of a1 + a2 + ... an1 + 2an is
(a) n(2c)1/n
(c) 2nc1/n

14.

If a, b, c, d are four positive numbers then


a b c d
(a) 4.
b c d e
(c)

15.

(b) (n + 1)c1/n
(d) (n + 1) (2c)1/n

a b c d e
5
b c d e a

a
e

a c b d
(b) 4.
b dc e
(d)

a
e

b c d e a 1

a b c d e 5

If a2 + b2 + c2 = 1 then ab + bc + ca lies in the interval.


1
(a) , 2
2

(b) [1, 2]

1
(c) ,1
2

1
(d) 1,
2

66

ARTHMETIC PROGESSION
16.

Let f(x) =

2 3
n +1
1- x n +1
and g(x) = 1 + 2 ... + (1)n
. Then the constant term in
x x
xn
1- x

f (x) g(x) is equal to

17.

18.

n(n 2 1)
(a)
, when n is even
6

(b)

n
(c) (n 1) , when n is even
2

(d)

(a)

1
an
,
1 a 1 a

(b)

1
an
,
1 a 1 a

(c)

1
an
,
1 a 1 a

(d)

1
an
,
1 a 1 a

Let S1, S2, S3, ... be squares such that for n 1, the length of a side of Sn equals the length of a
diagonal of Sn+1. If the length of a side of S1 is 10 cm then for which of the following values of
n is the area of Sn less than 1 cm2?
(b) 8
(d) 10

If a, b, c, d are distinct integer in A.P. such that d = a4 + b4 + c4, then a + b + c + d is


(a) 0
(c) 2

20.

(b) 1
(d) none of these

Three positive numbers form a GP. If the middle number is increased by 8, the three numbers
form an AP. If the last number is also increased by 64 along with the previous increase in the
middle number, the resulting numbers form a GP again. Then
(a) common ratio = 3
(c) common ratio = 5

21.

67

n(n 1)
, when n is odd
2

a n 1
A
B
, then S =

, where A & B are respectively


If tn =
n 1
n
n
(1 a x) (1 a x)
1 x 1 an

(a) 7
(c) 9

19.

n(n 1)
, when n is odd
2

(b) first number = 4/9


(d) first number = 4

If a, b, c are in GP and a, p, q are in AP such that 2a, b + p, c + q are in GP then the common
difference of the AP is
(a)

2a

(b) ( 2 + 1) (a b)

(c)

2 (a + b)

(d) ( 2 - 1) (b - a)

IIT- MATHS

22.

23.

1
1
1
Value of 1 + 1 2 1 2 3 .... 1 2 3 ....n is equal to

(a)

2n
n 1

(b)

(c)

4n
3n 1

(d) none of these

If x, y, z are positive numbers in AP then


(a) y2 xz
(c)

1
r 1

n
r 1

(a) 2n2
(c) n2

25.

(b) y 2 xz

xy
yz
xy
yz

4
has the minimum value 2(d)
2y x 2y z
2y x 2y z

24.

3n
2n 1

r 1

is equal to
(b) 3n2
(d) none of these

Between two unequal numbers, if a1, a2 are two AMs; g1, g2 are two GMs and h1, h2 are two
HMs then g1.g2 is equal to
(a) a1h1
(c) a2h2

(b) a1h2
(d) a2h1

68

ARTHMETIC PROGESSION

SECTION - II
COMPREHENSIVE PASSAGE
I

An A.P. is a sequence whose terms increase or decrease by a fixed number, called the common
difference of the A.P.. If a is the first term and d the common difference, the A.P. can be written
as a, a+d, a + 2d, The nth term an is given by an = a + (n 1)d. The sum sn of the first n terms
of such an A.P. is given by : sn =

n
n
(2a + (n 1)d) = (a + l) where l is the last term (i.e., the nth
2
2

a c
is the A.M. of a and c. The n numbers A1,
2
A2,.., An are said to be A.M.s between the numbers a & b if
term of the A.P. ). If a, b, c are in A.P., then b

a, A1, A2, ..,An, b are in A.P. If d is the common difference of this A.P., then d
Ar a r

1.

2.

3.

(b a)
th
n 1 , where Ar is the r mean

9
If 6 A.M.s are inserted between 1 and , then the 4th arithmetic mean is equal to
2
(a)

3
2

(b)3

(c)

2
3

(d)

The pth term of an A.P. is a and qth term is b, then sum of its (p + q) terns is
(a)

pq
2

pq
(b) a b

pq

(c)

pq
a b
ab

2
p q

(d)none of these

If log 2, log (2x 1) and log (2x + 3) are in A.P., then the value of x is
(a)5/2
(c)log35

4.

(b)log25
(d)log53

2
2
If am be the mth term of an A.P., then a12 a 22 a 32 a 42 ...... a 2n
1 a 2n is equal to

(a)

69

13
5

2n 1 2
a1 a 2n2
n

(b)

n
a12 a 2n2
2n 1

ba
n 1

IIT- MATHS
(c)
II

n
a12 a 22n

2n 1

(d)

2n 1 2
2
a1 a 2n

If x1, x2,.. . . . . xn are n positive real numbers; then A.M. G.M. H.M.

x1 x 2 .............xn
n
(x1 x 2....xn ) 1/n
1 1
1
n
......... equality occurs when numbers are same using
x1 x 2
xn
this concept.

5.

If a > 0, b > 0, c > 0 and the minimum value of a(b2 + c2) + b(c2 + a2)+ c(a2 + b2) is labc, then l is
(a) 1
(c) 3

6.

(b) 2
(d) 6

If a, b, c, d, e, f are positive real numbers such that a + b + c + d + e + f = 3, then


x = (a + f)(b + e)(c + d) satisfies the relation
(a) 0 < x 1

(b) 1 x 2
(d) 3 x 4

(c) 2 x 3
7.

8.

If a and b are two positive real numbers, and a + b = 1, then the greatest value of a3b4 is
(a)

32 43
75

(b)

(c)

77
33 44

(d) none of these

If x2 + y2 = 9, then the value of x + y lies in the interval


(a) [0,3]
(c) [-3 2 , 3 2 ]

III.

33 44
77

(b) [-3,3]
(d) [0,3 2 ]

If nth term Tn of a given sequence is of the form Vn Vn 1, where V1, V2, V3, . . . is some other
n

sequence, then Sn T k Vn V 0 . Similarly if the nth term Tn of a given sequence is of the form
K 1

Vn
, then Pn =
Vn 1

9.

Vn

T V
n

k 1

n
4
Lt tan -1 2
=
n
4n + 3
n =1

(a) 1
(c) 3

(b) 2
(d) 4

70

ARTHMETIC PROGESSION
10.

If nth term of a sequence is


(a) more than
(c) equal to

11.

12.

IV.

1
n 1 n , then sum of first n terms of the sequence is
(b) less than

n 1

n 1

(d) can not be determined

n 1

If cos 1 + cos 2 + cos 3 + . . . + cos n = Sn, then lim


n

Sn
n

(a) equals 1

(b) equals sin 1

(c) does not exist

(d) none of these

1 1 1
Sum of the infinitely many terms of the series 2 3 4 ..... is equal to

(a)

2
3

(b) 1

(c)

3
2

(d) does not exist as the sum tends to infinity

The sum of n terms of a series each term of which is composed of the reciprocal of the product of
r factors in arithmetical progression, can be done by following rule:
Write down the nth term, strike off a factor from the beginning, divide by the number of factors
so diminished and by the common difference, change the sign and add a constant.
The sum of n term of a series each term of which is composed of r factors in arithmetical progression. Write down the nth term, affix the next factor at the end, divide by the number of factors
thus increased and by the common difference and add a constant.

13.

14.

1
1
1
The sum of n terms of the series 1.2.3.4 2.3.4.5 3.4.5.6 .... is

1
1
(a) 18 3(n 1) (n 2) (n 3)

1
1
(b) 18 3(n 1) (n 2) (n 3)

1
1
(c) 18 3(n 1) (n 2) (n 3)

1
1
(d) 18 3(n 1) (n 2) (n 3)

The sum of n terms of the series

1
1

..... is
1.3.5.7.9 3.5.7.9.11

1
1
(a) 840 8(2n 1) (2n 3) (2n 5)

71

1
1
(b) 840 8(2n 1) (2n 3) (2n 5) (2n 7)

IIT- MATHS

1
1
1
1
(c) 840 8(2n 1) (2n 3) (2n 5) (2n 7) (d) 840 8(2n 1) (2n 3) (2n 5) (2n 7)

15.

If

1
b
c
3
4
5

... = a
n 3 (n 2) (n 3) (n 1) (n 2) (n 3) , then
1.2.4 2.3.5 3.4.6

(a) a

29
3
4
,b ,c
36
2
3

(b) a

29
3
4
,b ,c
36
2
3

(c) a

29
3
4
,b ,c
36
2
3

(d) a

29
3
4
,b ,c
36
2
3

72

ARTHMETIC PROGESSION

SUBJECTIVE
SECTION - III
LEVEL - I
1.

The interior angles of a polygon are in arithmetic progression. The smallest angle is 120 and the
common difference is 5. Find the number of sides of the polygon.

2.

The ratio between the sum of n terms of two A.P.s is 7n + 1 : 4n + 27. Find the ratio between their
nth terms.

3.

The r th , s th and t th terms of a certain G..P. are R, S and T respectively. Prove that
Rst . Str. Trs = 1.

4.

(i)
(ii)

If one G.M. G and two arithmetic means p and q be inserted between any given numbers,
then show that G2 = (2p q) (2q p).
If H be the H.M. between a & b, then show that (H 2a) (H 2b) = H2

5.

If pth, qth, rth terms of an A.P. be a, b, c respectively, then prove that


p(b c) + q (c a) + r (a b) = 0.

6.

(i)
(ii)

The sum of three numbers in G.P. is 42. If the first two numbers are increased by 2 and
third is decreased by 4, the resulting numbers form an A.P. Find the numbers of G.P.
The sum of an infinite geometric series is 162 and the sum of its first n terms is 160. If the
reciprocal of its common ratio is an integer, find all possible values of the common ratio,
n and the first term of the series.

7.

Evaluate: 1 + 2.2 + 3.22 + 4.23 + ... + 100.2 99 .

8.

Find the sum of n terms of the series, the rth term of which is (2r + 1) 2r.

9.

Let x = 1 + 3a + 6a2 + 10a3 + ..., |a| < 1; y = 1 + 4b + 10b2 + 20b3 + ..., |b| < 1. Find S = 1 + 3 (ab)
+ 5 (ab)2 ... in terms of x and y.

10.

If a, b, c are the sides of a triangle , then prove that a2 + b2 + c2 > ab + bc + ca.

73

IIT- MATHS

LEVEL - II
BRUSH UP YOUR CONCEPT
1.

Let Sn denote the sum of first n terms of an A.P. If S2n = 3Sn , then show that the ratio S3n/Sn is equal
to 6.

2.

Show that the number 1111.....1

is a composite number..
91 digits

3.

(i)

If a1, a2, a3, ... are in A.P. with common difference d, then prove that
tan 1

4.

d
d
d
nd
tan 1
... tan 1
tan 1
1 a 1a 2
1 a 2a 3
1 a n a n 1
1 a 1a n 1

(ii)

Sum to n terms the series


12 22 + 32 42 + 52 62 + ...

(i)

If the A.M. of a and b is t wice as great as their G.M., then show t hat

(ii)

a : b = (2 3):(2 3) .
If the (m + 1)th, (n + 1)th and (r + 1)th terms of an A.P. are in G.P., m, n, r are in H.P. show
2
that the ratio of the common difference to the first term in the A.P. is .
n

13 13 23 13 23 33

...16 terms.
1 1 3
1 3 5

5.

Evaluate:

6.

Find the sum Sn of the cubes of the first n terms of an A.P. and show that the sum of first n terms
of the A.P. is a factor of Sn.

7.

If the roots of 10x3 cx2 54x 27 = 0 are in harmonic progression, then find c and all the roots.

8.

An A.P. and a G.P. with positive terms have the same number of terms and their first terms as well
as last terms are equal. Show that the sum of the A.P. is greater than or equal to the sum of the G.P.

9.

1 1 1
If x, y, z are postive and x + y + z = 1, prove that 1 1 1 8
x y z

10.

In a triangle ABC prove that

3
a
b
c

2
2 bc ca ab

74

ARTHMETIC PROGESSION

LEVEL - III
1.

The sum of the squares of three distinct real numbers, which are in G.P. is S2. If their sum is S,

1
2
show that ,1 (1, 3) .
3

2.

[1986]

If p be the first of n arithmetic means between two numbers and q be the first of n harmonic means
2

n 1
between the same two numbers, prove that the value of q cannot be between p and
p.
n 1

[1991]
3.

If S1, S2, S3, ... Sn are the sums of infinite geometric series whose first terms are 1, 2,3, ..., n and
whose common ratios are

1 1 1
1
, , ,...,
respectively, then find the value of
2 3 4
n 1

2
S12 S 22 S32 ... S 2n
1 .

[1991]

4.

The real numbers x1, x2, x3 satisfying the equation x3 - x2 + x + = 0 are in A.P. Find the
intervals in which and lie.
[1996]

5.

Let a1, a2, ... be positive real numbers in geometric progression. For each n, let An, Gn, Hn be
respectively, the arithmetic mean, geometric mean and harmonic mean of a1, a2, ... an. Find an
expression for the geometric mean of G1, G2, ..., Gn in terms of A1, A2, ... , An, H1, H2, ... Hn.
[2001]

6.

The fourth power of the common difference of an arithmetic progression with integer entries is
added to the product of any four consecutive terms of it. Prove that the resulting sum is the square
of an integer.
[2002]

7.

Let a, b, be positive real numbers. If a, A1, A2, b are in arithmetic progression, a, G1 , G2, b are in
geometric progression and a, H1, H2 , b are in harmonic progression, show that
G1G 2 A1 A 2 (2a b) (a 2b)

.
H1H 2 H1 H 2
9ab

[2002]

8.

If a, b, c are in A.P. and a2, b2, c2 are in H.P., then prove that either a/2, b, c are in G.P.
or a = b = c.
[2003]

9.

Prove that (a + 1)7 (b + 1)7 (c + 1)7 > 77 a4 b4 c4, where a, b, c, R

[2004]

10.

An infinite G.P has first term x and sum 5, then find the exhaustive range of x ?

[2004]

11.

For n = 1, 2, 3, , let A n

3 3 3
3
..... (1) n 1 , and Bn = 1 An .
4 4 4
4

Find the smallest natural number n0 such that Bn > An for all n n0.
75

[2006]

IIT- MATHS

ANSWER
OBJECTIVE
SECTION - I
LEVEL - I
1.

2.

3.

4.

5.

6.

7.

8.

9.

10.

11.

16.

17. 18. 19.

20.

12. 13.

14. 15.

21. 22. 23. 24. 25.

LEVEL - II
1.

2.

3.

4.

5.

6.

c,d

a,c a,b

16.

17. 18. 19.

b,c

c b,c,d c,

20.

7.

8.

9.

10.

11.

12. 13.

14. 15.

a,b
c

a,b
c,d

a,b
c,d

12. 13.

21. 22. 23. 24. 25.

a,d b,d

a,d

b,d

SECTION - II
1.

2.

3.

4.

5.

6.

7.

8.

9.

10.

11.

14. 15.

76

ARTHMETIC PROGESSION

SUBJECTIVE
SECTION - III
LEVEL - I
1.
2.
6.

9
14n 6
8n 23
(i)
6, 12, 24 OR 24, 12, 6
(ii)
n = 4, 2 or 1 and a = 108, 144 or 160

7.
8.

99.2100 + 1
n2n+2 2n+1 + 2

9.

1 ab
where a = 1 x1/3 & b = 1 y 1/4
(1 ab)2

LEVEL - II

n(n 1)
n(n 1)
, when n be even &
, when n be odd
2
2

3.

(ii)

5.

446

6.

1 n
Sn . 2a (n 1)d [2a 2 2ad (n 1) nd 2 (n 1)]
2 2

7.

3 3
c = 9, roots are 3, ,
2 5

LEVEL - III
LEVEL-III
3.

1
n (2n 1) (4n 1) 1
3

5.

1
1
and
3
27
(A1 A2 .... An . H1 H2 ..... Hn)1/2n

10.

0 < x < 10.

11.

least value of n0 = 2

4.

77

IIT- MATHS

SUBJECTIVE SOLUTIONS
LEVEL - I (CBSE LEVEL)
REVIEW YOUR CONCEPTS
1.

Find the sum of first 24 terms of the A.P. a 1 , a 2 , a 3 , ... if it is known that
a1 + a5 + a10 + a15 + a20 + a24 = 225.
Solution :
Let s = a1 a2 ... a24 are in A.P.
Given a1 + a5 + a10 + a15 + a20 + a24 = 225
First term = a1 commoun ratio = d
a1 + a1 + 4d + a1 + ad + a1 + 14d + a1 + 19d + a1 + 23d = 225
6a1 + 69d = 225
2a1 + 23d = 75 = t24
Sn = n/2 (a + l) 524 =

2.

24
(75) = 900
2

The interior angles of a polygon are in arithmetic progression. The smallest angle is 120
and the common difference is 5. Find the number of sides of the polygon.
Solution :
First term a = 120 common ratio = 5
Given : sum of enterior angles of a polygon n is

(2n - 4)
= (n 2) (n 2) 180
2
n
Sn = [2a (4 1) d] = (n 2) 180
2
n
[240 5n 5] = (n - 2)180

2
n2 - 25n + 144 = 0 n = 16 n = a

3.

The ratio between the sum of n terms of two A.P.s is 7n + 1 : 4n + 27. Find the ratio between
their nth terms.
Solution :
Let a1 a2 ... a4 are om AP
b1b2 ... bn are in A.P.
n
n
Sn = {2a1 + (n 1) d1} Sn =
{2b1 + (n 1)d2}
2
2

78

ARTHMETIC PROGESSION
7n 1 2a1 (n 1)d
ratio is 4n 27 2b (n 1)d
1

2a1 (n 1)d
2 4 7(n 1)

31
2 4(n 1) 2b1 (n 1)d
2
2a1 (n 1)d
8 1 (7 7n) = 2b (n 1)d
1

2 4 7(n 1) 2a1 (n 1)d

31
2 4(n 1) 2b1 (n 1)d
2

comparing the values a1, b1 and d.


a1 = 4 b1 =
ratio of nth term is

4.

31
d = 7 d2 = 4
2 1

14n 6
a1 (n 1)d1
4 (n 1)7

=
31
23 8n
b1 (n 1)d 2
(n 1) 4
2

The rth, sth and tth terms of a certain G..P. are R, S and T respectively. Prove that
Rst . Str. Trs = 1.
Solution :
Given Tr = arr 1 = R
TS = ars1 = S
Tt = ar t 1 = T
St

then R

t
t St t
RS
a s .r rs s St r S a .r
= t tr t ;
Sr a r .r rs t
Rt
a .r

Trs =

5.

T r a r .r tr S

Ts a s .r ts s

RSt . St r . Tr s = r = 1

The sum of three numbers in G.P. is 42. If the first two numbers are increased by 2 and
third is decreased by 4, the resulting numbers form an A.P. Find the numbers of G.P.
Solution :
Let a, ar, ar2 are in G.P.

79

IIT- MATHS
2

Given a + ar + av = 42
a(1 + r + v2) = 42

... (1)

and a + 2, ar + 2, ar2 4 are in A.P.

2(ar + 2) = a + 2 + ar2 4

a (1 + r2 2r) = 6
6
a=
2
1 r 2r
Put a in 0
6
(1 + r + r2) = 42
2
(1 r 2r)
6r2 - 15r 6 = 0
1
r = 2,
2
a = 6, 24

If a = 6, r = 2
6, 12, 24
If

a = 24, r = 1/2
24, 12, 6

6.

If one G.M. G and two arithmetic means p and q be inserted between any given numbers,
then show that G2 = (2p q) (2q p).
Solution :
x, p, q, y in these A.M. of x, p, q
2p = x + q
x = 2p - q
and

p, q, y 2q = p + y
y = 2q p
G2 = 2y (G - Geometric mean)
G2 = (2p q) (2q p)

7.

The sum of an infinite geometric series is 162 and the sum of its first n terms is 160. If the
reciprocal of its common ratio is an integer, find all possible values of the common ratio, n
and the first term of the series.
Solution :
Sum of an infinite geometric series
80

ARTHMETIC PROGESSION
a
Sn =
= 162 a = 162 (1 r)
1 r
sum of first in terms is Sn =

a(1 r n )
160
1 r

162 (1 r) (1 r n )
160
(1 r)
1

160 n
r r is an integer..
162

1
rn
3

if n = 4, r = 1/3, then a = 108


1
if n = 1; r =
then a = 160
81
1
n = 2, r then a = 144.
a
a = 108, 160, 144

8.

Evaluate: 1 + 2.2 + 3.22 + 4.23 + ... + 100.2 99 .


Solution :
Let s = 1 + 2.2 + 3 . 22 + ..... + 100.299
2.5 = 2 + 2.22 + 3.23 + ..... 99.299 + 100.2100
s - 25 = (1 + 2 + . . . . . + 299) + 100.2100
8 = 1 + 2 (299 - 1) - 100.2100
s = 99.2100 1 = 99.2100 + 1

9.

If p th , q th , r th terms of an A.P. be a, b, c respectively, then prove that


p(b c) + q (c a) + r (a b) = 0.
Solution :
Given Tp = a1 + (p - 1) d1 = a
Tq = a1 + (q - 1) d1 = b
Tr = a1 + (r - 1) d1 = c

ab
a - b = (p - q) d1 =
= d1
pq

81

IIT- MATHS
Similarly

bc
ca
d1 ;
= d1
qr
rp

p(b - c) + q (c - a) + r (a - b) pd, (q - r) + qd1 (r.p) + rd1 (p q)


0.
10.

Find S of the G.P. whose first term is 28 and the fourth term is

4
.
49

Solution :
First term a = 28
tH =

4
ar 3 r = 1/7
49

S =

11.

a
1 r

If H be the H.M. between a & b, then show that (H 2a) (H 2b) = H2


Solution :
a, H, b are in H.P.
H

2ab
ab

Ha + Hb = 2ab Ha 2ab = Hb
a(H 2b)
b(H 2a)
H; and
H
b
a

(H).(H) =

a
b
(H 2b) (H.2a)
b
a

H2 = (H 2a) (H 2b)

12.

Find the sum of n terms of the series, the rth term of which is (2r + 1) 2r.
Solution :

13.

Let x = 1 + 3a + 6a2 + 10a3 + ..., |a| < 1; y = 1 + 4b + 10b2 + 20b3 + ..., |b| < 1. Find S = 1 + 3 (ab)
+ 5 (ab)2 ... in terms of x and y.
Solution :
Series can be written as
s = 1 + 3z + 5z2 + ....
82

ARTHMETIC PROGESSION
sz = z + 3z2 + ...

s(1 z) = 1 + 2 z + 2z2 + ... 1 + 2z [1 + z + z2 + ... ]

1 z
(1 ab)
1 1 z

s (1 z) = 1 + 2
s=
=
2

(1 z) (1 ab)2
1 z 1 z
where a = 1 x1/3 and b = 1 y1/4
x = 1 + 3a + 6a2 + ..... |a| < 1 b(1 - b) y =

b 2b 2 ....
1 b b2 ...

ax = a + 3a2 + .....
x(1 - a) = 1 + 2a + 3a2 + ...
ax(1 a) = a + 2a2 + .... (1 - b)3 y =

(1 a)2a = 1 + a + a2 + .... =

1
y = (1 - b)-1/4 1 y1/4 = b
1 b

1
1
x = (1 a)3 = (1- a)3
1 a

a = 1 x1/3 .
y = 1 + 4b + 10b2 + 20b3 + ... |b| < 1
by = b + 4b2 + 10b3 + ...
y (1 - b) = 1 + 3b + 6b2 + 10b3 + ...
b(1- b)y = b + 3b2 + .... y(1 - b)2 = 1 + 2b + 3b2 + ...

14.

If a, b, c are the sides of a triangle , then prove that a2 + b2 + c2 > ab + bc + ca.


Solution :
a, b, c are sides of a triangle
(a 2 b 2 c 2 ) a b c

3
3

a 2 b 2 c 2 a 2 b 2 c 2 2(ab bc ca)

3
9
9
3a 2 3b 2 3c 2 a 2 b 2 c2 2(ab bc ca)

9
9
a2 + b2 + c2 > ab + bc + ca

83

IIT- MATHS
15.

1 1
1 2
If xi > 0, (i = 1, 2, ... n), then prove that (x1 + x2 + ... + xn) x + x + ... + x n .
1
2
n
Solution :
xi > 0 (Given)
x1 + x2 + ..... + xn > 0
x1 x 2 .... x n
(x1. x2. x3 ... xn)n
n

(x1 x 2 .... x n )
n

n
1 1
1
...
xn
x1 x 2
1 1
1
(x1 + x2 + ... + xn) x x ... x
2
n
1

2
n

84

ARTHMETIC PROGESSION

LEVEL - I
BRUSH UP YOUR CONCEPTS
1.

Let Sn denote the sum of first n terms of an A.P. If S2n = 3Sn , then show that the ratio S3n/Sn
is equal to 6.
Solution :
Given S2n = 3Sn
2n
n
{2a (2n 1)d} 3 {2a (n 1)d}
2
2
2a = (n + 1)d
n
Sn {2a (n 1)d} {(n 1)d (n 1)d} n 2 d
2
3n
3n
S3n =
{2a + (3n - 1)d}
{(n + 1)d + 3nd d} = 6n2d
2
2
S3n = 6Sn
S3n = 6
Sn

2.

If the roots of the equation x3 12x2 + 39x 28 = 0 are in A.P., then find the common
difference.
Solution :
Let f(x) = x3 12x2 + 39x 28 = 0
if x = 1 f(x) = 0

(x 1) is a factor

x2 11x + 28 = 0
(x 7) (x 4) = 0
x = 7, 4

series 1, 4, 7 are in A.P. (or) 7, 4, 1, ...

common ratio is d = 3

3.

If a1, a2, a3, ... are in A.P. with common difference d, then prove that
tan 1

d
d
d
nd
tan 1
... tan 1
tan 1
1 a 1a 2
1 a 2a 3
1 a n a n 1
1 a 1a n 1

Solution :

85

IIT- MATHS
a1 a2 ... are in AP also
tan1a1 tan1a2 .... are in AP
tan-1 a2 tan1 a1
tan-1 a3 tan1 a2
tan-1 an + 1 tan1 an

1 (a n 1 a 1 )
tan1 an+1 tan1 a1 tan (1 a a ) = tan1
1 n 1
d = n (an + 1 an)

nd

1 a1 a n 1

n=1
d = a2 a1
nd = an+1 an
nd = an + 1 a1

4.

If x = 1 + a + a2 + a3 + ... to (|a| < 1) and y = 1 + b + b2 + b3 + ... to (|b| < 1), then prove
that 1 + ab + a2b2 + a3b3 + ... to =

xy
.
x + y -1

Solution :
x = 1 + a + a2 + ....
x=

1
x 1
a

1 a
x

y = 1 + b + b2 + ...

1
y 1
b
1 b
y

S = 1 + ab + (ab)2 + .....
S =

5.

xy
1
=
x y 1
1 ab

Show that the number 1111.....1

is a composite number..
91 digits

Solution :
We have 111 ..... 1 (91 digits) = 1090 + 1089 + ..... + 102 + 10 + 1

1091 1 1091 1 107 1

10 1 107 1 1 0 1

= (1084 + 1077 + 1070 + ... + 1) (106 + 105 + ..... + 10 + 1)

86

ARTHMETIC PROGESSION
[ 91 = 13 7, (91 divide and multiply by (101 1) or (1013 1]
Thus, 111 ... 1 (91 digits) is a compossite number.

13 13 + 23 13 + 23 + 33
+
+...16 terms.
6.
Evaluate: +
1 1+ 3
1+ 3 + 5
Solution :
Tn

13 23 33 ... n 3 n 2 (n 1)2 1 2

(n 2n 1)
1 3 5 ...n terms
4.n 2
4

16
16
1 16

1 1
1
S16 Tn n 2 1 (16.17.33) 2. .16.17 16 446
4 1
4 6
2

1
1

7.

Sum to n terms the series 12 22 + 32 42 + 52 62 + ...


Solution :
12 22 + 32 42 + ... we consider two cases
(i) Let n be even (1 2) (1 + 2) + (3 4) (3 + 4) + ... [(n 1) n] [(n 1) + n]
(1 2) (1 + 2) + (3 4) (3 + 4) + ..... + [(n 1) n] [(n 1) + n]

= 1 (1 + 2 + ... + n) =

n(n 1)
2

Case II
n is odd (12 22) + (32 42) + .... + [(n 2)2 (n 1)2 ] + n2
= 1 [1 + 2 + .... + (n 1)] + n2 =

8.

(n 1)n
n(n 1)
n2
2
2

If the A.M. of a and b is twice as great as their G.M., then show that a : b = (2 + 3 ):(2 - 3 ) .
Solution :
Am =

ab
GM =
2

ab and A = 2G

ab
2 ab a b 4 ab
2

a
a
1 4
0 put
b
b

a
=x
b

x2 + 1 4x = 0 x = 2 3

87

IIT- MATHS
taking positive sign

a
2 3
(2 3)2
b
2 3

taking negative sign a : b = 2


9.

3/2 3

a, b, c are the first three terms of a geometric series, If the harmonic mean of a and b is 12
and that of b and c is 36, find the first five terms of the series.
Solution :
a, b, c are in G. .P.,
Let choose them to br, b, b/r

12 =

Diving , r =

10.

2.br.b 2 br

b(1 r) 1 r

36 =

2b.b / r 2 b

1 1 r
b(1 )
r

1
and hence from any b = 24
3

five numbers are 8, 24, 72, 216, 648

In a triangle ABC prove that

3
a
b
c

+
+
<2.
2 b +c c+a a+b

Solution :
Ist part adding 3 to both sides
a bc a bc a b c 3

3
bc
ca
ab
2
9
(b + c)1 + (c + a) 1 + (a + b)1 2(a b c)

Applying A.N of nth powers (A.M)m


(b c) (c a) (a b)
then LHS 3

2
3 (a b c)3
3

3.

2
(a b c)3
3

2
9

3(a b c) 2(a b c)

2nd part b + c > a in a triangle


add (b + c) to both sides
2 (b + c) > (a + b + c)

1
1

2(b c) a b c
88

ARTHMETIC PROGESSION
a
2a

write similar inequalities and add


ac abc

a bc

a c 2 a b c 2

89

IIT- MATHS

LEVEL - II
CHECK UP YOUR SKILLS
1.

The sum of the squares of three distinct real numbers, which are in G.P. is S2. If their sum is
1
2 ,1 (1, 3) .
S, show that
3

Solution :
Let the numbers are ar, a1 a/r
1
a(r 1 ) S
r
1
2
2
2
and a (r 1 2 ) S
r
1
1 2
2
put r t, r 2 t 2
r
r
a (t + 1) = S and a2 (t2 1) = S2

Eliminating S, a2 (t2 1) =

(t 1) 2 = (t + 1)

2 1
d2 1

Now t = r

1
r

a 2 (t 1) 2
2

... (1)

r2 r t + 1 = 0

for t to be real t2 4 > 0

t < 2 or t > 2

from (1)

2 1
2 1

2
or
>2
2 1
2 1

In an enequality we can multiply only by a +ve quantity.

2 1 is we whether it is +ve or ve

3 2 1
3 2
2 3

0
2 1
2 1
2 1

1
3( 2 ) ( 2 1)
3
0
( 2 1)2

... (2)

and

( 2 1) ( 2 3)
<0
( 2 1) 2

... (3)

(3) implies that 2 lies between 1, and 3


90

ARTHMETIC PROGESSION

2.

(2) implies that 2 lies between 1/3 and 1.


1
2 ( ,1) (1,3)

3
Find the sum Sn of the cubes of the first n terms of an A.P. and show that the sum of first n
terms of the A.P. is a factor of Sn.
Solution :
Let A.P. be (a + d) + (a + d) + ... + (a + nd)
n
n
[2a (n 1)d]
Sn =
[2a + (n nd]
2
2
Sn = (a + d)3 + (a + 2d)3 + ..... + (a + nd)3
= na3 + 3a2 d n + 3ad2 n2 + d2 n3
= na3 + 3a2 d

n(4 1)
n(n 1) (2n 1) 3 n 2 (n 1) 2
3ad 2
d
2
6
4

1 n
. [2a + (n + 1)d] [2a2 + 2ad + (n + 1 + d2 n (n + 1)]
2 2

3.

If the roots of 10x3 cx2 54x 27 = 0 are in harmonic progression, then find c and all the
roots.
Solution :
, , be the roots in H.P..

then p, q, r be the roots of the equation


10 c 54
27 0
x3 x2 x

27x3 + 54x2 + (x 10 = 0 are in AP ... (1)


2q = p + r

or 3q = p + q = r =

54
2
27

q = 2/3 is roots of (1)

27x2 + 54x2 + 9x 10 = 0 has a root

(3x + 2) (9x2 + 12x 5) = 0

x = 1/3, 2/3, 5/3 are in A.P

2
or
3

3x + 2 is its factor

3, 3/2, 3/5 are in H.P.

4.

91

If the (m + 1)th, (n + 1)th and (r + 1)th terms of an A.P. are in G.P., m, n, r are in H.P. show that

IIT- MATHS
2
the ratio of the common difference to the first term in the A.P. is - .
n

Solution :
a + md, a + nd, a + rd are in G.P

(a + nd)2 = (a + md) (a + rd)


a (2n m r) = d (mr n2)

d 2n (m r)

a
mr r 2

... (1)

but m, n, r are in H.P.


2mr
mr

d 2n (m r)

a (m r) n n 2
2

... (2)

2 2n (m r)
2
by () & (2) (m r) 2n
n
n

1 1 1
If x, y, z are positive and x + y + z = 1, prove that - 1 - 1 - 1 8
x y z
Solution :

5.

A.M G.NM

yz

yz

... (i)

zn

zx

... (ii)

xy

xy

... (iii)

(i), (ii) & (iii)

(x y) (z x) (y z)
xyz
8

(1 x) (1 y) (1 z) 8 xyz

6.

Prove that
x2 + y 2 + z 2

x+y+z

x+ y + z

x+y +z
> xx y y zz >

x+ y + z

Solution :
Consider x numbers each equal to x, y numbers each equal to y and z numbers each equal to z.
then A.M. > G.M. On these x + y + z numbers
92

ARTHMETIC PROGESSION
x x x... x times) (y y ... y times) (z z ... z times)

x yz
[x.x ... x factors) (x. y ... y factors) (z.z ... z factors]1/x + y + z
But x + x ... x times = x.x = x2
and x.x ... . . x factors = xx

x 2 y2 z 2
x 2 y2 z 2
x y z 1/ x y z
(x y z )
or

xyz
xyz

x y z

x x y y z z ... (A)

2nd part : consider x number each equal to 1/x y is 1/y


1
z is
z
AM > GM
1 1
1 1
1 1

...x times ... y times ... z times


y
y
x x

z z
x yz
1

1 1
1 1 ... y factors 1 1
x yz
...z factors
. ...x factors

x x
y y

z z

1
1 1
1
xyz
x . z
3
1

x
y z
1 1 1 xyz
x y z
xyz
x . y . z
x yz

xyz
x y z

x yz

x y z

x x y y z z or

xyz
x y z >

x y z

... (3)

Both (A) & (B) prove the required results.

7.

a 2 + b2
Prove that

a+b
Solution :

a+b

> aa b b

Apply A.M > G.M

(a a .... a times a 2 . a 2
aa.a ...a factors a 2
a 2 b2

ab

8.
93

ab

> aa. bb

The sum of first ten terms of an A.P. is equal to 155, and the sum of the first two terms of a

IIT- MATHS
G.P. is 9, find these progressions, if the first term of A.P. is equal to common ratio of G.P.
and the first term of G.P. is equal to common difference of A.P.
Solution :
Let the AP be a + (a + d) + (a + 2d) + ...
By given condition G.P is d + da + da2 + ...
S1o of A.P = 155

S2 of G.P = 9

2a + ad = 31 d + da = 9
Solving
a 2, d = 3; or a =

A.P is 2 + 5 + 8 + ...

G.P. is 3 + 6 + 12 + 24 + ...
2 25 625
...
G.P. is
3 3
6
9.

25
2
d
2
3

or A.P

25 79 83
....and
2 6 6

The series of natural numbers is divided into groups (1); (2, 3, 4); (5, 6, 7, 8, 9); ... and so on.
Show that the sum of the numbers in the nth group is (n 1)3 + n3.
Solution :

The number of terms in successive groups are 1, 3, 5, ... and hence nth group will be nth terms of
this A.P = 2n n = N.
The last terms of successive groups are 12, 22 ...
hence nth groups is n2 and of (n 1)th group is (n 1)2 .
Hence 1st term of nth group is one more than the last term of (n 1) th group

A = (n 1)2 + 1 = n2 2n 2

Also terms in each group are in A.P. whose D = 1.

10.

n
sum of terms is nth group is sum of an A.P = [2A (n 1)D]
2
2n 1
2
=
[2n 4n + 4 + (2n 2)]
2
= 2n3 3n2 + 3n 1 n3 + (n 1)3 .

An A.P. and a G.P. with positive terms have the same number of terms and their first terms
as well as last terms are equal. Show that the sum of the A.P. is greater than or equal to the
sum of the G.P.
Solution :
Let a be the first terms, b the last term and n the numbers of terms of A.P and G.P. The C.d of

94

ARTHMETIC PROGESSION
1/ n 1

ba
b
A.P =
and C.r of G.P =
n 1
a

Let S be the sum of n terms of A.P and S the sum of n terms G.P..
n
Now S (a b)
2
S = a (1 + r + r2 ... + rn1)

S = a(rn1 +rn2 + ... + 1)


a
S [1 + rn1) + (r + rn1) + ... (rx + rn-k ) + ..... + (rn1 + 1)] ... (2)
2
k
(r + rnk1) (rn1 + 1) = (rk 1) + (rn1 (rx 1)
r n 1
k
n k 1
)0
= (r 1) 1 k (r 1) (1 r

r
k

[for 0 r 1 and also for 1 < r < ]

rk + rnk1

from (2) S

95

n1
+ 1 for k = 0, 1, 2, .... , n 1
r

a b ab
an
(1 r n 1 ) = n 1 =
n S [ from (1)]
2 a 2
2

S S .

IIT- MATHS

PROBLEMS ASKED IN IIT - JEE


1.

If p be the first of n arithmetic means between two numbers and q be the first of n harmonic means between the same two numbers, prove that the value of q cannot be between
2

n +1
p and
p.
n -1

[1991]

Solution :
Let two numbers be a and b
Since n A.M.s have been inserted between a and b
common difference d

ba
h 1

Now P = first A.M. = 2nd terms of A.P = a + d = a +

ba
n 1

Since n + 1 ms have been inserted a and b


C.d of A.P d1 =

ab
ab(n 1)

q = first H.M = 2nd term of H.P

1 1
1
ab
bn a
q a d1 a ab(n 1) ab(n 1)
q =

ab(n 1)
... (ii)
bx a

an b
... (iii)
n 1

x not will between and is ( x ) ( x ) > 0


n(a b)2
Now p q =
... (A)
(bn a) (n 1)
2
2
n 1 p ab(n 1) n 1 an b (n 1) (a b) n
Again q

=

= (n 1)2 (bn a) ... (B)
bn a
n 1
m 1 n 1

n 1 2 n 2 (a b) 2 (a b) 2
Now (q p) q
p = (n 1)2 (bn a) 2 0
n 1
2

n 1 p
Hence q can not lie between p and

n 1

96

ARTHMETIC PROGESSION
2.
If S1, S2, S3, ... Sn are the sums of infinite geometric series whose first terms are 1, 2,3, ..., n
and whose common ratios are

1 1 1
1
, , ,...,
respectively, then find the value of
2 3 4
n +1

S12 +S 22 +S 32 + ...+ S 2n-12 .

[1991]

Solution :
1

S1

1
2

2, S2

1
3

3,

...(i)

Now S12 S22 + ... + S22 + ... S22 (n 1)


= 22 + 32 + .... + (24)2 = S1 12 + s2 .... + (24)2} 12
=

3.

2n (24 1) (4n 1)
n(2n 1) (4n 1)
1
6
3

Let a1, a2, ... be positive real numbers in geometric progression. For each n, let An, Gn, Hn be
respectively, the arithmetic mean, geometric mean and harmonic mean of a1, a2, ... an. Find
an expression for the geometric mean of G1, G2, ..., Gn in terms of A1, A2, ... , An, H1, H2, ... Hn.
[2001]
Solution :
a1 a2 ... a4 are in G.P.
a2 = a1r : a3 = a1r2 ... an = a1rn1

An =

... (1)

a 1 a1
(1 r r 2 ...r n 1 ) by (1)
n
n

or An

a1 r 1

... (2)
n r 1

Gn = (a1 a2 ... an )1/n = a1(1.r..r2 ...n) ... rn1)1/n


orGn = a1 [r(1/2)

(n 1)

]1/4 = a1r1/2(n 1) ... (3)

1 1 1
1 1
1
1

. [1 r 2 ... n 1 ]
=
H n n a1
n a1
r
r
1 1 1
1 1 1 1
1 r n 1 1
1

.
1

...
= b.a r 1 r n 1
H n x a1 = n a1 r r 2
r n 1

na1 r n 1 (r 1)
... (4)
Hn =
r n 1
from (2) (3) & 4)

97

IIT- MATHS
2
n

2
1

G = Hn An = a

n1

i(r ) above is true for each n, hence

G12 . G22 ... Gn2 = (A1 A2 ... A4) (H1 ... H4)
(G1 ... G4)1/4 = [(A1A2 ... An) (H1 H2 ... Hn)]1/2x

LHS is G.M/. of G1, G2, G3, ... Gx whose value we have determined in terms of A1A2 ... An
and H1 H2 ... Hn .

4.

The fourth power of the common difference of an arithmetic progression with integer entries is added to the product of any four consecutive terms of it. Prove that the resulting
sum is the square of an integer.
[2002]
Solution :

Any four consecutive integers in A.P be taken as (a 3d), (a d), (a + d), (a + 3d) so that
common difference is 2d and their product is (a2 9d2) (a2 d2)
we have to evaluate
(2d)4 + (a2 9d2) (a2 d2)
a4 10a2 d2 + 25d2 = (a2 5d2)2
(a2 5d2)2 is also an enteger.

5.

Let a, b, be positive real numbers If a, A1, A2, b are in arithmetic progression, a, G1 , G2, b
are in geometric progression and a, H1, H2 , b are in harmonic progression, show that
G1G 2 A1 + A2 (2a + b)(a + 2b)
=
=
.
H1 H 2 H1 + H 2
9ab

[2002]

Solution :
a, H1, H2, b are in H.P
1 1 1 1
a , H , H , b are A.P..
1
2

1 1
3D
b a

11 1
D
33 a

1 1
1 1 1 2b a

3
H a
a b a 3ab
1
1 1
1
1
1 1 2b a
D 3 3

H1 a
a a b a 3ab
98

ARTHMETIC PROGESSION
G1G 2 ab (ab a) (2a b)
(2b a) (2a b)
=
2 2
HH
9a b
9ab
1 2

6.

If a, b, c are in A.P. and a2, b2, c2 are in H.P., then prove that either a/2, b, c are in G.P.
or a = b = c.
[2003]
Solution :
2b = a + c
ab, b, c are in H.P ... (1)

b2

2a 2 c 2
a2, b2, c2 are in A.P.
a 2 c2

... (2)

b2 [(a + c)2 2ac] = 2a2c2

b2 [(4b2 2ac)] = 2a2 c2


2b4 b2ac a2 c2 = 0 factorize
b2 ac = 0 or 2b2 + ac = 0
2

a c ac
if b = ac then
by

2
2

(a + c)2 4ac = 0 or (a c)2 = 0 a = c


and 2b = a + c = a + a = 2a
b=a

b=a
a=b=c
if 2b2 + ac = 0 then b2 =

1
c
2

a
, b, c are in G.P..
2

7.

Prove that (a + 1)7 (b + 1)7 (c + 1)7 > 77 a4 b4 c4, where a, b, c, R


Solution :
(a + 1) (b + 1) (c + 1)
= abc + ab + bc + ca + a + b + c + 1
> abc + ab + bc + (a + a + b + c = 7
4 4 4 1/7
7 (a b c )

99

abc ab bc ca a c
7

[2004]

IIT- MATHS
Thus (a + 1) (b + 1) (c + 1) > 7 (a4 b4 c4)1/7
(a + 1)7 (b + 1)7 (c + 1)7 > 77 (a4 b4 c4)

8.

An infinite G.P has first term x and sum 5, then find the exhaustive range of x ?
[2004]

100

ARTHMETIC PROGESSION
Illustration 1:
The mth term of an A.P. is n and its nth term is m. Prove that its pth term is m + n p. Also
show that its (m + n) th term is zero.
Solution:
Given Tm = a + (m 1) d = n and Tn = a + (n 1) d = m
Solving we get, d = 1 and a = m + n 1

Tp = a + (p 1)d = m + n 1 + (p 1) (1) = m + n p

Now, Tm + n = a + (m + n 1)d = (m + n 1) + (m + n1) (1) = 0.


Illustration 2:
1
2
Find the number of terms in the series 20, 19 , 18 , ..... of which the sum is 300. Explain the
3
3
double answer.
Solution:

Clearly here a = 20, d

2
and Sn = 300.
3

n
2
2 20 (n 1) 300
2
3

Simplifying, n2 61n + 900 = 0 n = 25 or 36.


Since common ratio is negative and S25 = S36 = 300, it shows that the sum of the eleven terms
i.e., T26, T27 , ....., T36 is zero.

Illustration 3:
The first term of an infinite G..P is 1 and any term is equal to the sum of all the succeeding
terms. find the series.
Solution:
Given that Tp = (Tp+1 + Tp+2 + ..... )
or,

arp1 = arp + arp+1 + arp+2 + ...

rp1 =

1r=r r=

rp
[sum of an infinite G.P.]
1 r
1
2

1 1 1
Hence the series is 1, , , , ... .
2 4 8

Illustration 4:
101

IIT- MATHS
If a1, a2, a3, ... an are in harmonic pregression, prove that
a1a2 + a2a3 + ... + an1 an = (n 1) a1an .
Solution:
Since a1, a2 , ... , an are in H.P.,
1 1 1
1
,
, ,...,
a1 a 2 a 3
a n are in A.P. having common difference d (say) .

1 1
1 1
1
1
d, d, ...
d
a 2 a1
a3 a 2
a n a n 1

or

a1 a2 = d(a1a2), a2 a3 = d (a2a3), ... , (an1 an) = d(an1 an )

Adding the above relations, we get


a1 an = d (a1a2 + a2a3 +... + an1 an)

... (1)

1
1
Now a a (n 1)d
n
1

1 1
(n 1)d
a n a1

or

(a1 an) = (n 1) d an a1

... (2)

Putting the value of a1 an from (2) in (1), we get


(n 1) an a1d = d (a1a2 + a2a3 + ... + an1 an)

(n 1) ana1 = a1a2 + a2a3 + ... + an1 an.

Illustration 5:
If A1, A2; G1 , G2 and H1 , H2 be two A.M.s, G.M.s and H.M.s between two quantities a and
b then show that A1H2 = A2 H1 = G1G2 = ab
Solution:
a, A1 , A2, b be are in A.P.
... (1)
a, H1 , H2 , b are in H.P.

1 1 1 1
, ,
,
a H1 H 2 b are in A.P..

Multiply by ab.

b,

ab ab
,
, a are in A.P.
H1 H 2

take in reverse order.


or

a,

ab ab
,
, b are in A.P..
H 2 H1

... (2)

Compare (1) and (2)


ab
ab
and A 2
H2
H1

A1

A1H2 = A2H1 = ab = G1G2


102

ARTHMETIC PROGESSION
Illustration 6:
ab

a b
Prove that

ab.ba , a,b N;a b.

Solution:
Let us consider b quantities each equal to a and a quantities each equal to b. Then since
A.M. > G.M.
(a a a ...b times) (b b b ...a times)
[(a.a.a...b times) (b.b.b. ... a times)]1/(a+b)
ab

ab ab
(a b ba )1/(a b)
ab

2ab
(a b ba )1/(a b)
ab

Now

a b 2ab

(A.M. > H.M.)


2
ab
ab

a b

a b .b a .

Illustration 7:
Prove that a4+ b4 + c4 abc (a + b + c), [a, b, c > 0]
Solution:
Using mth power inequality, we get
a 4 b4 c4 a b c

3
3

a b c a b c

3
3

or

abc
1/ 3 3

[(abc) ]
3

( A.M G.M)
.M)

a 4 b 4 c4 a b c

abc
3
3

4
4
4
a + b + c abc (a + b + c).

Illustration 8:
Prove that

s
s
s
9

, if s = a + b + c, [a, b, c > 0]
sa sb sc 2

Illustration 9:
Find the sum of n terms of the series 3 + 7 + 14 + 24 + 37 + ... .
Solution:
Clearly here the differences between the successive terms are
7 3, 14 7, 24 14, ... i.e., 4, 7, 10, ... which are in A.P.

Tn = an2 + bn + c
Thus we have 3 = a + b + c, 7 = 4a + 2b + c and 14 = 9a + 3b + c

103

IIT- MATHS

3
2

Solving we get, a , b

1
,c 2 .
2

1
Hence Tn (3n 2 n 4)
2

Sn =

1
[3n 2 n 4n]
2

1 n(n 1) (2n 1) n(n 1)


n
3

4n (n 2 n 4)
2
6
2
2

Illustration 10:
Find the sum of n terms of the series 3 + 8 + 22 + 72 + 266 + 1036 + .....
Solution:
1st difference 5, 14, 50, 194, 770, ...
2nd difference 9, 36, 144, 576, .....
They are in G.P. whose nth term is arn1 = a4n1
Tn of the given series will be of the form

Tn = a4n1 + bn + c
T1 = a + b + c = 3
T2 = 4a + 2b + c = 8
T3 = 16a + 3b + c = 22
Solving we have a = 1, b = 2, c = 0.
Tn = 4n1 + 2n

1
Sn = 4n 1 2n (4 n 1) n(n 1) .
3

104

ARTHMETIC PROGESSION

PROGRESSION AND SERIES


ARITHMETIC PROGRESSION (A.P.)
If a is the first term and d the common difference, the A.P. can be written as a, a+d, a+2d,...
The nth term a n is given by a n=a+(n-1)d. The sum S n of the first n terms of such an A.P. is
given by Sn

n
n
( 2a ( n 1)d) ( a l) where l is the last term (i.e., the nth term of the A.P.).
2
2

nth Term and Sum of n Terms :

If a fixed number is added (subtracted) to each term of given A.P. then the resulting
sequence is also an A.P. with the same common difference as that of the given A.P.

If each term of an A.P. is multiplied by a fixed number (say k) (or divided by a nonzero fixed number), the resulting sequence is also an A.P. with the common
difference multiplied by k.

REMARKS :

If a 1, a 2, a 3,.... and b1, b2, b3,..... are two A.P.s with common differences d and d'
respectively then a 1+b1, a 2+b2, a 3+b3 ,.... is also an A.P. with common difference d+d'.

If we have to take three terms in an A.P., it is convenient to take them as a-d, a,


a+d. In general, we take a-rd, a-(r-1)d,...... a-d, a, a+d,.... a+rd in case we have to
take (2r+1) terms in an A.P.

If we have to take four terms, we take a-3d, a-d, a+d, a+3d. In general, we take
a-(2r-1)d, a-(2r-3)d,..... a-d, a+d,....a+(2r-1)d, in case we have to take 2r terms in
an A.P.

If a 1, a 2, a 3,.... a n are in A.P. then a 1+a n= a 2+a n-1= a 3+a n-2 = ..... and so on.

ar

If three terms are in A.P., then the middle term is called the arithmetic mean (A.M.)

a r k a r k
k , k n r, k 0
2

between the other two i.e., if a, b, c are in A.P., then b

ac
is the A.M. of a and c.
2

Arithmetic Mean(s):

If a 1, a 2,.... a n are n numbers, then the arithmetic mean (A) of these numbers is
A

1
(a 1 a 2 a 3 .....a n )
n

The n numbers A1, A2,...... An are said to be A.M.s between the numbers a and b
if a, A1, A2,....., Anb are in A.P. A n a

105

n ( b a ) a nb

.
n 1
n 1

IIT- MATHS

GEOMETRIC PROGRESSION (G.P.)


nth Term and Sum of n Terms :
If a is the first term and r the common ratio, then G.P. can be written as a, ar, ar 2,....
The nth term, a n, is given by a n = ar n-1. The sum S n of the first n terms of the G.P. is

a (r n 1)
Sn
, r 1
r 1
na ,
r 1
If -1<r<1, then the sum of the infinite G.P. is a +ar + ar 2+.....

a
1 r

REMARKS :

If each term of a G.P. is multiplied (divided) by a fixed non-zero constant, then the
resulting sequence is also a G.P. with same ratio as that of the given G.P.

If each term of a G.P. (with common ratio r) is raised to the power k, then the resulting
sequence is also a G.P. with common ratio r k .

If a 1, a 2, a 3, .... and b1, b2 , b3,.... are two G.P.s with common ratios r and r respectively,
then the sequence a 1b1, a 2b2, a 3b3.... is also a G.P. with common ratio rr'.

If we have to take three terms in G.P., it is convenient to take them as a/r, a, ar. In
general, we take

a a
, k 1 ,..., a , ar ,..., ar k in case we have to take (2k+1) terms in a G.P..
k
r r

If we have to take four terms in a G.P., it is convenient to take them as a/r 3, a/r, ar,
ar 3. In general, we take

a
r

2 k 1

a
r

2 k 3

a
,....., , ar ,.....ar 2 k 1 , in case we have to take 2k terms
r

in a G.P.

If a 1, a 2,...., a n are in G.P., then a 1a n = a 2a n-1 = a 3a n-2 = ........

If a 1, a 2, a 3,...... is a G.P. (each a 1 > 0), then loga 1, loga 2 , loga 3...... is an A.P. The
converse is also true.

Geometric Means :

If three terms are in G.P., then the middle term is called the geometric mean (G.M)
between the two. So if a, b, c are in G.P., then b ac is the geometric mean of
a and c.

If a 1, a 2,...... a n are non-zero positive numbers, then their GM. (G) is given
G = (a 1a 2a 3 ......a n) 1/n

If G 1, G 2,..... G n are n geometric means, between a and b, then a, G 1, G 2,...., G n, b

106

ARTHMETIC PROGESSION
b
will be a G.P. G n a n 1
a

HARMONIC PROGRESSION (H.P.)


1 1 1
1
The sequence a 1, a 2, a 3,.....a n (a i 0) is said to be an H.P. if the sequence a , a , a ,.... a ,....
1
2
3
n

is an A.P.
1
1
1 1
The nth term, a n of the H.P. is a n a (n 1)d , where a a , and d a a .
1
2
1

NOTE : There is no formula for the sum of n terms of an H.P.

If a and b are two non-zero numbers, then the harmonic mean of a and b is a number
H such that the numbers a, H, b are in H.P. We have

1 11 1
2ab
H
.
H 2a b
ab

If a 1, a 2,..... a n are n non-zero numbers, then the harmonic mean H of these numbers
is given by 1 1 1 1 ..... 1
H n a 1 a 2
a n

The n numbers H 1, H 2,....., H n are said to be n-harmonic means between a and b,


1 1 1
1 1
if a, H 1, H 2......, H n, b are in H.P. i.e. if a , H , H ..... H , b are in A.P..
1
2
n
1
1 n (a b )
b na

H n a (n 1)ab ab(n 1)

ARITHMETIC - GEOMETRIC PROGRESSION :


The sum of S n of first n terms of an A.G.P. is obtained in the following way :
Sn = ab + (a + d)br + (a + 2d)br 2 +.....+ (a + (n - 2)d)br n-2 + (a + (n - 1)d)br n-1
Multiply both sides by r, so that
rS n = abr + (a+d)br 2 +....+ (a + (n - 3)d)br n-2 + (a + (n - 2)d)br n-1 + (a + (n - 1)d)br n
Subtracting, we get
(1-r)S n=ab+dbr+dbr 2+.....+dbr n-2+dbr n-1-(a+(n-1)d)br n

dbr (1 r n 1 )
ab
(a (n 1)d )br n
(1 r )

Sn
107

ab dbr (1 r n 1 ) (a (n 1)d )br n

1 r
(1 r ) 2
1 r

IIT- MATHS
If -1 < r < 1, the sum of the infinite number of terms of the progression

lim
n

Sn S

ab
dbr

1 r (1 r ) 2

MISCELLANEOUS PROGRESSION :
Some Important Results :
n ( n 1)
(sum of the first n natural numbers)
2

1 2 3 ..... n

12 2 2 32 ..... n 2

13 23 33 .... n 3

n ( n 1)(2n 1)
(sum of squares of the first n natural numbers)
6

n 2 (n 1) 2
(1 2 3 .... n ) 2 (sum of cubes of first n natural
4

numbers)

1 + x + x2 + x3 + ..... = (1 - x) -1 ,

if -1 < x < 1

1 + 2x + 3x2 + ..... = (1 - x) -2 ,

if -1 < x < 1

INEQUALITIES :
Let a 1, a 2,.....,a n be n positive real numbers, then we define their arithmetic mean (A), geometric
mean

(G)

and

harmonic

n
1 1 1
1
.....
an
a1 a 2 a 3

mean

(H)

as

a 1 a 2 ..... a n
,
n

G=(a 1 a 2 .....a n ) 1/n

and

It can be shown that A>G>H. Moreover equality holds at either place if and only if a 1=a 2=...=a n .

Weighted Means :
Let a 1, a 2,....., a n be n positive real numbers and m1, m2,....., mn be n positive rational numbers.
Then we defined weighted Arithmetic mean (A*), weighted Geometric mean (G*) and Weighted
mean (H*) as
A*

m1a 1 ..... m n a n
m
m
m
, G* a 1 1 .a 2 2 ......a n n
m1 m 2 .... m n

1
( m1 m 2 .... m n )

and H*

m1 m 2 .... m n
m1 m 2
m

..... n
an
a1 a 2

It can be shown that A* > G* > H*. Moreover equality holds at either place if and only if
a1 = a 2 = ..... = a n .

Arithmetic Mean of mth Power :

108

ARTHMETIC PROGESSION

Let a 1, a 2,......, a n be n positive real numbers (not all equal) and let m be a real number, then
m

a 1 a 2 ...... a n
a a 2 ..... a n
1
if m R [0, 1]
n
n

a 1 a 2 ..... a n
a a 2 ..... a n
1

However if m (0,1), then


n
n

Obviously if m {0,1}, then

a 1 a 2 ..... a n
a a 2 ..... a n
1

n
n

Cauchys Inequality :
If a 1's and bis are reals, then (a 12 + ..... + a n2) (b12 + .... + bn2) > (a 1b1+a 2b2+....+a nbn) 2 .

Proving Inequalities :
(i)

Any inequality has to be solved using a clever manipulation of the previous results.

(ii)

Any inequality involving the sides of a triangle can be reduced to an inequality involving
only positive reals, which is generally easier to prove.
For the triangle we have the constraints
a + b > c, b + c > a, a + c > b
Do the following : put

x = s-a, y=s-b, z=s-c

then, x+y+z=3s-2s=s and a=y+z, b=x+z, c=x+y


Substitute a=y+z, b=x+z, c=x+y in the inequality involving a, b, c to get an inequality
involving x, y, z.
Also note that the condition a+b>c is equivalent to
a + b + c > 2c i.e., 2s > 2c OR s - c > 0, i.e., z > 0
Similar b + c > a = x > 0, a + c > b = y > 0

109

The inequality obtained after the substitution is easier to prove. (involving only
positive reals without any other constraints).

IIT- MATHS

WORKED OUT ILLUSTRATIONS


ILLUSTRATION : 01
The sum of n terms of two series in A.P. are in the ratio 5n +4 : 9n + 6. Find the ratio of their 13th
terms.
(A)

129
131

(B)

127
132

(C)

125
134

(D)

121
139

Solution :
Ans : (A)
Let be the first terms of two A.P.s and are their respective common differences.

n 1 d
n
a1
2a1 n 1 d1 5n 4
1
5n 4
2
2

==> n
==>
n 1 d 9n 6
2a2 n 1 d 2 9n 6
a2
2
2
2

.................. (1)

a1 12d1
Now the ratio of 13th terms = a 12d
2
2

==> put

n 1 12
2

a1 12d1 5 25 4 129
i.e. n = 25 in equation (1) ==> a 12d 9 25 6 231

2
2

ILLUSTRATION : 02
At what values of parameter 'a' are there values of n such that the numbers : 51 x 51 x ,

a
x
, x
2 25 25

form an A.P. ?
(A) a 8

(B) a 8

(C) a 12

(D) a 12

Ans : (C)
Solution :
For the given numbers to be in A.P.,
Let 5x k
==> a 5k

5
1
k2 2
k
k

1 2 1

==> a 5 k k 2
k
k

As the sum of a positive number and its reciprocal is always greater than or equal to 2,
k

1
1
2 and k 2 2 2 ;
k
k

Hence a 5 2 2 ==> a 12

110

ARTHMETIC PROGESSION
ILLUSTRATION : 03
1
1
1
1

............
1.2.3 2.3.4 3.4.5 4.5.6

Find the sum of first n terms of the series :

1
1
(A) 2 2 n 1 n 2

1
1
(B) 4 n 1 n 2

1
1
(C) 2 n 1 n 2

1
1
(D) 4 2 n 1 n 2

Ans : (D)
Solution :

1
1
1
1
Let S = 1.2.3 2.3.4 3.4.5 ............ n n 1 n 2

n 2 n
3 1 4 2 5 3
2S = 1.2.3 2.3.4 3.4.5 ............ n n 1 n 2

1 1
1
1
1
1
2S = 1.2 2.3 2.3 3.4 .......... n n 1 n 1 n 2

1
1
2S = 1.2 n 1 n 2

==>

1
1
S = 4 2 n 1 n 2

Note : You should observe that here,


Tn

1
1
1
1

n n 1 n 2 2 n n 1 n 1 n 2

It is in the form f(n) - f(n+1).


ILLUSTRATION : 04
Find the three digit number whose consecutive number form a G.P. If we subtract 792 from this
number, we get a number consisting of the same digits written in the reverse order. Now if we increase the
second digit of the required number by 2, the resulting number will form an A.P.
(A) 901

(B)931

(C)981

Ans : (B)
Solution :
Let the three digit be a, ar, ar2 then according to hypothesis
100a + 10ar + ar2 +792 = 100ar2 + 10ar + a
==> a r 2 1 8 ..................................................(1)
111

(D)991

IIT- MATHS
and a, ar + 2, ar2 are in A.P.
then 2 ar 2 a ar 2
==> a r 2 2r 1 4

...............................................(2)

Dividing (1) by (2),


a r 2 1

8
Then a r 2 2r 1 4

r 1 r 1 2
2
r 1

==>

r=3

r 1
2
r 1

==>

from (1), a =1

Thus digits are 1, 3, 9 and so the required number is 931.


ILLUSTRATION : 05
Sum to n terms of the series.

cos ec 1 10 cos ec 1 50 cos ec 1 170 ........... cos ec 1


1
(A) tan n 1

1
(B) tan n 1

1 n 2 2n 2 is.............

1
(C) tan n 1

1
(D) tan n 1

Ans : (A)
Solution :

Let cos ec1


==>

1 n 2 2n 2

cosec2 =
=

1 n 2 2n 2

1 2n n 2 1 n 2 1

==>

cot2 =

==>

tan

n 1

1 n 2 1 2n 1
2

n 1 1

n 1 n
1

n n 1 1 n 1 n
2

n 1 n
1
1
tan 1
tan n 1 tan n
1

1
n

==>

Thus, sum to n terms of the given series.


=

tan

2 tan 1 1 tan 1 3 tan 1 2 +

tan

4 tan 1 3 ........ tan 1 n 1 tan 1 n

112

ARTHMETIC PROGESSION
= tan 1 n 1 tan 1 1 tan

n 1

ILLUSTRATION : 06
If a, b, c are in G.P. ,and loga - log2b, log2b - log3c. and
sides of a triangle which is
(A) acute - angled

(B) obtuse - angles

are in A.P., then are the lengths of the

(C) right - angled

(D) equilateral

Ans : (B)
Solution :
We have b 2 ac and 2 log 2b log 3c = log a log 2b log 3c log a
2a
4a
and 2b 3c ==> b
and c
3
9

==> b 2 ac
since a b

5a
10a
13a
b , therefore are the sides of a
> c, b c
>a and c a
3
9
9

triangle. Also, as a is the greatest side, let us find angle A of ABC.

cos A

b 2 c 2 a 2 29

0
2bc
48

Hence, ABC is an obtuse-angled triangle.


ILLUSTRATION : 07
If log x a, a x / 2 and log b x are in G.P. then is equal to
(A) log a log b a

(B) log a log e a log a log e b

(C) log a log a b

(D) log a log e b log a log e a

Ans : (a) and (b)


Solution :
As log x a, a x / 2 , log b x are in G.P..
x/2 2

log x a.log b x

x
==> a

log a log x log a


.

log b a
log x log b log b

==> x log a log b a log a log e a log a log e b

ILLUSTRATION : 08

113

IIT- MATHS
If there unequal positive real numbers a, b, c are in G.P. and b c, c a, a b are in H.P. then the
value of a b c 1is independent of
(A) a

(B) b

(C) c

(D) None of these

Ans : (D)
Solution :
As are in G.P., b 2 ac and b c, c a, a b are in H.P..

ac
2
1
1

= b c a b
c a b c a b
==> 2 b c a b a c
==> 2 ab ac b 2 bc =
==> 2 ab 2b 2 bc
==> 2b

a c

a c

a c

a c

a c

a c

a c

a c 0

==> 2b a c 2 ac a c 2b
==> a b c 3 ac
Which is not independent of a, b and c.

ILLUSTRATION : 09
An A.P. whose first term is unity and in which the sum of the first half of any even number of terms
to that of the second half of the same number of terms is in constant ratio, the common difference d is
given by
(a) 1

(b) 2

(c) 3

(d) 4

Ans: (B)
Solution :
Let denote the sum to n terms of the A.P.
According to the given condition
Sn
k n 1
S 2n S n
S1
S2
==> S S S S
2
1
4
2

==> S1 S4 - S1S2 = -S1S2


114

ARTHMETIC PROGESSION
==> S1S 4 S22

2
4

==> a 2a 4 1 d a a d
2

==> a 4a 6d 2a d

==> 4a 2 6ac 4a 2 4ad d 2

==> 2ad d 2 ==> 2a d As a =1, we get d = 2.

ILLUSTRATION : 10
n

If

t
r 1

n
1
1
n n 1 n 2 n 1 , Then lim
is
n

6
r 1 t r

(a) 2

(b) 3

Ans : (A)
Solution :
We have, for n 1,
n 1

t n t r t r
r 1

r 1

1
1
n n 1 n 2 n 1 n n 1
6
6

1
1
n n 1 n 2 n 1 n n 1
6
2

1
2
1
1
Now, for r 1 , t r r 1 2 r r 1

r
n

n
1
1
1
1

==> t
r r 1 2 1 n 1
r 1 r
r 1
n

1
1

lim 2 1

==> lim
2 1 0 2 z
n
n

n 1
r 1 t r

115

(c) 3/2

(d) 6

IIT- MATHS

SECTION A

SINGLE ANSWER TYPE QUESTIONS


1.

The mth term of an A.P is n and its nth term is m. Its pth term is
A) m +n+p

2.

B) m+n-p

B) x = y -1

If

B) (2p+1)(p+1)2

D) p3 + (p+1)3

D) p3,q3,r3 are in A.P

A.G.P consists of an even number of terms. If the sum of all the terms is 5 times the sum of the
terms occupying odd places, the common ratio will be
B) 3

If b-c, 2b-x, b-a are in H.P then a A) A.P

7.

C) (p+1)3

B) p2,q2,r2 are in A.P C) 1/p, 1/q,1/r in A.P

A) 2
6.

D) z2 = x

1
1
1
,
,
are in A.P then
q+r r+p p+q

A) p,q,r are in A.P


5.

C) z-3 = y

The first term of an A.P of consecutive integers is p2 + 1. The sum of (2p +1) terms of this series
can be expressed as
A) (p+1)2

4.

D) m-n-p

If 1, log y x , log z y -15log x z are in A.P then


A) z3 = x

3.

C) m-n+p

C) 4

D) 5

x
x
x
,b,care in
2
2
2

B) G.P

If S 1,S 2 ,S 3 denote the sums of

C) H.P

D) none of these

n1 , n2 , n3 t erms respectively of an A.P then

s1
s
s
n 2 - n 3 + 2 n 3 - n1 + 3 n1 - n 2 is equal to
n1
n2
n3

A) 0
8.

B) 1

10.

D) n1n2n3

If the interior angles of a polygon are in A.P with common difference 50 and smallest angle is
1200 then the number of sides of the polygon is
A) 9 or 16

9.

C) S1,S2,S3

B) 9

C) 16

D) 13

If the arithmetic mean of two positive numbers a and b (a > b) is twice their geometric mean then
a : b is
A) 2 +

3 :2-

C) 6+

7 :6-

3
7

B) 5 +

: 5-

D) 1 +

2 :1- 2

If m is a root of the equation (1-ab)x2 - (a2+b2)x - (1+ab) = 0 and m harmonic means are inserted
between a and b then the difference between the last and the first of the means equals
A) b - a

B) ab( b - a )

C) a (b-a)

D) ab(a-b)
116

ARTHMETIC PROGESSION
11. If a1 , a 2 a 3 , .... a n are in H.P then a1a 2 + a 2 a 3 + a 3a 4 + . . . . . + a n-1a n is equal to
A) n
12.

C) 2n

1
mn

B)

1 1

m n

C) 1

D) 0

If the sum of the roots of the quadratic equation ax2 + bx + c = 0 is equal to the sum of the square
of their reciprocals then

a b c
, ,
are in
c a b

A) Arithmetic - Geometric progression


C) Geometric progression
14.

D) (n-2)

Let Tr be te rth term of an A.P for r =1,2,3. If for some positive integers m, n we have Tm= 1/n and
Tn = 1/m, then Tmn is
A)

13.

B) (n-1)

B) Arithmetic progression
D) Harmonic progression

Suppose a,b >0 and x1 , x2, x3 (x1 > x2 > x3) are the roots of

x-a x-b
b
a
+
=
+
b
a
x -a x -b

and

then a,b,c are in


A) A.P
15.

18.

B) 1

D) does not exist

tan 2
x2 + x

C) 0 < x < 10

D) x > 10

is always greater than equal to


D) sec2

C) 2

Let the positive numbers a,b,c,d be in A.P. Then abc, abd, acd, bcd are
B) A.P

C) G.P

D) H.P

If a,b,c,d are +ive real no such that a+b+c+d =2 then M = (a+b) (c+d) satisfies the relation
B) 1 M 2

C) 2 M 3

D) 3 M 4

Let S (0, ) denote the set of values of x satisfying the equations 81 |cosx|+ cos2x + |cos3x|
+.... to = 43 then S = z
2
B) ,
3
3

2
C) ,
3 3

2
D) ,
3 3

Sum of the series 1+2.2 + 3.22 + 4.23 +.....+100.299 is


A) 100.2100+1

117

B) - 10 < x < 0

A) 2 tan


A)
3

21.

C) 3/4

x2 + x -

A) 0 M 1
20.

B) 1

If (0, /2) then

A) Not in A.P/G.P/H.P
19.

D) none of these

An infinite G.P has first term 'x' and sum then x belongs to
A) x < - 10

17.

C) H.P

If is the nth term of a G.P with first term 1 and common ratio r, then the minimum value of is
A) 1/4

16.

B) G.P

B)99.2100+1

C)99.299-1

D)100.2100-1

IIT- MATHS
22.

If a,b,c are in arithmetic progression then


A)A.P.

23.

B) G.P.

If

B) 17

D) None of these

C) 27/14

D) 56/15

cosec 2 q, 2cotq, sec , 0 < <


are in G.P. iff ? is equal to
2
2

A) /6
25.

C) H.P.

If the sum of first n terms of a series be 5n 2 + 2n , then its second term is


A) 16

24.

1
1
1
,
,
are in :
a+ b a+ c b+ c

If 3 +

B) /4

C) /3

D) None of these

1
1
3 + d + 2 3 + 2d + ....... to =8, then the value of is
4
4

A) 9

B) 5

C) 1

D) None of these

a n 1
n
log
n1
C) 2
b

a n 1
n
log
n 1
D) 2
b

ar
log
r-1 =
The value of
r=1
b
n

26.

an
n
log
n
A) 2
b
27.

The roots of the equation


A) form an A.P.

28.

- 4 | x -1 | +3 = 0

B) form a G.P.

C) Form a H.P.

B) -1

x -1

D) do not form any progression


2

- 4 | x -1 | +3 = 0

C) 1/2

B) are in A.P

1 1 1
2
+
+
+
....to
If 2
=
1 22 32
6

2
A)
8
31.

D) None of these

c
A 3b
If in a ABC acos2
+ccos2 =
then the sides a,b,c
2
2 2

A) satisfy a+b=c
30.

x -1

1 a n + bn 1
If , n+1 n+1 , are in A.P., then n is equal to
a a +b
b
A) 0

29.

a n 1
n
log
n
B) 2
b

If

2
B)
12

C) are in G.P

D) are in H.P

then equals :

2
C)
3

2
D)
2

a + be y b + ce y c + de y
=
=
then, a,b,c,d are in
a - be y
b - ce y
c - de y

A) A.P.

B) G.P.

C) H.P.

D) None of the above

118

ARTHMETIC PROGESSION
32. If f x is a function satisfying f x + y = f x f y for all x, y N , such that f 1 = 3
n

f x = 120. then the value of n is

and

x =1

A) 4
33.

34.

B) 5

C) 6

The sum of all the products of first n positive integers taken two at a time is
A)

1
n 1 n n 1 3n 2
24

B)

C)

1
n n 1 n 2 n 5
6

D) None of the above

B) G.P only when x > 0


D) G.P only when x < 0

If the A.M of the roots of a quadratic equation is 8/5 and the A.M of their reciprocals is 8/7, then
the quadratic equation is
A) 7x2 + 16x +5 = 0

36.

1
n 2 n n 1 n 2
48

If a, b, c are in A.P then 10ax+10, 10bx+10,10cx+10 are in


A) A.P
C) G.P for all x

35.

D) None of these

B) 7x2 - 16x + 5 = 0

C) 5x2-16x + 7 =0

D) 5x2 - 8x + 7 = 0

, be roots of the equation x2 - 3x + a =0 and , be roots of x2 - 12x + b = 0 and numbers

, , , form an increasing G.P then


A) a=3 b=12
37.

B) a=12 b=3

B) 11 : 10

C)

5 1
,
2

5 1
2

A) 0

B)

3 , 1/ 3

D)

3 1
,
2

B) 2+

sinx + cosx
is
sinx - cosx

C) 2 - 3

B) ab (b -a)

D) 1 +

C) a (b - a)
1

119

3 1
2

If m is a root of the equation (1-ab)x2 - (a2 + b2) x - (1+ab) = 0 and m harmonic means are
inserted between a and b then the difference between the last and the first of the means equals
A) b - a

41.

D) 1 : 2

If for 0 < x < /2 y = exp [ sin 2 x sin 4 x ..... ] is a zero of the quadratic equation
x2 -9x + 8 =0 then the value of

40.

C) 17 : 15

If the sides of a right angles triangle are in A.P then the sines of the actute angles are
A) 3/5, 4/5

39.

D) a=4 b=16

Let a,b, be roots of x2-3x+p=0 and Let c,d be the root of x2-12x + q= 0 where a,b,c,d form an
increasing G.P. Then the ratio of p+q : q - p is equal to
A) 8 : 7

38.

C) a=2 b=32

D) ab (a - b)
1

{an} and {bn} are two sequences given by an = x 2n + y 2n and bn = x 2n - y 2n

for all n =

IIT- MATHS
N. The value of a1, a2 . . .an is equal to
A) x - y
42.

xy
D) b
n

B) 20/9

C) 9/20

x2 -

D) 11/23

Suppose a,b,c are in A.P and a2,b2,c2 are in G.P. If a < b < c and a + b + c = 3/2 then the value of
a is
A)

44.

x y
C) b
n

If exp {tan2x - tan4x+tan6x - tan8 x +..} loge16 0 < x < /4, satisfies the quadratic equation
3x + 2 = 0 the value of cos2x + cos4x is equal to
A) 21/16

43.

x y
B) b
n

1
2 2

B)

C)

2 3

1 1

2
3

D)

1 1

2
2

If . . . . . . . are in G.P then the value of the determinant

is
A) -2
45.

46.

B) 1

C) 2

D) 0

If (a,b) (c,d) (e,f) are the vertices of a triangle such that a,c,e are in G.P with common ratio 'r' and
b,d,f ax is G.P with common ratio 's' then area of the triangle is
A)

ab
r 1 ( s 2)( s r )
2

B)

ab
r 1 s 1 s r
2

C)

ab
r 1 s 1 s r
2

D)

r 1 s 1 s r

The roots of equation x2 + 2(a-3) + 9= 0 lie between -6 and 1 and 2,h1,h2...h20[a] are in H.P, where
[a] denotes the integral part of a and 2, a1a 2 ... a 20 ,[a] are in A.P then a3h18 =
A) 6

47.

D) none of these

B) G.P

C) H.P

D) none of these

If the A.M of two numbers is twice of their geometric mean, then ratio of sum of numbers to the
difference of numbers equals
A) 2

49.

C) 18

If a,b,c,d and p are distinct real numbers such that (a2+b2+c2)p2 - 2(ab+bc+cd)p + (b2+c2+d2) 0
then a, b, c, d are in
A) A.P

48.

B) 12

B) 3 / 2

C) 4

D) 2/

Let , be the roots of x2 - x + p = 0 and , be the roots of x2 - 4x + q =0. If , , , are


in G.P, then the integral values of p and q respectively are
A) -2, -32

B) -2,3

C) -6,3

D) -6, -32
n

50.

r , r r < r are the roots of x 2 - r 2 r +1 x + r 5 =0 The value of

+ 2 r is

r=1

120

ARTHMETIC PROGESSION
1
2
A) n n 1 n 3n 1
2
C)
51.

3
n n 1 n 2 n 1
2

2 1

bc b 2

If for 0 < x <

A) 0

C) 12

B)

1 3 2 1

4 c 2 ca a 2

C)

3
2

2
b ab

2
4
6
, y = exp is a zero of the quadratic sin x + sin x + sin x + ....... to log2
2
sinx + cosx
is :
sinx - cosx

B) 2 3

C) 2 3

D) none of these

B) a b c a

C) 3ac

D) 3bd

B) bx 2 c

C) cx 2 d

B) 5000

C) -5050
2

58.

D) x a

The coefficient of x99in the expansion of (x - 1) (x - 2) ..... (x - 100) is


A) 5050

57.

D) None of the above

If a,b,c,d are in G.P., then a factor of ax 3 + bx 2 + cx + d is


A) ax 2 c

56.

D) None of these

If a,b,c,d are in H.P., then ab + bc + cd is


A) 3ad

55.

1
n n 1 n 2 n 3
2

B) 10

equation then the value of the value of

54.

D)

1 1 1 1 1 1
If a , b,c are in H.P., then the value of + - + - is
b c a c a b

A)

53.

1
n n 1 3n 2 n 1
2

1+x
1-x a
x
-x
The least value of 'a' for which 5 + 5 , , 25 + 25 are three consecutive terms of an A.P. is
2
:

A) 5
52.

B)

D) -5000
2

1 2 1 3 1

The sum of the first 10 terms of the series x + + x + 2 + x + 3 + ..... is


x
x
x

x 20 1 x 22 1
A) x 2 1 x 20 20

x18 1 x11 1
B) x 2 1 x 9 20

x18 1 x11 1
C) x 2 1 x9 20

D) None of these

When are in A.P., and when

1 1 1 1 1
1 1 1 5
, , , , are in A.P., + + = then the numbers a and b
a x y z b
x y z 3

are :
A) 8,2

121

B) 9,1

C) 7,3

D) none of these

IIT- MATHS
59.

Let a and b be the roots of x - 3x + p = 0 and let c and d be the roots of x -12x + q = 0,
where a,b,c,d form in increasing G.P. Then the common ratio of q + p : q - p is
A) 8:7

60.

B) 8

B) 2

D) 4

C) 3a2

D) 4a2

r r 1
B) 2 x

r r 1
C) 2 x xy

r r 1
D) 2 x rx

B) second term
D) last term

If the numbers a,b,c,d,e form an A.P then the value of a - 4b + 6c - 4d + e is


B) 2

C) 0

D) 3

If ax = by = cz = dt and a,b,c,d are in G.P then x,y,z,t are in


A) A.P

67.

D) 10

In an A.P the sum of terms equidistant from the beginning and end is equal to

A) 1
66.

C) 3

B) 2a2

A) first term
C) sum of 1st and last terms
65.

C) 9

n
A series whose nth terms is +y then sum of r terms will be
x

r r 1
A) 2 x ry

64.

D) none of these

The length of a side of a square is 'a' metre. A second square is formed by joining the middle
points of this square. Then a third square is formed by joining the middle points of the second
square and so on. Then the sum of the area of the squares which carried upto infinity is
A) a2

63.

C) 17 :15

If the roots of the equation are in A.P., then the common difference will be
A) 1

62.

B) 11:10

The sum of the cubes of first n natural numbers does not exceed 1360, the maximum value of n
is
A) 7

61.

B) G.P

C) H.P

D) none of these

If 1 ,2 ,3 ...... such that a1 + a 5 + a10 + a15 + a 20 + a 24 = 225, then

a1 + a 2 + . . . . . a 23 + a 24

equal to
A) 909
68.

C) 750

D) 900

C) 3ac

D) 3cd

If a,b,c,d are in H.P then ab+bc+cd is equal to


A) 3ad

69.

B) 75

B) (a+b)(c+d)

A1 +A2
If A1, A2 be two A.Ms and G1,G2 be two G.Ms between a and b then G G is equal to
1 2

A)

a b
2ab

B)

2ab
a b

C)

a b
ab

D)

a b
ab
122

ARTHMETIC PROGESSION
70.

a1
a2
a3
If a1,a2,a3. . .an are in H.P then a + a +. . . + a + a + a +. . . + a + a + a +. . . + a +. . . are in
2
3
n
1
3
n
1
2
n

A) A.P
71.

B) G.P

If

r=1

r=1

B) 3/2

D) a = 3d

1
is equal to
tr

C) 3/4

D) 3/8

The sixth term of an A.P is 2 and its common difference is greater than one. The value of the
common difference of the progrenion so that the product of the first, fourth and fifth terms is
greatest is
B) 2/3

C) 5/8

D) 3/2

Consider an infinite geometric series with first term 'a' and common ratio 'r'. If its sum is 4 and
the second term is 3/4 then
A) a = 7/4, r = 3/7

75.

C) 2a = d
n

A) 8/5
74.

B) a = d

t r = 2 3 n - 1 " n1 then lim


x

A) 3
73.

D) none of these

consider an A.P with first term 'a' and common difference 'd' Let Sk denotes the sum of the first
k terms. If Skx/Sx is independent of x then
A) a = 2d

72.

C) H.P

B) a = 2, r = 3/8

C) a=3/2, r =1/2

D) a=3, r = 1/4

If a1a 2 . . .. . a10 be in A.P and h1 h2 .....h10 be in H.P. If a1 = h1 = 2 and a10 = h10 = 3 then a4
h7 is equal to
A) 2

76.

B) 3

B) are in A.P
D) are in H.P

If 2(y-a) is the H.M between y - x , y - z then x - a , y - z then x - a, y - a, z - a are in


A) A. P

B) G.P

C) H.P

21

78.

If

= 693 where a , a ..........a are in A.P. then


1
2
21

A) 361

B) 396

2i+1

is

i=0

C) 363

D) data insufficient

If the roots of the equation x3-12x2+39x-28=0 are in A.P, then their common difference will be
A) 1

80.

D) none of these
10

a
j=1

79.

D) 6

If the system of linear equations x + 2ay + az = 0 ; x + 3by + bz = 0 ; x + 4cy + cz = 0 has a


non-zero solution then a,b,c
A) satisfy a+2b+3c =0
C) are in G.P

77.

C) 5

If

B) 2

C) 3

D) 4

are the +ve real numbers where product is a fixed number c, then the minimum value of

a1 + a 2 + a 3 + ........ a n -1 +2an is
1/ n

A) n 2c
123

B) n 1 c1/ n

C) 2nc1/ n

D)

1/ n

n 1 2c

81.

IIT- MATHS
If the sum of the first 2n terms of the A.P. 2,5,8...... is equal to the sum of the first n terms of the
A.P. 57,59,61....... Then n equals
A) 10

82.

B) 12

B) H.P

C) G.P

D) none of these

The harmonic mean of 2 numbers is 4. Their arithmetic mean is A and geometric mean is G. If G
satisfies 2A + G2 = 27, the numbers are
A) 1,13

84.

D) 13

If the lines a1y +b1x - a1b1 = 0, a2 y + b2 x = a2 b2 cut the co-ordinate axes in co cyclic points then
a1, b1, b2 a2 may be in
A) A.P

83.

C) 11

B) 9,12

C) 3,6

D) 4,8

If a1 , a 2 , a 3 ,.......a n are in Arithmetic series with common difference 'd' .The value of sin d (cosec
a1 cosec a2 + cosec a2 cosec a3 + .......+ cosec an-1 cosec an ) is .....
A) sec a1 sec an

85.

B) cos eca1 cos ecan C) tan a1 tan an

D) cot a1 cot an

The first term of an A.P. of consecutive integers is p2 +1. The sum of (2p+1) term of this series
can be expressed as:
A) (p+1)2

86.

B) (2p + 1) (p + 1)2

The sum of the first n terms of the series


A) 2n - n + 1

87.

B) 1 - 2-n

D) p3 + (p + 1)3

1 3 7 15
+ + + + ....... =
2 4 8 16

C) n + 2-n - 1

D) 2n - 1

Let a, b, c form a G.P. of common ratio r with 0 < r < 1. If a, 2b and 3c form an A.P., then r equals
:
A) 1/2

88.

C) (p + 1)3

B) 1/3

C) 2/3

D) None of these

The geometric and harmonic means of two numbers x1 and x2 are 18 and 16

8
respectively. The
13

value of | x1 - x 2 | is
A) 5
89.

B) 10

l 2 a2
B)
2S a l

l 2 a2
C) 2 S a l

D) None of these

If ax3 + bx2 + cx + d is divisible by ax2 + c then a, b, c, d are in


A) A.P.

91.

D) 20

If S denotes the sum of first n terms of the A.P. a + (a + d) + (a + 2d) +........ whose last term is l,
then the common difference 'd' of the A.P. is
la
A)
n

90.

C)15

B) G.P.

The sum of
A)

x 2

n2

x + 2

n-1

x 1

+ x + 2

C) H.P.
n-2

n-3

D) None of these

x +1 + x + 2 x +1
B) x 2

n 1

+ ....... + x +1

x 1

n-1

is equal to :

n 1

124

ARTHMETIC PROGESSION
n
n
C) x 2 x 1
92.

Given two numbers a and b, Let A denote the single A.M. and S denote the sum of n A.M.'s
between a and b, then
A) n, a, b

C) n, a
c

B) G.P

C) H.P

A)

n digits

4
10n 1

B)

4
102n 1
9

C)

2
4
10n 1

Given that 0 < x < ?/4, ?/4 < y < ?/2 and

tan

2k x cot 2k y

k=0

1
B) 1 1 1
p q pq

The sum of the first n terms of the series

A)

97.

6n
n 1

B)

9n
n 1

If In =

B) log 34

k cot 2k y= q

then

k=0

C) p + q + pq

D) p+q+pq

3
5
7
+...
2 + 2
2 + 2
1
1 +2
1 + 22 + 32

C)

12n
n 1

D)

15n
n 1

C) 1 - log3 4

D) log 3 0.25

1 - sin2nx
dx, then I1 , I 2 , I3 . .. . are in
1 - cos2x

B) G.P

C) H.P

D) none of these

For any odd integer n 1 n3 - (n-1)3 + . . . .+ (-1)n-1 13 is equal to


2

A)

125

k=0

-1

A) A.P
99.

k tan 2k x= P

4 n
10 2
9

1- x
+ 2 and log9 (4.3x -1 ) are in A.P, then x is equal to
If 1, log 9 3

A) log 4 3
98.

-1

D)

is

1 1 1
A)
p q pq

96.

D) none of these

(666... . 6)2 + (88.....8) is equal to


n digits

95.

D) n

1 1
1
If x > 1 and , , are in G.P, then a, b, c are in
x x
x

A) A.P
94.

S
depends on :
A

B) n, b
a

93.

D)None of the above

n 1 2n 1
4

n 2 2n 1
B)
4

C)

n 1 2n 1
4

D)

n 1 2n 1
4

IIT- MATHS
1 3 7 15
+ + + + . . . . is
2 4 8 16

100. The sum of first n terms of the series


A) 2n - 1

B) 1 - 2-n

C) 2-n - n + 1

D) 2-n + n -1

101. Let two numbers have arithmetic mean 9 and geometric mean 4. Then these numbers are the
roots of the quadratic equation
A) x2 - 18x - 16 =0

B) x2 - 18x + 16 =0

C) x2 + 18x - 16 =0

D) x2 + 18x + 16 =0

n n +1
102. The sum of first n terms of the series 1 +2.2 + 3 + 2.4 + 5 + 2.6 + .. . . is
2
2

when n

is even. When n is odd the sum is


n n 1
A)

B)

103. If in H.P and f(k) =


A) A.P

C)

D)

then
B) G.P

are in
C) H.P

D) none of these

104. If cos (x - y), cosx, cos (x +y) are in H.P then cosx sec y/2 is equal to
A)

B)

C) 2

D) none of these

105. 0.2 + 0.22 + 0.222 + . . . . . to n terms is equal to


A)
106. Let

B) nbe the roots of x2 - x + p = 0 and

C)

D)

be the roots of x2 - 4x + q = 0 If

are

in G.P. then integral values of p and q respectively are :


A) -2, -32

B) -2,3

C) -6, 3

107. If a,b,c and d are in G.P.,


A)-1
108. If

B) 2
, b,

A) A.P.

are in A.P. then


B) G.P.

D) -6, -32
is

C) 0

D) 1

are in :
C) H.P.

D) None of these

C) a + d > b +c

D) none of the above

109. If a,b,c, d are in H.P. then :


A) a + b > c +d

B) a +c > b +d

126

ARTHMETIC PROGESSION
110. Consider an A.P., with first term 'a' and common difference 'd'. Let
terms. If

is independent of x, then

A) a = 2d

B) a =d

C) 2a = d

denote the sum of first k

D) None of these

111. A G.P. consists of an even number of terms, if the sum of all the terms is 5 times the sum of the
terms occupying odd places, the common ratio will be equal to
A) 2

B) 3

C)4

112. The value of x, for which

A)
113. The coefficient of

D)5
and 1 are in A.P. is

B)

C)

D)

in the polynomial given by


is

A) 5511
114. If
which

B) 5151

C) 1515

D) 1155

are three consecutive terms of a G.P. with common ratio r, the value of r for
holds is given by

A) 1< r < 3

B) -3 < r < -1

C) r >3 or r <1

D) none of the above

115. If x1 x2 x3 as well as y1 y2 y3 are in G.P with some common ratio then (x1y1) (x2y2) (x3y3)
A) lie on a straight line

B) lie on an ellipse

C) lie on circle

D) are vertices of triangle

116. Let f(x) be a polynomial function of second degree if f(1)=f(-1) and a, b, c are in A.P then f--1(a),
f1(b), f1(c) are in
A) Arithmetic - Geometric progression
C) G.P

127

B) A. P
D) H.P

IIT- MATHS

SECTION - B
MULTIPLE ANSWER TYPE QUESTIONS
1.

If tan-1x, tan-1y, tan-1z are in A.P and x, y, z are also in A.P., (y being not equal to 0, 1 or -1), then
A) x, y, z are in G.P.
C) x = y = 1

2.

B) x, y, z are in H.P.
D) (x-y)2 + (y-z)2 + (z-x)2 = 0

If d, e, f are in G.P. and the two quadratic equations ax2 + 2bx + c = 0 and dx2 + 2ex + f = 0 have
a common root, then
A)

are in H.P..

B)

C) 2dbf = aef + cde


3.

4.

A) 1 -

: -2 : 1 +

B) 1 : -

:-

C) 1 -

D) 1 +

:-2:1-

If a, b, c, d are distinct positive numbers then the inequality an + an > bn + cn holds for every
positive integer n provided a, b, c, d are in

9.

D) increasing progression

B) Tpq = p+q

C) Tp+q > Tpq

D) Tpq > Tp+q

B) a + c = b

C) a

D) ac = b2

If a, b, c be three unequal positive quantities in H.P., then


A)

8.

C) H.P

If the first and (2n-1) th terms of an A.P., a G.P. and a H.P., are equal and their nth terms are a, b
and c respectively, then
A) a = b = c

7.

B) G.P

The pth term Tp of an H.P is q (p+q) and qth term Tq is p (p+q) when p > 1, q >1 then
A) Tp+q = pq

6.

D) b2 df = ace2

If three unequal numbers p, q, r are in H.P., and their squares are in A.P., then the ratio p : q : r is

A) A.P
5.

are in G.P..

B)

C)

The sum of n terms of the series

is

A)

B)

C)

D)

Given that 0 < x <

/4 and

/4 < y <

D)

/2 and

= a,

, then

128

ARTHMETIC PROGESSION

10.

A)

B) a+b-ab

C)

D)

If a, b, c are in H.P., then the value of

is

A)

B)

C)

D) none of these

A)

B)

C)

D)

11.

12.

If 1, logyx, logzy, - 15 logxz are in A.P., then


A) z3 = x
C) z -3 = y

13.

n2

B) x = y-1
D) x = y-1 = z3

I f b1, b2, b3 (b1 > 0) are three successive terms of a G.P. with common ratio r, the value of r for

which the inequality b3 > 4b2 - 3b1 holds is given by


A) r > 3
C) r = 3.5
14.

B) r < 1
D) r = 5.2

If logx a, ax/2 and logb x are in GP., then x is equal to


A) loga (logba)
C) -loga (loga b)

15.

B) loga (logea) - loge (logeb)


D) loga (loge b) - loga (logea)

If a, b, c are in H.P., then


A)

are in H.P.,

B)
C) aD)

129

are in G.P..
are in H.P.,

IIT- MATHS
16.

If
A) a = 1/2

17.

then
B) b = 8/3

C) c = 9/2

D) e = 0

I f 1, log9 (31-x+2) and log3(4.3x-1) are in A.P., then x is equal to

A) log43

B) log3 4

C) 1 - log3 4

D) log3 (0.75)

130

ARTHMETIC PROGESSION

131

IIT- MATHS

BASIC TRIGONOMETERY

132

BASIC TRIGONOMETRY

DEFINITION OF TRIGONOMETRIC RATIOS


Let a revolving line OP starts from OM and revolves into the position OP, thus tracing out the
angle MOP.
Draw PM perpendicular to the initial line OM.
In the triangle MOP, OP is the hypotenuse, PM is the perpendicular, and OM is the base.
The trigonometrical ratios, or functions, of the angle MOP are defined as follows.

Perp.
MP
, i.e.,
, is called the Sine of the angle AOP;
Hyp.
OP
Base
OM
, i.e.,
, is called the Cosine of the angle AOP;
Hyp.
OP
MP
Perp.
, i.e.,
, is called the Tangent of the angle AOP;
OM
Base

Base
OM
, i.e.,
, is called the Cotangent of the angle AOP;
Perp.
MP
OP
Hyp.
, i.e.,
, is called the Secant of the angle AOP;
OM
Base

Hyp.
OP
, i.e.,
, is called the Cosecant of the angle AOP;
Perp.
MP

FUNDAMENTAL RELATIONS BETWEEN THE TRIGONOMETRICAL


RATIOS OF AN ANGLE
It is clear from the definitions that if one of the trigonometric ratios of an angle is known, the
numerical magnitude of each of the others is also known.
Let the angle MOP (fig. of previous article) be , then

133

sin2 + cos2 = 1 or

sin2 =1 cos2 or cos2 = 1 sin2

1 + tan2 = sec2 or sec2 tan2 = 1

1 + cot2 = cosec2 or cosec2 cot2 = 1

tan =

sin . cosec = tan . cot = cos . sec = 1

It is possible to express a trigonometrical ratio in terms of any one of the other ratios:

sin
cos
and cot
cos
sin

IIT- MATHS
1

e.g. sin

, cos

1 cot

cos ec 1 cot 2 , sec

cot
2

, tan

1 cot

1
cot

1 cot 2
cot

i.e., all trigonometrical functions have been expressed in terms of cot . This is left as an exercise
for you to derive other results of this type.

SIGNS OF TRIGONOMETRIC RATIOS


Tracing of the changes in the sign and magnitude of the trigonometrical ratios of an angle, as the
angle increases from 0 to 360.
Let the revolving line OP be of constant length a.
When it coincides with OA, the length OM1 is equal to a;
and, when it coincides with OB, the point M1 coincides
with O and OM1 vanishes. Also, as the revolving line turns
from OA to OB, the distance OM1 decreases from a to zero.

Whilst the revolving line is in the second quadrant


and is revolving from OB to OA , the distance OM2 is
negative and increases numerically from 0 to a [i.e., it
decreases algebraically from 0 to a].

P1

P2

M2

M1
M3

P3

M4

P4
B

In the third quadrant, the distance OM3 increases algebraically from a to 0; and, in the fourth
quadrant, the distance OM4 increases from 0 to a.
In the first quadrant, the length M1P1 increases from 0 to a; in the second quadrant, M2P2 decreases
from a to 0; in the third quadrant, M3P3 decreases algebraically from 0 to a; whilst in the fourth quadrant
M4 P4 increases algebraically from a to 0.
Therefore it is clear that

134

BASIC TRIGONOMETRY
B
i

O
i

PERIODS OF THE TRIGONOMETRICAL FUNCTIONS


As an angle increases from 0 to 2 radians. i.e., whilst the revolving line makes a complete
revolution, its sine first increases from 0 to 1, then decreases from 1 to 1, and finally increases from 1
to 0, and thus the sine goes through all its changes, returning to its original value.
Similarly, as the angle increases from 2 radians to 4 radians, the sine goes through the same
series of changes.
Also, the sines of any two angles which differ by four right angles, i.e., 2 radians, are same.
This is expressed by saying that the period of the sine function is 2 .
Similarly, the cosine, secant, and cosecant go through all their changes as the angle increases
by 2 .
The tangent, however, goes through all its changes as the angle increases from 0 to radians, i.e.,
whilst the revolving line turns through two right angles. Similarly for the cotangent.
The period of the sine, cosine, secant and cosecant is therefore 2 radians; the period of the
tangent and cotangent is radians.
Since the values of the trigonometrical functions repeat over and over again as the angle increases,
they are called periodic functions.

GRAPHS OF THE TRIGONOMETRIC RATIOS


The variations in the values of the trigonometric ratios may be graphically represented to the eye
by means of curves constructed in the following manner.

135

IIT- MATHS
Y

Sine-Graph:

/ 2

1
O /2

Y
1
O

/ 2

Cosine-Graph :

/ 2

3 / 2 2

Tangent-Graph:
/ 2

3 / 2X

/2

Cosecant-Graph:

2
X

3
2

1
O

The secant-graph and the cotangent-graph are left as an exercise to the students.

136

BASIC TRIGONOMETRY

TRIGONOMETRIC RATIOS OF SOME ANGLES


Angle

30

45

60

1
2

1
2
1
2

3
2
1
2

3
2
1
2

1
3
2
3

1
3
2
3

3
2
1
3
3

2
2
3

90

120

135
1
2
1
2

150

180

1
2
3
2
1
3
3

2
2

2
3

Note: Later on we shall learn that infact tan(90) = and tan(90+) = etc. ]

TRIGONOMETRIC RATIOS FOR AN ANGLE OF ANY MAGNITUDE


Complementary Angles
Two angles are said to be complementary when their sum is equal to a right angle. Thus any angle
and the angle 90 are complementary..

Supplementary angles
Two angles are said to be supplementary when their sum is equal to two right angles, i.e., the
supplement of any angle is 180 .

Allied or Related Angles


1
1
n and n , where n is any integer, are known as allied or related angles.
2
2
The trigonometric functions of these angles can be expressed as trigonometric functions of ,

The angles

with either a plus or a minus sign. The following working rules can be used in determining these functions.
1.
Assuming that 0 < < 90, note the quadrant in which the given angle lies. The result has
a plus or minus sign according as the given function is positive or negative in that quadrant.
2.
If n is even, the result contains the same trigonometric function as the given function. But
if n is odd, the result contains the corresponding cofunction, i.e., sine becomes cosine, tangent becomes
cotangent, secant becomes cosecant and vice-versa.

137

IIT- MATHS
P2

P1
90

+ve

A ve
O

+ve

+ve A
ve

ve

P3

P4

Here

angle AOP1 =
angle AOP4 (measured clockwise) =
angle AOP2 = 180
angle OP1A = 90 and similarly the other angles.
It is clear from the figure that when
equals

sin

cos

tan

cot

sec

cosec

sin

cos

tan

cot

sec

cosec

90

cos

sin

cot

tan

cosec

sec

90 +

cos

sin

cot

tan

cosec

sec

180

sin

cos

tan

cot

sec

cosec

180 +

sin

cos

tan

cot

sec

cosec

360

sin

cos

tan

cot

sec

cosec

360 +

sin

cos

tan

cot

sec

cosec

TRIGONOMETRIC RATIOS OF COMPOUND ANGLES


An angle made up of the algebraic sum of two or more angles is called compound angle. Some
formulae and results regarding compound angles:

sin (A + B) = sin A cosB + cosA sinB

sin(A B) = sinA cosB cos A sinB

cos (A + B) = cosA cos B sinA sin B

cos(A B) = cosA cosB + sin A sin B.

tan A tan B
1 tan A
tan(A + B) = 1 tan A tan B , tan (45 + A) = 1 tan A

tan A tan B
tan(AB) = 1 tan A tan B ,

cot A cot B 1
cot A cot B 1
cot (A + B) = cot A cot B , cot (A B) = cot B cot A

sin(A + B) sin(A B) = sin2A sin2B = cos2B cos2A

1 tan A
tan (45 A) = 1 tan A

138

BASIC TRIGONOMETRY
cos(A + B) cos(A B) = cos2A sin2B = cos2B sin2A.

tan A tan B tan C tan A tan B tan C


tan (A + B + C) = 1 tan A tan B tan B tan C tan C tan A

.MAXIMUM AND MINIMUM VALUES OF acos + bsin


Let a = rsin , b = rcos so that r =

a2 b2

also, acos + bsin = r(cos sin + sin cos ) = rsin ( )


Now the maximum and minimum values of sin( ) are 1 & 1 respectively..
Hence r rsin( ) r

a 2 b 2 a cos b sin a 2 b 2

Hence the maximum value =

a2 b2

and minimum value is a 2 b 2 .

TRIGONOMETRIC RATIOS OF MULTIPLES OF AN ANGLE


2 tan A
1 tan 2 A

sin2A = 2sinA cosA =

1 tan 2 A
cos2A = cos A sin A = 1 2 sin A = 2 cos A1 =
,
1 tan 2 A
2

1 + cos2A = 2cos2A, 1 cos2A = 2sin2A


2 tan A
1 tan 2 A

tan2A

sin3A = 3sinA 4sin3A = 4sin(60 A) sinA sin(60 + A)

cos3A = 4 cos3A 3cosA = 4cos(60A) cosA(cos60+A)

3 tan A tan 3 A
tan 3A
= tan(60A) tanA tan(60+A)
1 3 tan 2 A

PRODUCT OF SINES/COSINES IN TERM OF SUMS

2 sinA cosB = sin (A + B) sin (A B)

2 cos A sin B = sin (A + B) sin (A B)

2 cos A cos B = cos (A + B) + cos (A B)

2 sin A sin B = cos (A B) cos (A + B)

SUM OF SINES/COSINES IN TERM OF PRODUCTS

139

sinC + sinD = 2sin

CD
CD
cos
2
2

IIT- MATHS

sinC sinD = 2 cos

CD
CD
sin
2
2

cosC + cosD = 2cos

CD
CD
cos
2
2

cosC cosD = 2sin

CD
CD
sin
2
2

tanA + tanB =

sin ( A B)
sin ( A B)
, tanA tanB =
cos A cos B
cos A cos B

TRIGONOMETRIC RATIO OF SUBMULTIPLE OF AN ANGLE

| sin

A
A
cos | 1 sin A
2
2

or sin

A
3

A
A
ve, if 2 n 2n
cos 1 sin A
4 2
4
2
2
ve, otherwise

A
A
cos | 1 sin A
2
2

| sin

or

A
A
sin cos

2
2

tan

A
5

2n
ve, if 2n
1 sin A
4
2
4

ve, otherwise

A tan 2 A 1 1

2
tan A

The ambiguities of signs are removed by locating the quadrant in which

A
lies or one can use the
2

following figure.
sin

A
A
cos is ve
2
2

sin
sin

sin

A
A
cos is ve
2
2

A
A
cos is ve
2
2
sin

sin

A
A
cos is ve
2
2

|a cosA + bsinA|
Also cosA sinA =

sin

sin

A
A
cos is ve
2
2

A
A
cos is ve
2
2

A
A
cos is ve
2
2

A
A
cos is ve
2
2

a2 b2

2 sin 4 A =

2 cos A
4

140

BASIC TRIGONOMETRY

IDENTITIES
A trigonometric equation is an identity if it is true for all values of the angle or angles involved. A
given identity may be established by (i) reducing either side to the other one, or (ii) reducing each side to
the same expression, or (iii) any convenient modification of the methods given in (i) & (ii).

CONDITIONAL IDENTITIES
When the angles A, B and C satisfy a given relation, many interesting identities can be established
connecting the trigonometric functions of these angles. In providing these identities, we require the
properties of complementary and supplementary angles. For example, if A + B + C = , then

sin (B + C) = sinA, cosB = cos (C + A)

cos (A + B) = cosC, sinC = sin(A + B)

tan (C + A) = tanB, cotA = cot(B + C)

cos

sin

CA
B
A
BC
cos , sin cos
2
2
2
2

tan

BC
A
B
CA
cot , tan cot
2
2
2
2

AB
C
C
AB
sin , cos sin
2
2
2
2

Some important identities:


If A, B, C are angles of a triangle (or A + B + C = ):

141

tanA + tanB + tanC = tanA tanB tanC

cotA cotB + cotB cotC + cotC cotA = 1

tan

A
B
B
C
C
A
tan + tan tan + tan tan 1
2
2
2
2
2
2

cot

A
B
C
A
B
C
cot cot cot cot cot
2
2
2
2
2
2

sin2A + sin2B + sin2C = 4sinA sinB sinC

cos2A + cos2B + cos2C = 1 4cosA cosB cosC

sinA + sinB + sinC = 4cos

cosA + cosB + cosC = 1 + 4 sin

A
B
C
cos cos
2
2
2
A
B
C
sin sin
2
2
2

IIT- MATHS

TWO SIMPLE TRIGONOMETRICAL SERIES

n
2 (n 1)
sin
sin
2
2

sin + sin( )+sin ( 2 ) + ... + sin{ ( n 1) }=

sin
2

n
2 (n 1)
cos
sin
2
2

cos +cos( )+cos( 2 )+ ... +cos{ ( n 1) }=

sin
2

142

BASIC TRIGONOMETRY

WORKED OUT ILLUSTRATIONS


ILLUSTRATION : 01

sin 3
cos3

is equal to
sin cos sin cos

(a) sin 2

(b) cos 2

(c) sin cos

(d) sin 2

Ans : (C)
Solution :
We can write the given expression as

sin cos sin 2 cos 2 sin cos


sin 3 cos3
1
1
sin cos
sin cos
=1- ( 1 sin cos )
= sin cos
ILLUSTRATION : 02
If cos

(a) 1

2cos 1
0 , , then tan
2 cos
2

(b)

(c)

(d)

tan

is equal to
2

1
Z
3

Ans : (C)
Solution :
From the given relation we have

2cos 1 2 cos 2 cos 1


1 cos = 1 2 cos
2 cos

or


2cos 2
1 cos
2
2cos 2

2 2 cos

1 2sin 2
2

or


cos 2

2
cos 2
2 1 2sin 2 ........... (1)

2




cos 2
1 2sin 2 cos2

2
2
2
1 cos 2 1
==>

2


1 2sin 2
1 2sin 2
2
2

143

IIT- MATHS

3sin 2

2
sin 2
or
2

1 2sin 2
2

............ (2)

From (1) and (2) we get



tan
2 3
2
2
tan
3 tan
==>

2
2
tan
2
ILLUSTRATION : 03
If Pn cos n sin n , then Pn Pn 2 kPn 4 where
(a) k 1

(b) k sin 2 cos 2

(c) k sin 2

(d) k cos 2

Ans : (B)
Solution :
We have Pn Pn 2 cos n sin n cos n 2 sin n 2
= cos n 2 cos 2 1 sin n 2 sin 2 1
= sin 2 cos n 2 cos 2 sin n 2
= sin 2 cos 2 cos n 4 sin n 4
= sin 2 cos 2 Pn 4 kPn 4
where k = sin 2 cos 2

ILLUSTRATION : 04
If

(a)

sin 4 cos4
sin 8
cos8

,
then
+
is equal to
a2
b2
a3
b3
1
a b

1
(b) a b 2

(c)

a b

(d) a b

Ans : (C)
Solution :

sin 4 cos 4

k
Let
a2
b2

144

BASIC TRIGONOMETRY
k

then 1 sin 2 cos 2 a k b k ==>

a b

sin 8 cos8 k 2 a 4 k 2b 4
1
2
so that a 3 b3 a3 b3 k a b a b 3

ILLUSTRATION : 05
sin 3 A sin 3 1200 A sin 3 240 0 A is equal to

(a) (9/4) sinA

(b) (3/4) sin3A(c) - (3/4) sin3A

(d) 0

Ans : (C)
Solution :
The given expression is equal to
1
0
0
0
0
3sin A sin 3 A 3sin 120 A sin 360 3 A 3sin 240 A sin 720 3 A
4

3
1
0
0
0
0
= sin A sin 180 60 A sin 180 60 A sin 3 A sin 3 A sin 3 A
4
4

3
3
0
0
sin A sin 60 A sin 60 A sin 3 A
4
4

3
3
0
= sin A 2 cos 60 sin A sin 3 A
4
4
3
= sin 3 A
4

ILLUSTRATION : 06
If tan , tan , tan are the roots of the equation x 3 px 2 r 0 then the value of (1 + tan?) (1 +
tan2 ) (1 + tan2 ) is equal to
2

(a) p r (b) 1 p r

(c) 1 p r

Ans : (B)
Solution :
From the given equation we have

tan tan 0 and

tan p

tan tan tan r

so that 1 tan 2 1 tan 2 1 tan 2

145

(d) None of these

IIT- MATHS
= 1 tan 2 tan 2 tan 2 tan 2 tan 2 tan 2
=1 +

tan

2 tan tan tan tan

2 tan tan tan tan tan 2 tan 2 tan 2


= 1 + p 2 2 pr r 2
= 1 p r

ILLUSTRATION : 07
If A and B be acute positive angles satisfying 3sin 2 A 2sin 2 B 1, 3sin 2 A 2sin 2B 0
then
(a) B

4 2

(b) B

4 2

(c) B

2 4

(d) A

4 2

Ans : (A)
Solution :
From the given relations we have
3
Sin2B = sin 2 A
2

and 3sin 2 A 1 2sin 2 cos 2 B


3
sin 2 A
so that tan 2 B 2
cot A or 1- tan2BtanA =0
3sin 2 A
==> A+2B =

A
==> B
2
4 2

ILLUSTRATION : 08
If

(a)

a c
cos x cos x cos x 2 cos x 3

then
is equal to
b d
a
b
c
d
a
d

(b)

c
d

(c)

b
c

(d)

d
a

Ans : (C)
Solution :

146

BASIC TRIGONOMETRY
cos x cos x 2
2 cos x cos
a c
b
= b d cos x cos x 3 = 2 cos x 2 cos c

ILLUSTRATION : 09
sec 2

4 xy

x y

(a) x y 0

is true if and only if


(c) x y

(b) x y , x 0

(d) x 0, y 0

Ans : (C)
Solution :
2

Since sec 2 1 so 4 xy x y x y 0 x y
Which is true if and only if x y 0 , because for x y 0 , sec 2 becomes indeterminant.

ILLUSTRATION : 10
cos A sin C
If cos B sin C , then is equals

(a) tan

A B tan A B tan C

(b) tan

A B tan A B tan C

(c) tan

A B tan A B sin C

(d) tan

A B tan A B cos C

Ans : (B)
Solution :
cos A sin C

cos B sin C
cos A cos B sin C sin C
==> cos A cos B sin C sin C

==> tan

A B
AB
tan
cot C tan
2
2

==> tan tan

147

A B
A B
tan
tan C
2
2

IIT- MATHS

SECTION A
SINGLE ANSWER TYPE QUESTIONS
1.

If cos20 - sin 20 = P the cos40 is equal to


A) - p

2.

2 p2

B) p 2 p 2

B) 0

B) 8 cos2A

B) p2 = q (q+2)

The period of
A) 2

9.

B) -1/2

D) f(x) 2

is

C) 1/2

D) 1

B) 1

C) 3

D) 1

C) /2

D) /4

sin2x
is
3cos4x

B)

The period of sin (x + 4x + 9x+ ......... n2x ) is

4
C) n n 1 2n 1

3
D) n n 1 2n 1

C) 1/4 and 4

D) 1/6 and 6

The value of tan3 cot can't lie between


A) 1/2 and 2

11.

C) 2 < f(x) < 1

3
5
7
9
+ cos
+ cos
+ cos
+ cos
11
11
11
11
11

12
6
A) n 2n 1 n 1 B) n 2n 1 n 1
10.

D) p2 =q2 (q2 - 4)

Maximum value of 2sin2x - cos2x is


A) 2

8.

D) 4cos2A

C) p2 + q2 = 2q

B) f(x) = 1

The value of cos


A) 0

7.

C) 1/8 cos2A

If f(x) = cos2x + sec2x its value always is


A) f(x) < 1

6.

D) 1/2

If sec and cosec are the roots of x2-px+q=0 then


A) p2 = q (q-2)

5.

C) 1

sin 2 3A cos 2 3A
is equal to
sin 2 A cos 2 A
A) cos2A

4.

D) none of these

If tan2 - 2 tan2 +1 then cos2 + sin2 is equal to


A) -1

3.

C) p 2 p 2

B) 1/3 and 3

The maximum value of cos 1, cos 2 cos 3.......cos n under the restriction
0 1 , 2 , 3 ,......... n

A) 1/2n/2

and cot 1, cot 2 ....... Cos n = 1 is


2

B) 1/2n

C) -1/2n

D) 1

148

BASIC TRIGONOMETRY

5
12. If tan , x and tan
9
18
A) 2x = y
13.

B) x > y

tan

D) 1/5

B) (0,1]

C) [1/3 )

D) [1 )

C) 2

D) 4

B) 2

C) 3

D) 4

If 3sin2A + 2sin2B = 1 and 3sin2A - 2sin2B = 0 where A and B are acute angles, then (A + 2B) is
equal to
B) /4

C) /2

D) /6

The number of solutions of the equation |sinx| = |cos3x| in [-2 , 2 ] is


B) 28

1
1
2
+
+
4
4
2
sec cosec sec + cosec 2

B) 1

C) 24

D) 30

C) sin2

D) cos2

If sin + cosec = 2 then sinn + cosecn =


B) 2n

C) 0

D) 2

If x2 = 1 - tan2 then tan3 cosec + sin =


A)

149

If tan x - tan2x = 1 then the value of tan4x - 2tan2x - tan2x + 2tanx + 1 is

A) n
23.

C) 3/4

B) 1

A) 0
22.

D)

The no. of roots of equation x sinx = 1 in the interval 0 < x < 2 is

A) 32
21.

1 2
2 2

Let ABC be a acute angled triangle such that A = /3 and cot B cos C = P. The possible values of
P will be

A) /3
20.

C) 1

B) 1/2

A) 1
19.

D)

If sin ( cos ) = cos ( sin ), then sin2 is equal to

A) 0
18.

C) 1/8

B) 1/ 3

A) (0, 1/3]
17.

D) x = 2y

- 3 tan
tan
- tan
is equal to
5
15
5
15

A) 1/4
16.

are also in A.P. then

C) x = y

B) cos /8

A) 3
15.

7
, y and tan
9
18


3
5
7

1 + cos 1 + cos 1 + cos 1+ cos is equal to


8
8
8
8

A) 1/2

14.

are in A.P. and tan

2 1/ 2

2 x

B)

3/ 2

3/ 2

C) 2 x 2

D)

2 5/ 2

2 x

IIT- MATHS
24.

The value of the expression 1 A) 1

1- cos 2 A
sinA 1 + cosA
+
is equal to
1 + cosA 1 - cosA
sinA

B) tanA

C) cosA

D) sinA

25.

If

sin
i=1

= n , then cos 1 + cos 2 + cos 3 +......+cos n =

A) n
26.

B) n-1

For 0 < <

if , x =
2

A) xyz = xz + y
27.

D) zero

cos 2n , y = sin 2n , Z =

n =0

n=0

n =0

B) xyz = xy+z

cos 2n sin 2n , then

C) xyz = x+y+z

D) xy2 = y2+x

C) m [-2, 2]

1
D) m , 1
4

If sin6 + cos6 a = m, then


1 1
A) m ,
6 6

28.

C) 2n

B) m 0, 1

The smallest value of in the first quadrant which satisfies the equation
exp {(1+cos2 +cos4 +cos6 +..... )loge16} = 256 is
A) /4

29.

If x =

B) /6

B) cos

B) zero

C) x

D) 13

B) a2 + 2ab - b2 =0

C) a2 + 2ac + b2=0

D) a2 + 2ac - b2=0

If (1-sinA)(1-sinB)(1-sinC)=(1+sinA)(1+sinB)(1+sinC) then each equal side is equal to


B) 1

C) sinAsinBsinC

D) cosAcosBcosC

If cosec + cot = 11/2, then tan is equal to


A) 21/22

35.

D) None

If sin and cos are roots of equation ax2 + bx + c =0 then the relation-ship between a, b, c is

A) zero
34.

C) -1

B) cosx

A) a2 + 2ab + b2 =0
33.

D) None

3(sinx - cosx)4 + 6(sinx + cosx)2 + 4 sin 6 x + cos 6 x is


A) sinx

32.

C) x

tan 2
1
If 0 < < /2, the value of the expression tan -1 + tan 1 - tan - seccosec is

A) 1
31.

D) /2

2sin
1 - cos + sin
then =
1 + cos + sin
1 + sin

A) sin
30.

C) /3

1 + tanAtanB

B) 15/16
2

+ tanA - tanB

C) 44/117
2 1/ 2

D) 117/44

is equal to

150

BASIC TRIGONOMETRY
A) tan2A + tan2B
36.

B) cos2Acos2B

C) secAsecB

D) tanAtanB

If tan2 tan2 + tan2 tan2 + tan2 tan2 + 2tan2 tan2 tan2 = 1 then the value of
sin2 + sin2 + sin2 is
A) 0

37.

4 3 m 2 1

B)

1
3 3

151

D)

C) k <

1
9

D) k > 1/3

C) 1/2

D) 1

C) tan c tan

D) tan c cot

B) 2

C) 1/ 2

D) 2

B) 8

C) 10

D) 12

The value of sin /14 sin 3 /14 sin 5 /14 sin 7 /4 sin 9 /14 sin11
1 /4 sin

If

sin +
cos -

B) 1/16
=

1- m
1+ m

C) 1/64

then tan - tan -


4

B) 1 + m

13
is equal to
14

D) 1/32
is equal to

C) 2m

D) 2 + m

If A and B are acute angles such that A+B and A-B satisfy the equation tan2 - 4tan +1=0 then
(A, B) be
B) ( /4, /6)

C) ( /6 /4)

D) ( /2, /6)

The equation cos4x - sin4x + cos2x + 2 + a = 0 will have at least one solution if
A) -2 2

47.

4 3 m 2 1

The number of integral values of k for which the equation 7cosx + 5sinx = 2k+1 has a solution is

A) ( /6, /6)
46.

If sec( - ), sec , sec ( + ) are in A.P, then cos sec /2 be

A) m
45.

3 3

B) cot c tan

A) 1
44.

C)

3 4 m 2 1

A+B
A -B
If cos A/ cos B = sin (c - )/ sin (c + ) then tan
tan
is
2
2

A) 4
43.

B) 3/4

A) 2
42.

If cos6 + sin6 + k sin22 = 1 then k is equal to

A) cot c cot
41.

4 3 m 2 1

B) k

A) 1/4
40.

D) 1/2

If A, B, C are acute positive angles such that A +B + C = and cot A cot B cot C = k then
A) k

39.

C) -1

If sin + cos = m then sin6 + cos6 is equal to


A)

38.

B) 1

B) -3 1

C) 2 1

D) 1 2

The number of points inside or on the circle x2+y2=4 satisfying tan4x + cot4x + 1 = 3sin2y

IIT- MATHS
A) one
48.

B) two

C) four

D) infinite

I f tan 1, tan 2, tan 3, tan 4 are roots of the equation x4-x3 sin 2 + x2 cos2 - x cos - sin

= 0 then tan 1 + 2 + 3 + 4 is equal to


A) sin
49.

B) cos

C) tan

D) cot

If x = , satisfies both the equations cos2x + acosx + b= 0 and sin2x + p sinx + q = 0 then
relation between a, b, p and q is
A) 1+b+a2 = p2-q-1

50.

B) 2

C) 3

B) cosec A

D) 4

C) sin A

D) tan A

(cosec - sin ) (sec - cos ) (tan + cot ) is equal to


A) 4 cos sin

53.

D) a2+b2 = p+q

sin 2 - cos + A tan - A


2

is equal to

sin - A cos 2 + A sin( - A)


2

A) cosA

52.

C) 2(b+q) = a2+p2-2

The number of solutions of the equation 1 + x2 + 2x sin (cos-1y) = 0 is


A) 1

51.

B) a2+b2 = p2+q2

B) 4 sec tan

C) 4 cosec cot

D) 1

If , and are variables subject to the relation 2tan + 3tan + 6 tan = 7, then the
minimum value of tan2 + tan2 + tan2 =
A) zero

54.

B) 1

d a a c
B) b d c b

B) a=0

If p =

a b

D) None

bc

C) a > 1

D) a > 0

a 2 cos 2 + b 2 sin 2 + a 2 sin 2 + b 2 cos 2 then the maximum value of P is

A) a - b

57.

C)

If 0 < A < /2 and sinA+cosA+tanA+cotA+secA+cosecA=7, and sinA and cosA are the roots of
the equation 4x2 - 3x + a =0, then
A) a=1

56.

D) 3

If a sin2 + b cos2 = C, bsin2 + acos2 = d and atan - b tan , then a2/b2 =


A) 1

55.

C) 2

B) b + a

C)

a b
2

D)

a b
2

4
4
sin 8 cos 8
+
=
If sin + cos = 1 , then
a3
b3
a
b
a+b

A)

1
8

a b

B)

a b

1
4

C)

a b

D)

1
a b
152

BASIC TRIGONOMETRY
58. If A and B are positive acute angles satisfying the equations 3cos2A + 2cos2B = 4 and
3sinA 2cosB
=
sinB
cosA

then A + 2B is equal to

A) /4
59.

a d c a
b c d b

B)

B) 2

d a c a
b c d b

C)

C)

B) -1/2

2
B) n n 1

B) - 5

B)

13
A1
16

B) tan + tan

If /2 < < , < <

A)

153

C) tan A + tan B = 1

D) tan A tan B = 0

a2
is equal to
b2
D)

b c b d
a c a d

D) 1/ 3

C) 1

D) -1

C) 2 n 1

D) 2n n 1

C) 5

D) - 1/5

C)

3
13
A
4
16

D)

3
A 1
4

If + = /2 and + = then tan equals


A) 2 (tan + tan )

69.

B) tan A tan B < 1

If A = cos2 +sin4 , then for all values of


A) 1 A 2

68.

D) 12

If 3cosx = 2cos (x - 2y) then tan (x - y) tan y is equal to


A) 1/5

67.

C) 6

The period of tan (x + 2x + 3x + ......... + nx) is

A) n n 1
66.

B) 9

The minimum value of cosx. cos (120-x) cos (120+x) is


A) -1/4

65.

D) - 13/18

If A + B = /4 then (tan A +1) (tan B +1) is equal to


A) 1

64.

C) 13/18

If a sin2 x + b cos2x = c ; bsin2y + a cos2y = d and a tanx = b tany then

b c d b
A) a d c a

63.

B) - 24/25

If A+B+C = (A, B, C > 0) and the angle C is obtuse then


A) tan A tan B > 1

62.

D) /2

The minimum value of 9tan2 + 4cot2 is


A) 13

61.

C) /6

If is the root of 25cos2 + 5cos - 12 = 0, /2 < < then sin 2 is equal to


A) 24/25

60.

B) /3

21
221

B)

C) tan + 2tan

D) 2tan + tan

3
15
12
; sin =
and tan =
the value of sin ( - ) is
2
17
5
21
221

C)

171
221

D)

171
221

70.

IIT- MATHS
The number of values of x in the interval [0, 5 ] satisfying the equation 3sin x-7sinx+2=0
2

A) 0
71.

B) 5

76.

C) [ /2, ] U [3 /2, 2 ]

D) x [ , 2 ]

If

A
1- sinA = sinA / 2 - cosA / 2 then 2 - 4

D) 1

B) II, III

could lie in quadrant


C) III, IV

D) I, IV

B) 8

C) 10

D) 12

If cosec - sin = m; sec - cos = n then (m2n) 2/3 + (mn2) 2/3 =

If < /8 the value of

B) 1

C) 2

D) - 1

2 + 2 + 2 + 2cos4

B) 2cos 2

C) 2cos /2

D) 2sin /2

If sinx + sin2x = 1, then the value of cos2x + cos4x is


B) 2

C) 1.5

D) None

6 sin 6 + cos 6 - 9 sin 4 + cos 4 + 15 is equal to


B) 10

C) 12

D) - 12

If sin + sin2 +sin3 =1, then cos6 - 4cos4 +8cos2 =


A) 3

83.

D) 2

Total number of roots of the equation 3cosx = |sinx| belongs to [-2 , 2 ] are

A) -10
82.

C) - 3

B) (0, /2

A) 1
81.

C) -1

A) x , , 2
2 2

A) 2 cos
80.

D)

If [cosx] + [sinx + 1] = 0, then value of x satisfying f(x) where x 0, 2

A) 0
79.

C) /3

B) 5

A) 6
78.

D) 2cos36

The value of y for which the equation 4sinx+3cosx=y2 - 6y + 14 has a real solution is/are

A) I, II
77.

B) /2

B) 1

A) 3
75.

C) 2cos18

If sin x + sin2x = 1 then the value of cos12 x + 3cos10 x + 3cos8x + cos6x - 1 is equal to
A) 0

74.

B) 2sin18

If A and B are acute positive angles satisfying the equation 3sin2A + 2sin2B = 1 and 3sin2A 2sin2B = 0 then A +2B =
A) /4

73.

D) 10

If cos A = tan B, cos B = tan C, cos C = tan A, then cos2 A is equal to


A) sin 18

72.

C) 6

B) -3

C) 4

D) -4

If sinx+cosx = a, then |sinx-cosx| =


154

BASIC TRIGONOMETRY
A)
84.

B)

2 a2

C)

2a

D)

2 a2

2a

If x = sin .cos , y = sin .sin , z = cos then x2 + y2 + z2 =


A) sin2

B) sin2

D) 2

C) cos2
4xy

85.

For all real values of x and y, the equation sec = x + y 2 is possible when

A) x = y

86.

B) x y

sin + cos = m, then sin6 +cos6 =


A) m R

87.

B) 2 /7

D) -2 /7

B) x ? 4

C) x ? 6

D) None

B) a - b

C) a/b

D) 2 ab

If x = a cos3, y = bsin3 , then

a
C)
x

2/3

2/3

If tan =

y

b

b

y

2/3

x
B)
b

2/3

2/3

b
D)
x

y

a

2/3

2/3

a

y

1
2/3

sin - cos
, then
sin + cos

A) sin -cos = 2 sin


C) sin = 2 (sin +cos )
92.

C) - /7

The minimum value of a2tan2? + b2cot2 is

x
A)
a

91.

D) None

If x = sin2 + cos2 +tan2 +cosec2 +sec2 +cot2 , 0, then

A) a + b
90.

C) m

If f(x) = 3(sinx-cosx)4 + 6(sinx+cosx)2 + 4(sin6x+cos6x) and g(13) = /7 then (gof)x =

A) x =1
89.

D) x = y = 0

2
1
4 - 3 m 2 -1 if

B) m R

A) /7
88.

C) x = y 0

B) sin +cos
D) None

2 cos

If tan equals to the integral solution of the inequality 4x2 - 16x + 15 < 0 and cos equals to
the slope of the bisector of the first and quadrant, then sin ( + ) sin ( - ) is equal to
A) 3/5

93.

D) 4/5

B) a2 + (b - c )2

C) c2 + (a - b )2

D) b2 + (a +c)2

For what value of lying between 0 and is the inequality sin cos3 > sin3 cos valid
A) (0, /4)

155

C) 2 / 5

1
If maximum value of |a sin2 + b sin cos +c cos2 (a+c)| is k/2 then k2 is equal to
2

A) b2 + (a -c)2
94.

B) - 3/5

B) (0, /2)

C) ( /4, /2)

D) (- /4 /4)

IIT- MATHS
95.

The values of a for which the equation cos2x + a sin x = 2a - 7 has a solution
A) 2 a 6

96.

B) 1 a 5

B) 7/16

C) (

B) ( / 2, 2 / 3]

D) 4/15

2
,5 / 6]
3

D) (

5
, ]
6

If ABCD is a cyclic quadrilateral such that 12 tan A - 5 = 0 and 5cos B + 3 = 0, then the quadratic
equation whose root are cosC, tanD is
A) 39x2 - 16x - 48 = 0
C) 39x2 - 88x + 48 = 0

99.

C) 1/16

Given both and are acute angles and sin = 1/2 cos = 1/3 then the value of +
belongs to
A) ( / 3 / 2]

98.

D) 1 a 6

If sinx cos y = 1/4 and 3 tanx = 4 tany then sin(x+y)


A) 5/16

97.

C) a 5

B) 39x2 + 88x + 48 = 0
D) 39x2 + 16x + 48 = 01

If sinA, cos A, tan A are in G.P. then cot6A - cot2A is equal to


A) 1

B) -1

C) 1/2

D) -1/2

C) -1/2

D) 0

100. Minimum value of 4x2 - 4x |sin | - cos2 is


A) -2

B) -1

101. The number of solution of equation e cos 2 x - e -cos2 x + 4 = 0 in [0, 2 ]


A) 1

B) 2

C) 3

D) none of these

102. sinx + siny = y2 - y + a will have no solution in x and y if a belongs to


A)

0, 3

B) 3, 0

C)

D) 2 ,
4

, 3

103. In an isosceles right angled triangle, a straight line drawn from the mid point of one equal sides to
the opposite angle. It divides the angle into two parts and ( /4 - ), Then tan and tan ( /4
- ), then tan and tan ( /4- ) are equal to
A)

1 1
,
3 4

B)

1 1
,
4 5

C)

1 1
,
5 6

D)

1 1
,
2 3

104. The equation of sinx (sinx + cosx) = k has real solutions then
A) 0 k

1 2
2

C) 0 k 2 3

B) 2 3 k 2 3

D)

1 2
1 2
k
2
2

105. cosec + + x cos cot + = sin + then x =


2

156

BASIC TRIGONOMETRY
A) cot
B) sin

C) tan

D) cos

4 3

- + sin 4 3 + - 2 sin 6 + + sin 6 5 - =


106. 3 sin
2

A) 0

B) 1

C) 3

D) 2

107. (sin + cosec )2 + (cos + sec )2 = k + tan2 + cot2 then k =


A) 9

B) 7

C) 5

D) 3

108. If x cos + y sin = x cos + y sin = 2a , 0 < , < then


2

A) cos + cos =

C) sin + sin =

4ax
2
x y2

B) cos cos =

4ay
2
x y2

D) sin sin =

109. If x = acos2 sin , y = a sin2 cos , then


A) a sin

B) a2sin

B) -2cos

4a 2 x 2
x2 y 2

x 2 y2

C) a2sin2

3
110. If < <
, then the expression is equal to
2

A) +2sin

+ y2

4a 2 y 2
x2 y 2

1 - cos
1 + cos
+
1+ cos
1- cos

C) +2sec

111. If cosecA = cosecBcosecC + cotBcotC, then cosecB =


A) cosecAcosecC+cotAcotC
C) cosecAcosecC ? cotAcotC

157

D) a2(sin2 +cos2 )

B) cosecAcosecC - cotAcotC
D) None

D) -2cosec

IIT- MATHS

SECTION - B
SINGLE ANSWER TYPE QUESTIONS
1.

For any real x one has


A) cos (cos x) > sin (sin x)
C) cos (sin (cos x)) > sin (cos (sin x))

B) cos (sin x) > sin (cos x)


D) cos (cos (cosx)) > sin (sin (sinx))
7

2.

I f sec 20 + sec 40 + sec 80 = a and

tan
k =1

= b then the value of a-b can be expressed as


16

A) sec2 - tan2
B) cos2 - sin2
C) 3(sin2 + cos4 ) - 2(sin6 + cos6 ) D) none of these
3.

If 3 sin = sin (2 + ) then


A) [cot + cot ( + )] [cot - 3 cot (2 )] = 6
B) sin = cos ( + ) sin
C) 2 sin = sin ( + ) co s
D) tan ( + ) = 2 sin

4.

5.

Let Pn (u) be a polynomial in u of degree n. Then, for every positive integer n, sin 2nx is expressible is
A) P2n (sinx)

B) P2n (cos x)

C) cos x P2n-1 (sinx)

D) sin x P2n-1 (cos x)

If cos =

3
5
and cos =
, then
5
13

A) cos ( + ) =

33
65

1
C) sin2

2 65

6.

56
65

D) cos ( - ) =

63
65

The equation sin6x + cos6x = a2 has real solutions if


A) a (-1, 1)

7.

B) sin ( + ) =

B) a 1,
2

1 1
C) a ,
2 2

1
D) a ,1
2

If tan and tan are the roots of the equation x2 + px + q = 0 (p 0), then
A) sin2 ( + ) + p sin ( + ) cos ( + ) + q cos2 ( + ) = q
B) tan ( + ) = p/q -1
C) cos ( + ) = 1-q
D) sin ( + )=-p

8.

If sin + sin = a and cos + cos = b, then


158

BASIC TRIGONOMETRY
1

A) cos

2
2


B) cos

b2

4 a2 b2

2 2
C) tan 2

a b
9.

11.

B) (a2+b2 ) cos = 2ab


D) sin = 2.375

(1+sec ) (1+sec2 ) (1+sec 4 ) .... (1+sec2n ). Then


2

Let fn ( ) = tan

A) f2 =1
16


B) f3 =1
32


C) f4 =1
64


D) f5
=1
128

If (secA + tanA) (secB + tanB) (secC + tanC) = (secA-tanA) (secB-tanB) (secC-tanC)


B)-1

C) 0

For 0 < /2 if
A) xyz = xz + y
If tanx =

D) none of these

2n
x = cos , y =
n =0

13.

b2

Which of the following statements are possible a, b, m and n being non-zero real numbers ?

A) 1
12.

a2 b2 2
D) cos
2

A) 4 sin2 = 5
C) m 2 n 2 cos ec m 2 n 2
10.

sin 2n

z=

n =0

B) xyz = xy + z

cos

2n

sin 2n , then

n =0

C) xyz = x+y+z

D) xyz = yz+x

C)y-z = a-c

D) y-z = (a-c)2 + 4b2

2b
, (a c)
a -c

y = a cos2x + 2b sinx cos x + c sin2x


z = a sin2x - 2b sinx cosx + c cos2x, then
A) y = z

B) y + z = a+c
n

14.

cosA + cosB sinA + sinB

sinA - sinB cosA - cosB

AB
A) 2tann

15.

AB
B) 2cotn

C) 0

D) none of these

B) cot 760

C) tan 460

D) cot 440

In a triangle tanA + tanB + tanC = 6 and tanA tanB = 2, then the values of tanA.tanB and tanC are
A) 1, 2, 3

159

(n, even or odd) =

3 + cot76 0 cot16 0
=
cot76 0 + cot16 0
A) tan 160

16.

B) 2, 1, 3

C) 1, 2, 0

D) none of these

IIT- MATHS
17.

If cot + tan = x and sec - cos = y, then


1
x
2 2/3
2 2/3
C) (x y) - (xy ) = 1

A) sinq cosq =

18.

19.

If

D) (x2y)1/3 + (xy2)1/3 = 1

x cosA
=
where A B then
y cosB

A B x tan A y tan B
A) tan

x y
2

A B x tan A y tan B
B) tan

x y
2

sin A B y sin A x sin B


C) sin A B y sin A x sin B

D) x cosA + y cosB = 0

If tan =

sin - cos
then
sin + cos

A) sin -cos =
C) cos2 = sin2
20.

B) sin tan = y

2 sin

B) sin + cos = 2 cos


D) sin2 + cos2 = 0

Let 0 /2 and x = X cos + Y sin , y = X sin -Y cos such that x2 + 4xy + y2 = aX2 + bY2,
where a, b are constants. Then
A) a = -1, b = 3

B) = /4

C) x = 3, b = -1

D) = /3

160

BASIC TRIGONOMETRY

SECTION - A
SINGLE ANSWER TYPE QUESTIONS

1.

2.

3.

4.

5.

6.

7.

8.

9.

10.

11.

16.

17. 18. 19.

20. 21. 22. 23

24. 25.

26.

12. 13.

27. 28.

14. 15.

29. 30.

B,C

31

32

33

34

35

36

37

38

39

40

41

42 43

44

45

46

47

48

49

50

51

52

53

54

55

56

57 58

59

60

61

62

63

64

65

66

67

68

69

70

71

72 73

74

75

161

IIT- MATHS

76

77

78

79

80

81

82

83

84

85

86

87 88

89

90

91

92

93

94

95

96

97

98

99 100 101 102 103 104 105

A,B,C D

106 107 108 109

B A,B,C A

110 111

SECTION - B
SINGLE ANSWER TYPE QUESTIONS
1.

2.

3.

AB

AD ABC CD BCD BD AB ACD BD

16

17

18

4.

5.

19

20

AB ABC ABC AB
CD

BC

6.

7.

8.

9.

10.

11.

12. 13.

14. 15.

A B AB
CD

BC BC

BC CD

162

BASIC TRIGONOMETRY

163

IIT- MATHS

TRIGONOMETRIC
EQUATIONS

164

TRIGONOMETRY EQUATIONS
An equation involving one or more trigonometrical ratios of unknown angle is called trigonometric equation e.g. cos2x 4 sinx = 1
It is to be noted that a trigonometrical identity is satisfied for every value of the unknown angle
whereas, trigonometric equation is satisfied only for some values (finite or infinite in number) of unknown angle. e.g. sin2x + cos2x = 1 is a trigonometrical identity as it is satisfied for every value of x R.

SOLUTION OF A TRIGONOMETRIC EQUATION


A value of the unknown angle which satisfies the given equation is called a solution of the equation
e.g. / 6 is a solution of sin =

1
.
2

GENERAL SOLUTION
Since trigonometrical functions are periodic functions, solutions of trigonometric equations can
be generalized with the help of the periodicity of the trigonometrical functions. The solution consisting of
all possible solutions of a trigonometric equation is called its general solution.
We use the following formulae for solving the trigonometric equations: ( n I)
sin = 0 = n,
cos = 0 = (2n + 1)

,
2

tan = 0 = n,
sin = sin = n + ( 1)n
cos = cos = 2n
tan = tan = n +
sin2 = sin2 or cos2 = cos2 or tan2 = tan2 = n ,
sin = 1 = (4n + 1)
sin = 1 = (4n 1)

cos = 1 = 2n
cos = 1 = (2n + 1)
sin = sin and cos = cos = 2n +

Note:
Everywhere in this chapter n is taken as an integer, if not stated otherwise.
The general solution should be given unless the solution is required in a specified nterval or
range.

SOME IMPORTANT POINTS TO REMEMBER


While solving a trigonometric equation, squaring the equation at any step should be
avoided as far as possible. If squaring is necessary, check the solution for extraneous
values.
165

IIT- MATHS
Never cancel terms containing unknown terms on the two sides, which are in product. It may
cause loss of genuine solution.
The answer should not contain such values of angles, which make any of the terms
undefined.
Domain should not be changed. If it is changed, necessary corrections must be
incorporated.
Check that the denominator is not zero at any stage while solving equations.
Some times you may find that your answers differ from those in the package in their
notations. This may be due to the different methods of solving the same problem.
Whenever you come across such situation, you must check their authenticity. This will
ensure that your answer is correct.
While solving trigonometric equations you may get same set of solution repeated in your
answer. It is necessary for you to exclude these repetitions, e.g. n +
of

, ( n I) forms a part
2

k
, k I the second part of the second set of solution (you can check by putting
5 10

k = 5 m + 2 (mI). Hence the final answer is

k
,k I .
5 10

Some times the two solution set consist partly of common values. In all such cases the
common part must be presented only once.
Now we present some illustrations for solving the different forms of trigonometric
equations.
Which will highlight the importance of above mentioned points.

SOLVING SIMULTANEOUS EQUATIONS


Here we will discuss problems related to the solution of two equations satisfied simultaneously.
We may divide the problems in two categories.
(i)

Two equations in one unknown

(ii)

Two equations in two unknowns.

TRIGNOMETRIC INEQUATIONS

/6

5/6

y = 1/2
x

166

TRIGONOMETRY EQUATIONS
From, the graph of y = sin x,
it is obvous that, between 0 and 2p
sinx > 1/2 for

5
x
.
6
6

Hence sin x > 1/ 2

2np + p/6 < x < 2np+ 5p/6

2n , 2 n
The required solution set = n
I
6
6

PROBLEMS BASED ON BOUNDARY CONDITIONS


If the problem involves only one equation consisting of more than one variable or equation involves variable of different natures then the boundary conditions of trigonometric functions is generally
used. It must be noted that
|sinx| 1 ; | cosx | 1; |sec x| 1; | cosec x | 1; |tan x| 0; |cot x| 0

167

IIT- MATHS

WORKED OUT ILLUSTRATIONS


ILLUSTRATION : 01
Number of solutions of the equation tan x sec x 2cos x lying in the interval [0. 2 ] is
(a) 0

(b) 1

(c) 2

(d) 3

Ans : (A)
Solution :
The given equation can be written as

sin x 1
2 cos x
cos x

== sin x 1 2cos 2 x cos x 0

==> 2sin 2 x sin x 1 0

==>

2sin x 1 sin x 1 0

==> x

==> sin x

sin x 1 0

as cosx 0

5
,
in 0, 2 so that required number of solutions is 2.
6 6

ILLUSTRATION : 02
2
If 6 cos 2 2 cos

(a)

2 sin 2 0, , then =
2

(b)

3
, cos 1
3
5

1 3
(c) cos
5

(d)

3
, cos 1
3
5

Ans : (D)
Solution :
The given equation can be written as
==> 10 cos 2 cos 3 0
==> 5cos 3 2cos 1 0
==> cos

1
3
or cos
2
5

==>

1 3
or cos as
3
5

==>

3
, cos 1
3
5

ILLUSTRATION : 03
The set of values of x for which

tan 3x tan 2 x
1 is
1 tan 3x tan 2 x
168

TRIGONOMETRY EQUATIONS

(b)
4

(a)

(c) n / n 1, 2,3,.....
4

(d) 2n / n 1, 2,3......
4

Ans : (C)
Solution :
The given equation can be written as tan 3x 2 x 1 ==> tanx = 1 ==> x n

But for these values of x, tan2x is not defined so the given equation has no solutions.
ILLUSTRATION : 04
The number of all possible triplets such than a1 a2 cos 2 x a3 sin 2 x 0 for all is
(a) 0

(b) 1

(c) 3

(d) infinite

Ans : (D)
Solution :
The given equation can be written as a1 a2 cos 2 x

a3
1 cos 2 x 0
2

a3
a3

==> a1 a2 cos 2 x 0
2
2

which is zero for all values of x.


If a1

a3
a2
2

k
k
or a1 , a2 , a3 k for any
2
2

Hence the required number of triplets is infinite.


ILLUSTRATION : 05
In a triangle ABC, the angle A is greater than the angle B. If the values of the angles A and B satisfy
the equation 3sinx - 4sin3x - k =0, 0 < k < 1, then the measure of angle C is
(a)

(b)

(c)

2
3

(d)

5
6

Ans :(C)
Solution :
The given equation can be written as sin 3 x k , 0 k 1
Since A and B satisfy this equation
0 3 A, 3B < as 0 < k < 1

169

IIT- MATHS
Also sin 3 A k sin 3B ==> sin 3 A sin 3B 0
==> 2cos

3 A B
3 A B
sin
0
2
2

==> either cos

But sin

3 A B
3 A B
0 or sin
0
2
2

3 A B
3 A B
0 as A > B and 0 < 3A, 3B < so cos
0
2
2

==> cos

3
C 0
2

==> sin

3C
2
0 ==> C
.
2
3

ILLUSTRATION : 06
The equation sinx + cosx =1 has a solution in the open interval

(a) 0,
2


(b) ,
2

3
(c) ,
2

(d) None of these

Solution :


sin x cos x 1 ==> sin x 4 2 sin 4


n
n

==> x n 1 ==> x n 1 , n I
4
4
4 4


==> x 2n or x 2n
2

so that x does not belong to the intervals given by (a), (b), or (c) for any value of x.
ILLUSTRATION : 07

3
7
1
1
sin

The principal value of


2 cos cos 6 is

(a)

5
6

(b)

(C)

3
2

(d) None of these

Ans : (B)
Solution :

3
3

7
1
1
sin 1
sin
and cos cos

3
6

2
2
170

TRIGONOMETRY EQUATIONS
5
5

1
1
= cos cos 2
cos cos
6
6


3
7
1
sin 1
cos cos
6

so that

a

3 6
2

ILLUSTRATION : 08
If sin 4 x cos 4 y 2 4sin x cos y, and 0 x, y
(a) -2

(b) 0

(c) 2

then sinx + cosy is equal to

(d) none of these

Ans : (C)
Solution :
The given equation can be written as
sin 4 x cos 4 y 2 4sin x cos y 0
2

==> sin 2 x 1 cos 2 y 1 2sin 2 x 2 cos2 y 4sin x cos y 0


==> sin 2 x 1 cos 2 y 1 2(sin x cos y )2 0

which is true if sin 2 x 1 , cos 2 y 1 and sin x cos y as 0 x, y we get sinx=cosy=1


2
==> sin x cos y 2
ILLUSTRATION : 09
n

1 sin x ....... 1 sin n x ...... 1 cos 2 x


The general solution of the equation

1 sin x ....... sin n x ........


1 cos 2 x
(a)

(c) 1


n
3

n 1

n
(b) 1 n
6


n
6

(d)

n 1


n , n I
3

Ans :
Solution :
n

1 sin x ....... 1 sin n x ...... 1 cos 2 x


The equation

1 sin x ....... sin n x ........


1 cos 2 x
==>

171

1
1-sinx 2sin 2 x
X

1 sin x
1
2 cos 2 x

is

IIT- MATHS
==> cos 2 x cos 2 x sin x sin 2 x sin 3 x

(sinx +1 0)

==> cos 2 x sin 2 x sin x cos 2 x sin 2 x


==> 2sin 2 x sin x 1 0
==> sin x

1
1 1 8 1 3

==> or sin x
2
4
4

ILLUSTRATION : 10
5
5
The number of solutions of the equation sin x cos x

(a) 0

(b) 1

(c) infinite

1
1

sin x cos x is
cos x sin x

(d) none of these

Ans :
Solution :
The given equation can be written as

sin 5 x cos5 x
sin x cos x
sin
x
cos
x
sin x cos x
==>
sin x cos x 1
sin x cos x

1
4
3
2
2
3
4
==> sin 2 x sin x sin x cos x sin x cos x sin x cos x cos x =1
2
2
2
2
2
2
2
2
2
2
==> sin 2 x sin x cos x 2sin x cos x sin x cos x sin x cos x sin x cos x 2

==> sin 2 x 1 sin 2 x cos 2 x sin x cos x 2


1
1 2

==> sin 2 x 1 sin 2 x sin 2 x 2


2
4

==> sin 3 2 x 2 sin 2 2 x 4 sin 2 x 8 0


2

==> sin 2 x 2 sin 2 x 2 0


==> sin 2 x 2 , which is not possible for any .
ILLUSTRATION : 11
cos 3
1

if
2 cos 2 1 2

(a) n

(b) 2n

(c) 2n

(d) 2n

Ans :(B)

172

TRIGONOMETRY EQUATIONS
Solution :
4 cos3 3cos 1
The given equality can be written as 2 2 cos 2 1 1 2

4cos 3 cos 1
2

==>

4 cos 2 3

==> cos

1
2

==> 2n

3
, as for this value of L.H.S. of the given equation is not defined.
cos
2

173

IIT- MATHS

SECTION - A
SINGLE ANSWER TYPE QUESTIONS
1.

sin2x - 2cos x +

1
= 0 then x equal to
4

A) 2n /3
2.

B) 2n /4

The set of values of x for which

C) 2n /6

tan3x - tan2x
= 1 is
1 + tan3xtan2x

A)
C) {n /4 n = 1,2......}
3.

B) /4
D) 2n /4 ; n = 1, 2, 3 .......

The most general value of which satisfies sin = -1/2 and tan = 1/ 3
A) 2n /6

4.

B) 2n + 11
1 /6

C) 2n + 7 /6

D) ? = n + 3 /6

If 1 + cos + cos2 +....... = 2 - 2 then (0 < < ) is


A) 3 /4

B) / 4

B) x = 6 (n - 1)

D) x = 5 (n+

1
)
2

C) 13/18

D) -13/18

If |K| = 5 and 0 3 6 0 , then the number of different solutions of 3cos + 4sin = K is

sin 4 x + cos 4 x =

A) x =

B) two

C) one

D) infinite

7
sinxcosx then x equal to
2

n
n
1
2
2

C) x = n 1
10.

1
)
2 2

C) x = 5 (n +

B) -24/25

A) zero
9.

D) / 8

If is the root of 25cos2 + 5cos - 12 = 0 /2 < <


A) 24/25

8.

C) / 6

6x
x
If sin = 0 and cos = 0 then
5
5

A) x = (n - 5)
7.

D) 2n + /4

B) = n + (-1)n (3 /10)

C) = 2n /6

6.

is

Solution of the equation 4cos2 sin - 2sin2 = 3sin is


3
A) n + (-1)n

10

5.

D) n /3

B) x = n 1

24

D) x = n 1

6
n

12

If tan x + tan 4x + tan 7x = tan x. tan 4x tan 7x then x is equal to


A) n /6

B) n /12

C) /12

D) n /2
174

TRIGONOMETRY EQUATIONS
11.

12.

If sin 4 = cos - cos 7


A)

n
n
1 / 18
3

B)

n
n
1 / 9
3

C)

n
n
1 / 6
3

D)

n
n
1 / 12
2

If tan 7 = cot 5 then general solution for is


A) 2n /24

13.

B) (2n-1) /24

B) 5

C) 8

D) 9

The most general solutions of the equation secx - 1 = ( 2 1) tanx are given by
A) n /8

15.

D) n + /12

The number of distinct solutions of sin 5 cos 3 = sin9 . cos7 in [0,2 ] is


A) 4

14.

C) n + /24

B) 2n , 2n + /4

D) n /4

C) 2n

The most general values of ? satisfying the equation (1+2sin )2 +

3sin -1

= 0 are given

by
B) n + (-1)n

A) n /6

16.

sinx.sin(600-x) sin(600+x) =
A) x = n + (-(A)n
C) x n 1

17.

D) 2n + 11
1 /6

B) x =
D) x

n
n
1
3
18
n
n
1
3
9

cos3
1
= , if
2cos2 -1 2

B) = 2n /3

C) =2n /6

D) =2n /6

General solution of the equation 1+sin2x = (sin3x-cos3x)2 is/are given by



A) n
2

19.

C) 2n + 7 /6

1
, n I, then
8

A) = n + /3
18.

7
6

B) n , n C) n , n
4
4
4

D) None

If [x] denotes the greatest integer less than or equal to x and let f(x) = sinx +

3 cosx, then the


most general solution of f(x) = f are
10
A) n /4
20.

175

B) 2n /3

C) n + (-1)n /6 - /3 D) None

The smallest positive value of x for which tanxtannx = 1(n N) is

IIT- MATHS

A) 2 n 1
21.

B)

2
n 1

C)

3
2 n 1

D) None

If 5 cos2 + 2cos2 /2 +1 = 0 - then is equal to


A) = /4

B) = /3

C) = /2

D) /6

22.

If the system of equations (sin3 ) x - y + z = 0 (cos2 ) x + 4y +3z = 0 and 2x + 7y + 7z = 0 have


non - trivial solution, then is equal to

A) = n
B) = (2n+1)
C) 2n 1
D) 4n 1 / 2
2
2

23.

The equation 3sin2x+2cos2 x + 31-sin2x+2sin 2x = 28 is satisfied for the values of x given by


A) tan x = 1

24.

B) cosx = 0

B) 1

If sin =

B) x =

n
2

C) tan x = 1 / 4

D) x n / 4

B) /3, /2

C) /6,/4

D) /4, /3

B) {x|x = 2np 2p/3 n e z}


D) [p/3, p/2]

B) [0, 5/6]

C) [5/6, 2]

D) [/6, 5/6]

The equation cos4x (a+2) cos2x (a+3)=0 possesses a solution if


B) a < -2

C) 3 a 2

D)

If sin8x + cos8y + 2 = y sin2x cos2y and 0 x, y /2 then sinx + cosy is equal to


A) 2

32.

D) 2

If sin 4sin2x 8sinx + 3 0 0 / 2 then the solution set for x is

A) a >-3
31.

C) infinite

If max {5sin + 3sin () = 7 then the set of possible values of is

A) [0, /6]
30.

D) {/4, /3}

3 , + 4sin = 2 ( 3 1) 0 / 2 then is equal to

A) {x|x = 2np p/3 n e z }


C) [x/3, 2p/3 ]
29.

C) {/3}

2sin 2 cos 2 x = 1-cos sin 2x if


2

A) /6, /3
28.

1
is
2

x
1
2
2
.sin
x

The number of solutions of 2cos 2


x 2 0 x / 2 is

A) x = 2n 1 / 2
27.

A) 0
26.

B) {/3, /4}

D) tan x =

0 <q < p solution for the equation cos2 = ( 2 1) COS

A) {/4, /6}
25.

C) tan x =

B) 0

C) 2

D) None

{2, e, , 2 4 7} are given by


The most general values of x for which 3 sin x cos x min
R

176

TRIGONOMETRY EQUATIONS
A) 2n
B) 2n + 2/3
33.

If

2 cos 2 X 1 2 sin 2 X 1 2 2 then x =

A) (2n+1)
34.

B) (2n-1)

38.

B) 2n /4

The general solution of the equation

42.

D) n + (-1)n /4

C) 2n + 7/4

C) n - /4

3 1 sin

D) 2n /4

3 1 cos 2

n
A) n 1

/ 12
4

B) 2n / 4 / 12

n
C) n 1

/ 12
4

D) 2n / 4 / 12

sin x

cos x
The number of distinct real roots of
cos x

cos x
sin x

cos x

cos x 0
in the interval - /4 x / 4 is
sin x

cos x

B) 2

C) 1

D) 3

The smallest positive root of the equation tan x x = 0 lies in


B) (/2, )

C) (, 3/2)

D) (3/2, 2)

cos3x + sin 2 x
= -2 then x is equal to
6

A)

41.

5
, cos-1 (-3/2} D) {/3, /6}
3

If sin cot cos tan , then is equal to


2

A) (0,/2)
40.

C) {/3,

B) n + (-1)n 7/4

A) 0
39.

D) All

The most general value of which satisfies both the equations tan = -1 and cos = 1 / 2 will be

A) n +/4
37.

C) (2n1)

B) {/3, 5/3}

A) n +7 /4
36.

The solution set of (2cosx 1) (3+2 cosx) = 0 in the interval 0 x 2 is


A) {/3}

35.

C) n + (-1)n/4 +/6 D) None

6k 1
3

B)

6k 1
3

The general solution of sin2 sec +

C)

6k 1
3

D)

6k 1
2

3 tan = 0 is

A) = n + (-1)n+1 /3, = n n z

B) = n n z

C) = n + (-1)n+1 /3 n z

D) =

n
nz
2

The value of x between 0 and 2 which satisfy the equation sin x 8 cos 2 x 1 are in A.P. with
common difference
A) /4

177

B) /8

C) 3/8

D) 5/8

IIT- MATHS
43.

The inequality 2sin2x 3sinx + 1 > 0 holds for all values of x


5
,
6

B) (0, /6)

5
,
6

2 4

D) (0, /3) ,

A) (0, /3)

C) (0, /2) ,
44.

1 + sin x + sin2x +.. = 4 + 2 3 , where x e [0,]


A) /3 or 2 /3

45.

D) n - /3

C) 8

D) 10

B) 2n + /3 n z

C) 2n n z

D) n + (-1)n /3 n z

B) 4

C) 2

D) 1

B) 1

B) n + /4

C) 2

D) 3

sin x i cos x
is purely imaginary are given by
1 i
C) n

D) 2n + /2

The equation (cosp-1)x2 + (cosp)x+sinp=0 where x is a variable has real roots if p lies in the
interval
A) (0, 2)

52.

B) 6

The values of x [-2, 2] such that


A) n - /4

51.

C) 2n- - /3

The number of values of x for which sin2x + cos4x = 2 is


A) 0

50.

B) 2n + /3

The number of solutions of |cosx|= sinx 4 is


A) 8

49.

D) /2

= 2 are
The most general values of satisfying tan + tan
4

A) n /3 n z
48.

C) /4

The number of values of x in [0,5] satisfying the equation 3cos2x 10cosx + 7 = 0 is


A) 5

47.

B) /6

If sec + tan = 2 3 then general solution for is


A) n + /3

46.

B) (-, 0)

C) (-/2, /(B)

D) (0, )

n
If 2tan2x 3secx is equal to 0 for exactly 7 distinct values of x 0,
, n N then the greatest
2
value of n
A) 4

53.

B) 10

C) 13

D) 15

If [x] denotes the greatest integer less than or equal to x and sinx = [1+sinx]+[1-cosx] has
A) no solution in [-/2, /2]

B) no solution in [/2, ]

C) no solution in [, 3/2]

D) no solution for x

178

TRIGONOMETRY EQUATIONS
54.

If cos2 =

2 1 cos
then =
2

A) 2n + /4
55.

B) 2n /3

B)

n
/8
2

n
C) 1

If cos6 + cos4 + cos2 + 1 = 0 0


A) {/6, /4, /2}

57.

D) None

The general solution of sinx - 3sin2x + sin3x = cosx - 3cos2x + cos3x is


A) np + p/8

56.

C) both

B) {/6, /4}

3
2

n
/8
2

1
D) 2n cos

then q is equal to
2
D) {/3, /6}

C) {/4}

The value of lying between = 0 and = /2 and satisfying the equation


1 cos 2
sin 2
4 sin 4

2
2
4 sin 4 0
cos 1 sin
cos 2
sin 2 1 4 sin 4

A) 10/24
58.

B) 7/24

The value of a for which the equation a2 2a + sec2 { (a+ x) } = 0 has solution is
A) a = 2

59.

B) a = 1

B) 1/3

B) 2 a 2

D) 1/ 2 a 1/ 2

C) 110

1
x e [0, 6p] is
4

D) 15/3

If max {5 sin+ 3sin(-a)}=7, then the set of possible values of is .

C) ,
3 3

B) x : x 2n

, n I
3

D) None

1
1
The number of solutions of the equation sin5x cos5x = cos x sin x (sinx cosx) is

A) zero

179

D)

C) 1 a 1

B) 30

A) x : x 2n , n I

63.

C) 2/3

The sum of all the solutions of the equation cosx. cos ( / 3 x) cos ( / 3 x )
A) 15

62.

D) 1

The equations sin4x 2cos2x + a2 = 0 is solvable if


A) 3 a 3

61.

C) a =

2
19
3 sinpx + cos px = x2 - 3 x + 9 , then x is equal to

If

A) 1/3
60.

D) /24

C) 5/24

B) 5

C)

D) None

IIT- MATHS
64.

If the system of equations (sin3)x y +z=0, (cos2)x +4y+3z=0, 2x+7y+7z=0 have non-trivial
solutions, then =

A) n, n+ (-1)n
6
C) both

B) n a, n + (-1)n
3
D) None

180

TRIGONOMETRY EQUATIONS

SECTION - B
MULTIPLE ANSWER TYPE QUESTIONS
1.

2.

2 sin x cos2x = sinx if


A) x = n + /6 (n l)

B) x = n - /6 (n l)

C) x = n (n l)

D) x = n + /2 (n l)

X
X
cos 2 X 2sin sin 2 X cos2 X sin 2 X has a root for which
2
2

The equation 2 sin


A) sin 2x=1

3.

B) sin2x=-1

B) sin (x - p/4) =

2
1

C) cos (x + p/4) =

5.

D) cos (x-p/4) =

C) two values of x and two values of

D) two pairs of values of (x, )

The equation sinx = [1+sinx] + [1-cosx] has (where [x] is the greatest integer less than or equal
to x)

3
2

B) no solution in ,
2
D) no solution for x

Solutions of the equation sin7x + cos2x = -2 are


A) x =

2 k 3

, n, k I
7
14

C) x = n + /2, n I

,nI
4
D) none of these

B) x = np +

The solutions of the system of equation sinx siny =

3 /4, cosx cosy =

A) x1 =


2n k
3 2

B) y1 =


k 2n
6 2

C) x2 =


2n k
6 2

D) y2 =


k 2n
3 2

3 /4 are .........

2sin2x + sin22x = 2, x, then x =


A) /2

181

B) one value of x and two values of

C) no solution in ,

8.

A) no value of x and

7.

3 cos = 6x - x2-11, 4, holds for

sin +

A) no solution in ,
2 2

6.

D) cos2x = -1/2

sinx + cosx = 1 + sinx cosx, if


A) sin (x+p/4) =

4.

C) cosx = 1/2

B) /4

C) 3/4

D) none of these

IIT- MATHS

9.

1 tan x

If 1 tan x = tany and x-y , then x, y are respectively


6

A)
10.

5
,
24 24

If cosx =
A) p

11.

B) -

7 11
,
24
24

C)

115 119
,
24
24

D) none of these

1 sin 2 x , 0 x, then a value of x is


B) 0

C) tan-12

D) none of these

The solution of the equation cos103x - sin103x = 1 are


A) -

B) 0

C)

D) p

182

TRIGONOMETRY EQUATIONS

SECTION - A
SINGLE ANSWER TYPE QUESTIONS
1.

2.

3.

4.

5.

6.

7.

8.

9.

10.

11.

16.

17. 18. 19.

20. 21. 22. 23

24. 25.

26.

12. 13.

27. 28.

14. 15.

29. 30.

31

32

33

34

35

36

37

38

39

40

41

42 43

44

45

46

47

48

49

50

51

52

53

54

55

56

57 58

59

60

D A,B, A,B
C,D

61

62

63

64

183

IIT- MATHS

SECTION - B
MULTIPLE ANSWER TYPE QUESTIONS
1.

AB
C

2.

3.

4.

A B AD BD
CD

5.

6.

7.

8.

A B AC AB AB
CD
C C

9.

AB
C

10.

11.

BC AB

184

TRIGONOMETRY EQUATIONS

185

IIT- MATHS

INVERSE TRIGNOMETRIC
FUNCTIONS

186

INVERSE TRIGONOMETRY FUNCTIONS

DEFINITION
If a function is one to one and onto from A to B, then function g which associates each element
y B to one and only one element x A, such that y = f(x), then g is called the inverse function of f
denoted by x = g(y).
Usually, we denote g = f-1 {Read as f inverse}
1
x = f (y).

If cos = x, then may be any angle whose cosine is x, and we write = cos1 x. It means that is
an angle whose cosine is x.
Thus, sin1
where,

is an angle, whose sine is , i.e. = sin1 = n + (1)n


2
2
2
6

is the least positive value of .


6

The functions sin1 x, cos1 x, tan1 x, cot1 x, cosec1 and sec1 x are called inverse circular or
inverse trigonometric functions.
Each of the inverse circular function is multivalued. To make each inverse circular function single
valued, we define principal value. If x is positive, the principal values of all the inverse circular functions

lie between 0 and . If x is negative, the principal values of cos1x, sec1 x and cot1x lie
2

between and , while that of sin1x, tan1x and cosec1 x lie between and 0.
2
2

Function

Domain

sin1x

[ 1, 1]


2 , 2

cos1 x

[1, 1]

[0, ]

tan1x


,
2 2

cot1 x

(0, )

sec1 x

R (1, 1).

[0, ] { /2}

cosec1 x

R (1, 1)


2 , 2 {0}

Note. sin1 x is not to be interpreted as

Range (Principal Values)

1
. The sin1 x is merely a symbol denoting a certain angle
sin x

whose sine is x. The 1 used in sin1 x is not an exponent. Similar argument also works for cos
1
x, tan1x etc.
Remark 1. The inverse trigonometric functions are also written arc sin x, arc cos x etc.

187

IIT- MATHS

GRAPHS OF INVERSE TRIGONOMETRIC FUNCTIONS


sin = x = sin1 x,

where ,
2 2

y = sin1x

y=x

/2

y = sin x

x
1

and x [ 1, 1]
/2

y
y = cosec x
y=x

cosec = x = cosec1 x

y = cosec x

- 2 1


where , 0 0,
2 2

1
1
2

and x ( , 1] [1, )

y = cos x

cos = x
= cos1 x
where [0, ] and x [1, 1]

y=x

y = cos x

y=x

sec = x = sec1x

where 0, ,
2 2
and x ( , 1] [1, )

1
1

y = sec1x
x

y = sec x

188

INVERSE TRIGONOMETRY FUNCTIONS


y
y = tan x

y=x

2
1

y = tan x
2

tan = x = tan1 x

where , and x ( , )
2 2

y
y = cot x
y=x
1

y = cot x

cot = x = cot x
where (0, ) and x ( , )

PROPERTIES OF INVERSE TRIGONOMETRIC FUNCTIONS


Property I:
(i)
(ii)
(iii)
(iv)
(v)
(vi)

sin1 (sin ) = ; for all [ /2, /2]


cos1 (cos ) = ; for all [0, ]
tan1 (tan ) = ; for all ( /2, /2)
cosec1 (cosec ) = ; for all [ /2, /2], 0
sec1 (sec ) = ; for all [0, ], /2
cot1 (cot ) = ; for all (0, )

Property II:
(i)
(ii)
(iii)
(iv)
(v)
(vi)

sin (sin1x) = x,
cos (cos1 x) = x,
tan (tan1 x) = x,
cosec (cosec1 x) = x,
sec (sec1 x) = x,
cot (cot1 x) = x,

for all x [1, 1]


for all x [1, 1]
for all x R
for all x [ , 1] [1, )
for all x ( , 1] [1, )
for all x R

Property III:
(i)
(ii)
(iii)
(iv)
189

sin1 ( x) = sin1 ( x),


cos1 ( x) = cos1 (x),
tan1 ( x) = tan1 x,
cosec1 ( x) = cosec1 x,

for all x [1, 1]


for all x [1, 1]
for all x R
for all x [ , 1] [1, )

IIT- MATHS
(v)
(vi)

sec1 ( x) = sec1 x,
cot1 ( x) = cot1 x,

for all x ( , 1] [1, )


for all x R

Property IV:
1
x

for all x ( , 1] [1, )

1
x

for all x ( , 1] [1, )

1
1 cot x,
=
1
x cot x ,

for x 0
for x 0

(i)

sin1 = cosec1 x,

(ii)

cos1 = sec1 x,

(iii)

tan

Property V:
(i)

sin1 x + cos1x =

,
2

for all x [1, 1]

(ii)

tan1 x + cot1 x =

,
2

for all x R

(iii)

sec1 x + cosec1 x =

,
2

for all x ( , 1] [1, )

Property VI:
If x, y 0, then

(i)

1 x y
tan
, if xy 1
1 xy

,
if xy 1
tan1x + tan1 y =
2

1 x y
tan
, if xy 1
1 xy

(ii)

1
tan1x tan1y = tan 1 xy .

xy

Property VII:
(i)

sin1 x + sin1y =

sin 1 x 1 y2 y 1 x 2 , if 1 x, y 1 and x 2 y2 1

or if xy 0 and x 2 y 2 1.

1
x 1 y 2 y 1 x 2 , if 0 x, y 1 and x 2 y 2 1.
sin

sin 1 x 1 y2 y 1 x 2 , if 1 x, y 0 and x 2 y 2 1.

190

INVERSE TRIGONOMETRY FUNCTIONS


(ii)
sin 1 x sin1 y =

sin 1 x 1 y 2 y 1 x 2 , if

or if

1
x 1 y 2 y 1 x 2 , if
sin

1
x 1 y 2 y 1 x 2 , if
sin

1 x, y 1 and x 2 y 2 1

xy 0 and x 2 y2 1.
0 x, y 1, 1 y 0 and x 2 y 2 1.
1 x 0,0 x y 1 and x 2 y 2 1.

Property VIII:
(i)

cos1x + cos1y =

cos1 xy 1 x 2 1 y2 ,

2 cos 1 xy 1 x 2 1 y2

(ii)

if 1 x, y 1

if

and

xy0

1 x, y 1 and x y 0

cos1 x cos1 y =

cos1 xy 1 x 2 1 y 2 , if 1 x, y 1 and x y.

cos1 xy 1 x 2 1 y 2 , if 1 y 0, 0 x 1 and x y.

Property IX:
(i)

2
1 x

sin1 x = cos1 1 x 2 tan 1

= cot

(ii)

1 x2
x
sec 1

2
x
1 x

cos ec 1 1
, x (0, 1)

1 x2

cos1 x = sin1 1 x 2 tan 1

1
1 1
cosec 1
sec

2
2
x
1 x
1 x

= cot 1

(iii)

1
cos
2
1 x

tan1 x = sin1

= cot

191

2
1 x

, x (0, 1)

1 x2
1
1
2
1
sec 1 x cos ec
x

, x>0

IIT- MATHS

Property X:

1
2x 1 x 2 ,
sin

1
2
2sin1 x = sin 2x 1 x ,

1
2x 1 x 2 ,
sin

(i)

if

2
1
2

2
1
2

x 1

if 1 x

if

1
2
1
1
if x
2
2
1
if
x 1
2

1
3x 4x 3 ,
sin

1
3sin x = sin 1 3x 4x 3 ,

1
3x 4x 3 ,
sin

(ii)

if 1 x

Property XI:
(i)

2 cos 1 2x 2 1 ,

2 cos x =
cos1 2x 2 1 ,

if

0 x 1

1
4x 3 3x , if 1 x
2 cos
2

1
1

1
3
3 cos1 x = 2 cos 4x 3x , if x
2
2

1
1
4x 3 3x ,
if
x 1
cos
2

(ii)

if 1 x 0

Property XII:

(i)

1 2x
,

tan
2

1 2x
2tan1x = tan 1 x 2 ,

1 2x
tan
,
1 x2

if

x 1

if 1 x 1
if x 1

192

INVERSE TRIGONOMETRY FUNCTIONS

(ii)

3x x 3
1
,
tan

1 3x

1 3x x
1
tan
,

3 tan x =
1 3x 2

3
tan 1 3x x ,

1 3x 2

if

if

1
3

1
1
x
3
3

if x

1
3

Property XIII

(i)

(ii)

193

1 2x
,

sin
2

1 2x
2 tan1 x = sin
,
1 x2

1 2x
sin
,
1 x2

1 1 x 2
cos
,
1 x 2

2 tan1 x =
1 x2

cos
,

1 x

if

x 1

if 1 x 1
if x 1

if 0 x

if x 0

IIT- MATHS

WORKED OUT ILLUSTRATIONS


1ILLUSTRATION : 01
If 0 x 1 and sin 1 x cos 1 x tan 1 x , then
(a) /2

(b) /4

(c) = /4

(d) /4 /2

Ans : (D)

Solution :

sin 1 x cos 1 x tan 1 x


tan 1 x and 0 tan 1 x


since 0 x 1 when we fined
2
4
4
2

ILLUSTRATION : 02
If x

(a)

1
1
1
, the value of cos cos x 2sin x is
5

24
25

(b)

24
25

(c)

1
5

(d)

1
5

Ans :(C)

Solution :
The given expression is equal to

cos cos 1 x sin 1 x cos sin 1 x


2

1
= sin sin x x

1
5

ILLUSTRATION : 03

2 tan 1 cos ec tan 1 3 tan cot 1 3 is equal too


(a) /16

(b) /6

(c) /3

(d) /2

Ans :(C)

Solution :
The given expression is equal to

2 tan 1 cos ec tan


3
6

194

INVERSE TRIGONOMETRY FUNCTIONS


1

1

2
1 1

2x
2 tan
6 3
3
3
3

= 2 tan

ILLUSTRATION : 04
2 tan 1 1 tan 1 2 tan 1 3 is equal too
(a) /4

(b) /2

(c)

(d) 2

Ans : (D)

Solution :
1 2

1
tan 1 3
The given expression is equal to 2 tan
1 2

1
1
= 2 tan 3 tan 3 2

ILLUSTRATION : 05

1 x2 1
4 , then
x

1
If tan

(a) x tan 2

(b) x tan 4

(c)x = tan(1/4)

Ans : (D)

Solution :
Taking x tan , tan

1
= tan

1 x2 1
sec 1
tan 1
x
tan

1 cos
1

1
tan 1 tan tan 1 x
sin
2 2

So that according to the given condition

1 1
tan x 4 tan 1 x 8 or x tan 8
2

ILLUSTRATION : 06
sec 2 tan 1 2 cos ec 2 cot 1 3 is equal too
(a) 1
Ans :

195

(b) 5
(D)

(c) 10

(d) 15

(d) x tan 8

IIT- MATHS
Solution :
The given expression is equal to
2

1 tan tan 1 2 1 cot cot 1 3

= 1+ 4 +1 9 = 15

ILLUSTRATION : 07

1
1
2
The equation 2 cos x sin 2 x 1 x

(a) 1 x 1

(b) 0 x 1

is valid for all values of satisfying


(c) 0 x

1
2

(d)

1
x 1
2

Ans : (D)

Solution :
If we denote cos 1 x by y, then
Since 0 cos 1 x 0 2 y 2
Also since

(1)

sin 1 2 x 1 x 2
2
2

sin 1 sin 2 y
2
2

2y
2
2

..(2)

From (1) and (2) we find 0 2 y

0 y

1
0 cos x

which holds if

4
1
x 1
2

ILLUSTRATION : 08

If u cot 1 tan tan 1 tan , then tan is equal to


o
4 2
196

INVERSE TRIGONOMETRY FUNCTIONS


(a)

(b) cot

tan

(c) tan

(d) cot

Ans :

Solution :
Let

1
1
tan tan x, then u cot tan x tan tan x



x x 2x
2
2


u
2

2x

u

4 2

u

4 2

tan x tan


tan x tan
4 2

ILLUSTRATION : 09
1

1
2

1
2

The value of cos 2sin 3cos


(a) 7/4

(b) 11/4

(c) /12

1
1
4 tan 1 is equal too
2

(d) 25/12

Ans : (D)

Solution :
1

1
2

1
2

We have - cos sin 3cos

3
25
2 x 3 x
4
3
6
4
4 12

ILLUSTRATION : 10
3
5

1
1
1
If cos ec x 2 cos 7 cos then the value of is

(a) 44/117
Ans : (B)
197

(b) 125/117

(c) 24/7

(d) 5/3

IIT- MATHS
Solution :
2
3
3
1 7 1
2cot 7 cos cot
cot 1
2x7
4
5
1

1
= cot

24
3
cot 1
7
4

24 3
x 1
cot 1 7 4
=
24 3

7 4

cot 1

44
117

cos ec 1

125
117

198

INVERSE TRIGONOMETRY FUNCTIONS

SECTION - A
SINGLE ANSWER TYPE QUESTIONS
1.

cos 1 (cos 5 / 4) is given by


A) 5/4

2.

B) 3/4

cos 1

B) /2

140
221

tan

1
1
If cot x sin

A) 2
9.

199

C) /2

B) 0

D)

B) 0, /4

C) - /4, /4

D) /4, /2

B) /2

C)

D) 3/2

B) 1/ 3

C) 1

D)

C) 2

D)

1
/ 4 then x equal to
5
B) 3

Number of solutions of sin -1 x + sin -1 2x = /3 is


A) 2

10.

D)

tan 3 tan
tan
is equal to
4
15
5
15

A) - 3
8.

If x + 1 / x = 2, the principle value of sin 1 x is


A) /4

7.

C)

1 1 x
The smallest and the largest values of tan
; 0 x 1 are
1 x

A) 0,
6.

171

B) cos 1
221

2x
If x 1 then 2tan-1 x + sin-1 1 x 2 is equal to

A) 4tan-1 x
5.

D) /4

C) 0

15
1
2 tan 1 is equal to
17
5

1
A) cos

4.

D) - 5/4

yz
xz
1 xy
tan 1
tan 1
If x 2 y 2 z 2 y 2 then tan
is equal to
zr
xr
yr
A)

3.

C) -/4

B) 3

C) 1

x2 x3
x6
-1
-1 2 x
sin
x
+
........
+
cos
x
+
....... = for 0 <|x| <

If
2
4
2
4

D) 4

IIT- MATHS
A)
11.

1
2

B) 1

C)

If a and b are roots of the equation 6X 2 +11X + 3= 0 then


A) both cos 1 and cos 1 are real
C) both cot 1 and cot 1 are real
2n

12.

i 1

X
i 1

B) x

tan

D) x 2 , 2

C) x 1,0

C)

D) 2

B)

cot

C) tan2a

D) cot a

C) 16/7

D) 17/6

-1 3
-1 3
The value of tan sin + cot is
5
2

B) 7/16

3
If sin-1x + sin-1y + sin-1z =
then the value of
2
B) 1

x
x

101

y 101

303

303

x
x

202

y 202

404

404

is equal to

C) 2

D) 3

C) a = x = b

D) a > b, x

ax
x b
1
cos-1 a b sin
a b is possible if
B) a < x < b

If sin-1x = cot--1x , then


A) x2 =

20.

u
1
tan a then tan is equal to
If u = cot-1 tan a tan
4 2

A) a > x > b
19.

D) 2n-1

holds for

B) /2

A) zero
18.

n n 1
2

2(tan-11 + tan-12+tan-13) is equal to

A) 6/17

17.

C)

A)

16.

is equal to

1
1
2
The formula 2 sin x sin 2 x. 1 x

A) /4
15.

B) 2n

A) x0,1
14.

B) both cos ec 1 and cos ec 1 are real


D) none of these

2n

If X i n then
A) n

13.

D) 1

5 1
2

B) x2 =

5 1
2

C) sin(cos-1x)=

5 1
2

D) x =

5 1
2

If x1, x2, x3, x4 are roots of the equation x 4 x 3 sin 2 x 2 cos 2 x cos sin 0 then
tan 1 x 1 tan 1 x 2 tan 1 x 3 tan 1 x 4 is equal to

200

INVERSE TRIGONOMETRY FUNCTIONS


A)
B) /2 -
21.

x2 2
x2 1

If minimum value of

-1
If cos

201

D) 1,

C) 0

D) none of these

C) (a + b) (a2 + b2)

D) (a - b) (a2 - b2)

C)
2

sin x sin x
y

D) 3/2

2 then the value of K is


K

C) 8

D) 16

B) 5/4

C) 1

D)

C) 2 / 8

D) 2 / 32

Minimum value of (sec-1 x)2 + (cosec-1 x)2 is


B) /32

The number of real solutions of tan 1 x( x 1 ) sin 1 x 2 x 1 / 2 is


B) one

C) two

D) infinite

1
3
1 1
1
3 sin 1 sin 1 = is then
The two angles are A = 2 tan 2 2 1 and B = 3 sin
3
3
5

B) A < B

C) A = B

D) A + B = 0

An integral solution of the equation tan-1x + tan-1y = tan-13 is equal to


A) (2, 7)

32.

C) 0, 1/2

x
y 5
y
x 2 y2
1 x
+ cot -1 =
sin 1
and sin
then value of 2 2 is
2
b 12
2
b 12
a
b

A) A > B
31.

B) (a + b) (a2 - b2)

B) 6

A) zero
30.

B) 0,1/2

B)

A) /8
29.

x2 2

x 2 xy y 2
x
1 y

cos
/ 6 then the value of
If cos
is
4 2 3 9
2
3

A)
28.

x2 2

D)

A) 4
27.

x2 2

x2

1
3

21
1
cos ec 2 tan 1
sec tan

2
2
is equal to
2
2

A)
26.

C)

If sin 1 (1 x ) 2 sin 1 x / 2, then x equals

A) (a - b) (a2 + b2)
25.

x2 1

B) 1, 0

A) 0, -1/2

24.

B)

If sin 1 x sin 1 (1 x ) cos 1 x , then x equals


A) 1, -1

23.

D)

The value of sin (cot-1 (cos (tan-1x)) is


A)

22.

C)

B) (4, -13)

The number of real solutions of (x, y) where

C) (5, -8)

D) (1, 2)

cos 1 (cos X ) tan x ,0 x 2 is


2

A) 2
33.

B) 1

IIT- MATHS
D) None

C) 4

Sum of infinite series

3
3
2 3

1 cot 1 22 cot 1 32 ....

cot
is
4
4
4

-1

B) tan-12

A) /4

34.

tan

m 1

B) -/4

1 2
x2
2

x
1

cos-1 2
4

A) |x| 1

36.

B) x R

2x

C) 0 x 1

1 x2

if 0 x 1

2a
The solution of sin 1 a 2

ab
1 ab

B) f(x) =

1 x2

if x < 1

B)

1 1 b

cos

1 b

1 2x
tan
2 is
1 x

1 ab
ab

C)

ab 1
ab

D)

ab
1 ab

B) /2

C) 1

D) none of these

C) p/3

D) 4p/3

2
1
The principal value of sin sin
is
3

A) 2p/3

B) - 2p/3

3
1
1
1
The equation sin x cos x cos 2 has

A) no solution
C) infinite no.of solution
41.

2x

If xy + yz + zx = 1 then tan-1x + tan-1 y + tan-1z is equal to


A)

40.

D) 1 x < 0

D) None

-1

A)

39.

D) None

x
cos 1

2 - cos--1 x holds for

C) not finite if x > 1

38.

C) /4

2
1
1
1 2x
1 1 x
sin

cos

The value of f(x) = tan 2


1 x2 2
1 x 2 is

A) f(x) =

37.

D) None

2m
4
is equal to
2
m m 2

zA) /4

35.

C) tan-13

B) unique solution
D) none of the these

If tan + tan ( + /3) + tan ( - /3) = k tan 3 then the value of k is


202

INVERSE TRIGONOMETRY FUNCTIONS


A) 1
B) 1/3
42.

B) 1

1 3
2 7

1
If tan

D)

1 7
2 3

D) 3/2

C) - 3tan-1 x

D) -2tan-1 x

4 3

5 5

4 3
3 8

B) ,

4 3
5 7

C) ,

D) ,

B) xR

C) x(--1] [1]

D) x(-1, 1)

x
1 x2

sin 1 x holds is
C) [0,1]

D) [-1,0]

C) 1/2

D) 3 / 2

C) 25

D) 27

If sin 1 x cos 1 x / 6 then x is equal to


B)

3 /2

sec2(tan-12) + cosec2(cot-13)+2 is equal to


B) 15

x
y
2
2
-1
If cos a +cos b = a, then x 2xy cos y
a 2 ab
b2
-1

A) sin2

B) cos2

C) tan2

D) cot2

1
1
1
If tan-1 ( x 1 ) tan x tan ( x 1 ) tan ( 3 ) then x=

A) zero
203

B) + 3 tan-1 x

B) (-1,1)

A) 17

52.

C)

1
The set of values of x for which tan

A) 1/2

51.

1 7
2 3

Let f(x) = sec 1 x tan 1 x then f(x) is real for

A) R

50.

C)

x 1
2x 1
23
tan 1
tan 1
then x equal to
x 1
2x 1
36

A) x[-1, 1]

49.

3
7

3
1

1 3x x
tan
,
3

If x e
the value of
2 is
3

1 3x

4 3
5 8

48.

1
2

B)

A) ,
47.

B)

x 2 xy y 2
x
-1 y
+

+ sin
= / 6 hen value of
If sin
is
9 4 3 16
3
4

A) - 3tan-1 x
46.

D) 3

-1

A)

45.

C) 2

If sin -1x + sin -1 2x = / 3 then x equal to


A)

44.

D) 1/6

9
100
100
100
If sin -1x + sin -1 y + sin -1z = 3 / 2 the value of x y z x101 y101 z101 is

A) 0
43.

C) 3

B) 1

C) 3

D) 13

IIT- MATHS
53.

1
If l is a root of x2 + 3x+1=x0, then tan-1l + tan1 is equal to
A) /2

54.

B) - /2

The equation 2cos-1x = sin-1(2x


A) 1 x 1

55.

If cos tan

sin cot

C) y = -1

D) None

C) 60

D) 30

C) 2

D) infinite

C) 0

D) 1/2

B) 1

If 2tan-1 (cosq) = tan-2 (2 cosecq) then q equal to


B) n + /4

C) - /4

D) 2n - /4

3 5 cos x
cos 1
is equal to
5 3 cos x
x
2

x
2

1
B) 2 tan 2 tan

C)

1
x

tan 1 2 tan
2
2

1
2

x
2

1
D) 2 tan tan

Minimum value of (tan1 x)2 + (cot1 x)2 is


A)2/4

B) 2/8

C) 2/2

D) 2/32

1
1 x
3 3x 2 then
If f ( x ) cos x cos
2 2

2
3

1
B) f 2 cos

1
3

1
/3
D) f 2 cos

A) f / 3
C) f / 3
63.

x1

5 2
If (tan x ) (cot x )
, then x equals
8
1

1
2

62.

B) 1

1
A) tan tan

61.

If < 1/32 then the number of solutions of (sin-1x)3 + (cot-1 x)3 = ap3 is

A) n - /4
60.

D)

2 y then

B) 45

A) 1
59.

2x k
x 3
1
-1
tan

and B = tan k 3 then the value of A B is


If A =

2k x

A) 0
58.

C) 0 x

B) y = 3 / 2

A) 0
57.

D) None of these

1 x 2 ) is valid for all values of x satisfying

B) 0 x 1

A) y = 4/5

56.

C) /3

2
3

2
/3
3

1
3

1
3

If a, b, g are the roots of the equation x 3 mx 2 3x m 0, then the general value of


204

INVERSE TRIGONOMETRY FUNCTIONS


tan -1 + tan -1 + tan -1 is
A) (2n+1) /2
64.

D) /4 + n

B) 1

C) 3

D) 4

1
x 2 x 1 /2 is
The number of real solutions of tan-1 x( x 1 ) sin

A) zero
66.

C) n/2

The number of positive integral solution of the equation tan1x + tan1 1/y = tan1 3
A) 2

65.

B) n

B) 1

C) 2

D)

The sum of the infinite series


2 1
3 2
n n 1
1
1
1
sin 1
sin
...... sin
....

sin 2
6
12
n
(
n

1
)

-1

A) /8

67.

D)

B) 4

C) 6

D) 8

If sin1x + sin-1y = 2/3, cos-1x cos-1y = /3, then the number of values of (x, y) is
A) two

205

C) /2

n 2 10n 21.6
, , n N, then the maximum value of n is
If cot-1

6
A) 2

68.

B) /4

B) four

C) zero

D) None

IIT- MATHS

SECTION - B
MULTIPLE ANSWER TYPE QUESTION
1.

The x satisfying sin-1x + sin-1 (1-x) = cos-1 x are


A) 0

2.

If

B) 1/2

D) cos-1x

B) x (0, 1/ 2 )

C) x (1/ 2 , 1)

D) x = 0.75

C) x = 3

D) x = 4

B) x = 2

5
2

Let q = tan-1 tan


and f = tan-1 tan then
4
3

A) >

6.

C) sec-1x

6 sin-1 (x2-6x+8.5) = , if
A) x = 1

5.

B) tan-1x

sin-1x > cos-1x holds for


A) all values of x

4.

8
= 5 / 18
9 e

B) f

7
= /12
4 e

D) f

8
= 13 / 8
9 e
7
= 11 / 12
9 e

If a sin-1 x + cos-1x + sin-1 x b, then


A) =0

B) = /2

C) = /4

D) =

The greatest and least values of (sin-1x)3 + (cos-1x)3 are


A) 3/32

9.

D) none of these

Let f(x) = e cos sin( x / 3 ) then

C) f

8.

C) + = 7/12

B) 4 - 3 = 0

A) f

7.

D) 2

1
X 1 then which of the following are real ?
2

A) sin-1x
3.

C) 1

tan-1
A) /4

B) - 3/8

C) 73/8

D) /2

a( a b c )
b( a b c )
c( a b c )
tan 1
tan 1
is
bc
ca
ab
B) /2

C)

D) 0

206

INVERSE TRIGONOMETRY FUNCTIONS

SECTION - A
SINGLE ANSWER TYPE QUESTION

1.

2.

3.

4.

5.

6.

7.

8.

9.

10.

11.

16.

17. 18. 19.

20.

21. 22. 23

24. 25.

26.

12. 13.

27. 28.

14. 15.

29. 30.

A,C

31

32

33

34

35

36

37

38

39

40

41

42 43

44

45

A,B
C,D

A,C

46

47

48

49

50

51

52

53

54

55

56

57 58

59

60

61

62

63

64

65

66

67

68

207

IIT- MATHS

SECTION - B
MULTIPLE ANSWER TYPE QUESTIONS

1.

2.

AB

AB
D

3.

4.

CD BD

5.

6.

7.

8.

BC BC AD AC

9.

CD

208

INVERSE TRIGONOMETRY FUNCTIONS

209

IIT- MATHS

PROPERTIES OF TRIANGLE

210

PROPERTIES OF TRIANGLE
In a triangle ABC the angles are denoted by capital letters A, B and C and the length of the sides
opposite to these angles are denoted by small letters a, b and c. Semi perimeter of the triangle is
given by s =

abc
and its area is denoted by .
2

SINE RULE
In a triangle ABC,
a
b
c

sin A sin B sin C

COSINE RULE
In a triangle ABC,
b2 c2 a 2
(i) cosA =
2bc

(iii)cosC =

c2 a 2 b2
(ii) cosB =
2ca

a 2 b2 c2
2ab

PROJECTION FORMULAE
(i) a = b cos C + c cos B

(ii) b = c cos A + a cos C

(iii) c = a cos B + b cos C

NAPIERS ANALOGY (TANGENT RULE)


A
BC bc
cot

2
2 bc

(i) tan

B
CA ca
cot

2
2 ca

(ii) tan

C
A B a b
cot

2
2 ab

(iii) tan

HALF ANGLE FORMULAE


(a)

(b)

(i) sin

(iii) sin

(i) cos

A
=
2

(iii) cos

211

s bs c
bc

(ii) sin

s c s a
ca

s a s b
ab

ss a
bc

s s c
C

2
ab

(ii) cos

B
=
2

ss b
ca

IIT- MATHS

(c)

(i) tan

s b s c
s s a

A
=
2

2
2
s s a s b s c
bc
bc

(iii) sin C =

B
=
2

s cs a
ss b

s a s b
ss c

(iii) tan 2
(i) sin A =

(ii) tan

(ii)

sinB=

2
2
s s a s b s c
ca
ca

2
2
s s a s b s c
.
ab
ab

m-n THEOREM
Let D be a point on the side BC of a ABC such that BD : DC = m : n and ADC = ,
BAD = and DAC = . Then
(i)

(m + n) cot = m cot n cot

(ii)

(m + n) cot = n cot B m cot C

CENTROID AND MEDIANS OF A TRIANGLE


The line joining any vertex of a triangle to the mid point of the opposite side of the triangle is
called the median of the triangle. The three medians of a triangle are concurrent and the point of concurrency
of the medians of any triangle is called the centroid of the triangle. The centroid divides the median in the
ratio 2 : 1
a sin B

sin =

2b 2 2c 2 a 2

CIRCUM CIRCLE
The circle which passes through the angular points of a ABC, is called its circumcircle. The
centre of this circle i.e., the point of concurrency of the perpendicular bisectors of the sides of the ABC,
is called the circumcentre.

O
A

B
a/2

a/2

212

PROPERTIES OF TRIANGLE
Radius of the circumcircle is given by the following formulae
R=

a
b
c
abc

2sin A 2 sin B 2 sin C 4

BD
In BDM,
= tan A or
MD

a
2 = tan A, i.e., a = tan A,
2x
x

Similarly, 2 y = tan B,
= tan C
2z

z
M
A

a
b
c
tan A + tan B + tan C = 2x 2y 2z

and

tan A. tan B. tan C = 2x . 2 y . 2z

y
x

But in a triangle ABC,tan A + tan B + tan C = tan A. tan B. tan C

a b c abc

x y z 4xyz .

ORTHOCENTRE AND PEDAL TRIANGLE OF A TRIANGLE.


In a triangle the altitudes drawn from the three vertices to the opposite sides are concurrent and
the point of cuncurrency of the altitudes of the triangle is called the orthocentre of the triangle. The
triangle formed by joining the feet of these perpendiculars is called the pedal triangle i.e.
DEF is the pedal triangle of ABC.
A

F
E
P
0

90 C

INCIRCLE
The circle which can be inscribed within the triangle so as to touch each of the sides of the triangle
is called its incircle. The centre of this circle i.e., the point of concurrency of angle bisectors of the
triangle is called the incentre of the ABC.
A

r
I

900 B/2

B/2

213

r
D

F
r

C/2

IIT- MATHS
Radius of the Incircle is given by the following formulae
r=

A
B
C
A
B
C
= (s a) tan = (s b) tan = (s c) tan = 4R sin sin sin .
s
2
2
2
2
2
2

ESCRIBED CIRCLES
The circle which touches the side BC and the two sides AB and AC produced is called the escribed
circle opposite the angle A. Its centre and radius will be denoted by I1 and r1 respectively.
Radii of the excircles are given by the following formuale
A

(i)
(ii)

r1 =

A
A
B
C
s tan 4R sin cos cos
sa
2
2
2
2

F1

B
A
B
C
s tan 4R cos sin cos
r2 =
sb
2
2
2
2

E1
L

(iii)

D1

I1

C
A
B
C
s tan 4R cos cos sin .
r3 =
sc
2
2
2
2

BISECTORS OF THE ANGLES


If AD bisects the angle A and divide the base into portions x and y, we have, by Geometry,
x AB c

y AC b

x y xy
a

c b bc bc

ac
ab
and y =
bc
bc
Also let be the length of AD
we have ABD + ACD = ABC

x=

1
A 1
A 1
c sin b sin bc sin A,
2
2 2
2 2

i.e.,

bc sin A
2bc
A

cos
b c sin A b c
2
2

SOLUTION OF TRIANGLES
When any three of the six elements (except all the three angles) of a triangle are given, the triangle
is known completely. This process is called the solution of triangles.
(i)
(ii)

b 2 c2 a 2
If the sides a, b and c are given, then cos A =
. B and C can be obtained in
2bc
the similar way.
If two sides b and c and the included angle A are given, then using

tan

BC b c
A
BC

cot , we get
.
2
bc
2
2
214

PROPERTIES OF TRIANGLE
BC
A
Also
= 900
, so that B and C can be evaluated.
2
2
The third side is given by a =
(iii)

b sin A
.
sin B

c
If two sides b and c and the angle B (opposite to side b) are given, then sin C = sin B,
b
b sin A
give the remaining elements. If b < c sin B, there is
sin B
no triangle possible (fig 1). If b = c sin B and B is an acute angle, then there is only one

A = 1800 (B + C) and a =

triangle possible (fig 2). If c sin B < b < c and B is an acute angle, then there are two
values of angle C (fig 3). If c < b and B is an acute angle, then there is only one triangle
(fig 4).
A
A

c
b

(Fig 1)

b c sinB

c sinB

D
(Fig 2)
A

A
b

c
b

b
D

C2

c sinB

b
c

c sinB

C2

C1

C1

(Fig 4)

(Fig 3)

This case is, sometimes, called an ambiguous case.


Let I be the in-centre of O be the circumcentre of the triangle ABC. Let OL be parallel to BC.
Let IOL . IM = r OC = R, NOC A

tan

IL IM LM IM ON

OL BM BN BM NC

r R cos A
B
r cot R sin A
2

A
B
C
sin sin R cos A
2
2
2

A
B
C
B
4R sin sin sin .cot R sin A
2
2
2
2

4R sin

215

cos A cos B cos C 1 cos A cos B cos C 1

sin A sin C sin B sin A


sin C sin B

cos B cos C 1
tan 1
.
sin C sin B

I
O
B

NM

IIT- MATHS

WORKED OUT ILLUSTRATIONS


ILLUSTRATION : 01
1

If two sides of a triangle are

(a)

(b)

6 2

3
2

(c)

and

6 2

1
3

, and the included angle is 600, then the third side is

(d) 2 3

Ans : (B)

Solution :
If a represents the third side then
2

1
1


a 2
6 2
6 2
cos 60 0
1
1
2x
x
6 2
6 2

1 26 2 a 2 6 22

2
26 2

2
a

3
3
a
4
2

ILLUSTRATION : 02
If the angles A, B, C of a triangle ABC are in arithmetical progression then
(a) tan A tan C 3 tan A tan C 3

(b) tan A tan C 3 tan A tan C 3

(c) tan A tan C 3 tan A tan C 3

(d) tan A tan C 3 tan A tan C 3

Ans : (C)

Solution :
Since A, B, C are in Arithmetical progression

2 B A C also A + B +C = 1800 so that B = 600


In a triangle ABC, we know
TanA + tanB + tanC = tanAtanBtanC
tanA+tanC =

3 (tanAtanC-(A)

tanA+tanC - 3 tanAtanC = 3

ILLUSTRATION : 03
216

PROPERTIES OF TRIANGLE
If the angles of a triangle are in the ratio 1: 3: 5 and denotes the smallest angle, then the ratio of the largest
side to the smallest side of the triangle is
3 sin cos
2 sin

(a)

(b)

3 cos sin
2 sin

(c)

cos 3 sin
2 sin

(d)

3 cos sin
2 sin

Ans : (D)

Solution :
Since the angles of the triangle are in the ration 1: 3: 5
Let A 20 0 , B 60 0 , C 100 0

We have 3 5 180 0 20 0 ;
a
b
c

then from
sin A sin B sin C

c sin C sin 120 0

We get the required ratio =


a sin A
sin

cos 1 sin
2
2
=

sin

ILLUSTRATION : 04
The expression

a b cb c a
c a b a b c is equal to

2 A
(a) cos
2

2 A
(b) sin
2

2 A
(c) cot
2

2 A
(d) tan
2

Ans : (C)

Solution :
The given expression is equal to

b c2 a 2
a 2 b c 2

b 2 c 2 a 2 2bc

2bc b 2 c 2 a 2

2b cos A 2bc 1 cos A


A

cot 2
2bc 2bc cos A 1 cos A
2

ILLUSTRATION : 05
In a cyclic quadrilateral ABCD; a, b, c, d denote the length of the sides AB, BC, CD and DA respectively, then
cosA is equal to

(a)

a 2 b2 c2 d 2
2ab cd

217

(b)

b2 c2 d 2 a2
2bc da

(c)

c2 d 2 a 2 b 2
2cd ab

(d)

d 2 a 2 b 2 c2
2da bc

IIT- MATHS
Ans : (D)

Solution :

We have from ABD

BD2 = a 2 d 2 2ad cos A

from BDC

A
a

BD2 = b 2 c 2 2bc cos C

= b 2 c 2 2bc cos A

Equating the two values we get

cos A

d a 2 b2 c 2
2 da bc

ILLUSTRATION : 06
2 p q 1 pq
A
B
tan

p
,
tan

q
,
In a triangle ABC, if
then 1 p 2 1 q 2 is equal to


2
2
Ans : (C)

Solution :
The given expression is equal to
A
B
A B
2 tan tan 1 tan tan
2
2

2 2

A
B
1 tan 2 1 tan 2
2
2

C
C
A B A B
= 2 sin cos 2 cos sin sin C
2
2
2 2 2 2

ILLUSTRATION : 07
In a triangle ABC if
(a) an acute angle

cos A cos B cos C a

, then A is
a
b
c
bc

(b) an obtuse angle

(c) a right angle

(d) equal to B-C

Ans :

Solution :
We have

cos A cos B cos C

a
b
c

218

PROPERTIES OF TRIANGLE

b 2 c 2 a 2 c 2 a 2 b 2 a 2 b 2 c 2
2abc
a 2 b2 c2
a

(given)
2abc
bc

a 2 b 2 c 2 2a 2 b 2 c 2 a 2

b 2 c 2 a 2 0 cosA = 0 A =
2

ILLUSTRATION : 08
If A, B, C, D are the angles of a quadrilateral, then

tan A tan B tan C tan D


is equal to
cot A cot B cot C cot D

(a) tanAtanBtanCtanD

(b) cotAcotBcotCcotD

(c) tan 2 A tan 2 B tan 2 C tan 2 D

(d)

Ans:

tan A tan B tan C

(A)

Solution :

We have tanA B tan 360 0 C D tanC D

tan A tan B
tan C tan D

1 tan A tan B
1 tan C tan D

tan A tan B1 tan C tan D 1 tan A tan Btan C tan D 0


tan A tan B tan C tamD

tan A tan B tan C

tan A tan B tan C tan D


1
1
1
1

tan A tan B tan C tan D


tan A tan B tan C tan D

tan A tan B tan C tan D


tan A tan B tan C tan D
cot A cot B cot C cot D

ILLUSTRATION : 09
A
B acb
cos
k, then
2
2
2c
B
C
(b) k sin
(c) k sin
2
2

In a triangle ABC, if sin


A
(a) k cos
2

Ans : (D)

Solution :
sin

219

A
B acb
cos
k
2
2
2c

C
(d) k cos
2

IIT- MATHS

s b s c x ss - b a c b k
bc

ca

2c

s b ss c s b

k
c
ab
c

cos

C
k
2

220

PROPERTIES OF TRIANGLE

SECTION - A
SINGLE ANSWER TYPE QUESTIONS
1.

In any DABC if cot A/2, cot B/2, cot C/2 are in A.P. then a,b,c are in
A) A.P

2.

C) H.P

D) none of these

If the triangles A,B, C of a triangle are in A.P and sides a,b,c are in G.P. then a2, b2, c2 are in
A) A.P

3.

B) G.P

B) H.P

C) G.P

D) none of these

If twice the squares of the diameter of a circle is equal to half the sum of the squares of the sides
of inscribed triangle ABC then sin 2 A sin 2 B sin 2 C is equal to
A) 1

4.

B) 2

C) 4

sin A sin( A B )
If in a triangle ABC sin C sin( B C ) then

B) a2, b2, c2 are in A.P


D) a2, b2, c2 are in H.P

A), a, b, c in A.P
C) a,b,c are in H.P
5.

a 2 b 2 sin( A B )

If in a triangle ABC, 2
then the triangle is
a b 2 sin ( A B )

A) right angled or isosceles


C) equilateral
6.

1
A

C) G.P

D) none of these

B) G.P

C) H.P

D) none of these

B)

1
A

C) A

1
1
1

A1
A2
A3
D)

1
A2

B) 12 cm

C) 16 cm

D) 18 cm

If orthocentre H of a DABC bisect the altitude AD of the triangle ABC, then value of tanB tan C is
A) 3

221

D) none of these

If length of the side BC of a DABC is 6cm and BAC = 120 then the distance between in
centre and excentre of the circle touching the side BC internally is
A) 10 cm

11.

B) H.P

If A, A1, A2, A3 are the areas of incircle and the ex-circles of a triangle, then

A)
10.

C) G.P

If r 1 2r 2 3r 3 then a, b, c are in
A) A.P

9.

B) H.P

In a triangle ABC if tan A/2, tan B/2, tan C/2 are in H.P then a,b,c are in
A) A.P

8.

B) right angled and isosceles


D) none of these

If in a DABC, cosA +2cosB + cosC = 2 then a, b, c are in


A) A.P

7.

D) 8

B) 2

C) 1

D) 0

12.

IIT- MATHS
Which of the following pieces of data do not uniquely determine an acute angle triangle ABC (R
being the radius of the circumcircle)
A) a, sinA, sin B

13.

B) a, b, c

C) a, sin B, R

D) a, sin A, R

In a triangle ABC. Let C / 2 .If r is the inradius and R is the circumradius of the triangle,
then 2(r + R) is equal to
A) a + b

14.

D) l > 4

B) 3a2 = b2 3c2

C) b2 = a2 c2

D) a2+b2 = 5c2

B) tanB = b/a

D) sin2A+sin2B+sin2C=0

C) cosC =0

B) H.P

C) G.P.

D) None of these

B) 16/9

C) 25/27

D) 27/25

B) 20 3 3

C) 20 3 3

D) 3- 3

If the sides of a triangle are in A.P., and the greatest angle of the triangle exceeds the least by 900
then the sine of the third angle is
7 /4

B)

7 /2

C)

7 /8

D)

If one angle of a triangle is 300 and the length of the sides adjacent to it are 40 and 40 3 then the
triangle is
A) right angled

24.

C) 0 < l < 4

If ABC is a triangle in which B=450, C=1200 and a=40, the length of the perpendicular from A on
BC produced is

A)
23.

B) l > 6

In a triangle ABC a : b : c = 4 : 5 : 6 then the ratio of the circumcircle to that of incircle is

A) 3+ 3
22.

D) 4

In a triangle ABC, if tan A/2 = 5/6, and tan B/2 = 20/37 and the sides a, b, c are in

A) 16/7
21.

C) 6

If the tangents of the angles A and B of a triangle ABC, satisfying the equation abx2 -c2x+ab=0
then

A) A.P.
20.

D) 120

If D is the midpoint BC of a triangle ABC and AD is perpendicular to AC, then

A) tanA = a/b
19.

C) 90

In a triangle ABC, (a+b+c) (b+c-a) = l bc if

A) 3b2 = a2 c2
18.

B) 60

B) 7

A) l < 0
17.

D) a + b + c

Let Tn denotes the number of triangles which can be formed using the vertices of a regular polygon of n sides. If Tn +1 Tn = 21 then n equals
A) 5

16.

C) c + a

In a triangle ABC 3sinA = 6sinB = 2 3 sin C then angle A is


A) 30

15.

B) b + c

B) isosceles

C) obtuse angled

D) None of these

Points D,E are taken on the side BC of a triangle ABC, such that BD = DE = EC. If

222

PROPERTIES OF TRIANGLE
BAD X , DAE Y EAC X then value of

A) 1
25.

B) 2

sin( x y ) sin ( y z )
is equal to
sin x sin z

C) 4

D) None of these

If A, B, C are angles of a triangle, then the minimum value of tan a 2 A / 2 tan 2 B / 2 tan 2 C / 2
is equal to
A) 0

26.

B) 1

If in a triangle ABC,
A) 1/5

27.

C)

D) none of these

bc ca a b

then cosA is equal to


11
12
13
B) 5/7

C) 19/35

D) 20/35

If P1, P2, P3 are the altitudes of a triangle ABC from vertices A,B, C and D is the area of the
triangle then p1 1 p 2 1 p 3 1 is equal to
A)

28.

sa

B)

B) G.P

B) /4

B)

2 :1:1

B) 5 cm

B) 3 abc

C) 5/12

D) /2

C) 1 : 2 : 1

D) 1 : 1 : 1

C) 5.5 cm

D) 6 cm

C) abc

D) 4 abc

B) sin /n : cos /n
D) tan /n : cos /n

P
Q
In a triangle PQR R / 2 If tan and tan are roots of the equation ax 2 bx c 0
2
2

a 0

then

A) a + b = c

223

b
3

then angle A is equal to


c
2

The ratio of the radius of circumcircle and incircle of a regular polygon of side n is
A) cosec /n : cot /n
C) tan /n : cot /n

34.

D) none of these

The minimum value of bc (b+c) cos A + ca(c+a) cos B + ab (a +b) cos C is equal to
A) 2abc

33.

2s

Radius of the circumcircle of ABC is 3cm. If I1, I2, I3 are the centres of the excircles of the
triangle ABC then radius of the circumcircle of I1 I2 I3 is
A) 4.5 cm

32.

D)

If in a DABC cos B cos C + sin A sin B sin C = 1 then a : b : c is equal to


A) 1 : 1 :

31.

C) H.P

Angles A, B and C of a triangle ABC are in A.P. If


A) /6

30.

C)

If A1, A2, A3 denote the respectively the areas of an inscribed polygon of 2n sides, inscribed
polygon of n sides and circumscribed polygon of n sides then A2, A1, A3 are in
A) A.P

29.

sb

B) b + c = a

C) a + c = b

D) b = c

IIT- MATHS
35.

There exists a triangle ABC satisfying


A) tanA + tanB + tanC = 0
B)

sin A sin B sin C

2
3
7
2 sin A cos A 3

C) (a+b)2 = c2+ab and


D) sinA + sinB =
36.

3 1
3
sin A sin B
, cosAcosB 2
4

Given an isosceles triangle with equal side of length b, base angle a < p/4, R, r the radii and 0, I
the centres of the circumcircle and incircle respectively, then
A) R =

1
b cos ec
2

B) D = 2b2 sin2a
b cos

b sin 2

C) r = 21 cos

37.

If in a triangle ABC,

D) OI =

B) /4

2
sin

B)

A)

abc
2

C)

sin
2

D) None

bc ca a b


r1
r2
r3
r

B) s

C) 2s

D) 3s

B) 2

C) 3

D) 4

In a triangle the length of the two larger sides are 24 and 22 respectively. If the angles are in A.P.
then the third side is
A) 12 2 3

B) 12 2 3

C) 2 3 2

D) 2 3 2

If A + B + C = , n = z then tan nA + tan nB +tan nC is equal to


A) 0
C) tan nA tan nB tan nC

43.

D) 2/3

In any ABC b2 sin2C +C2 sin2B is equal to


A)

42.

C) /6

2
sin

39.s In a triangle ABC the value of

41.

If the area and an angle of of a triangle are given, then the side opposite to the given angles is
minimum when the triangle is isosceles with the length of the equal sides equal to
A)

40.

2 sin cos

2CosA CosB 2CosC a


b


, then A is
a
b
c
bc ca

A) /2
38.

3
2

B) 1
D) none of these

AB
If in a ABC, a tanA + a tan B = (a+b) tan
then
2
224

PROPERTIES OF TRIANGLE
A) A = B
B) A = -B
44.

B) b

The value of

C) 2b

D) b

1
1
1
1
2 2 2 is
2
r1
r2
r3
r

A) 0
46.

D) B = 2A

Let in a ABC such that A 45 , A 75 then a + c 2 is equal to


A) 0

45.

C) A = 2B

B)

a 2 b2 c2
2

C)

2
a 2 b2 c2

D)

a 2 b2 c2

If the angles of a triangle are 30 and 45 and the included side is ( 3 1) cm then the area of the
triangle is
A)

47.

1
3 1

B) 6

3 /2

225

B)

5 /2

C)

D)

B) tanA+2tanB = 0 C) tanA2tanB = 0

D) 2tanAtanB = 0

B) a 2 b 2 3c 2

C) a 2 c 2 2 b 2

D) a 2 b 2 2 c 2

B) obtuse angled

C) isosceles

D) equilateral

B) c 2 a 2 b 2

C) b 2 c 2 a 2

D) c 2 a 2 b 2

The sides of a triangle are in the ratio 1 : 3 : 2 then the angles of the triangle are in the ratio
B) 2 : 3 : 4

C) 3 : 2 : 1

D) 1 : 2 : 3

In a triangle, the lengths of the two larger sides are 10 and 9 respectively, if the angles are in A.P.,
then the length of the third side be
A) 5- 6

56.

D)

1
In a triangle ABC 2ac sin ( A B C ) is equal to
2

A) 1 : 3 : 5
55.

C) 1

In a ABC if a2 sin (B-C) + b2 sin (C A) + c2 Sin (A B) = 0 then triangle is

A) a 2 b 2 c 2
54.

D) 4

In a ABC if median AD is perpendicular to the side AB, then which of the following is true

A) right angled
53.

D) 4

If in a ABC the median AD is perpendicular to the side AB then

A) a 2 c 2 3b 2
52.

3 1

C) 1

B) 0

A) 2tanA+tanB = 0
51.

If length of the sides of a DABC are 3,4 and 5 then distance between its incentre and circumcentre is
A)

50.

C)

In a if a = t 2 1, b t 2 1c c = 2t then value of r r1 r2 r3 is
A) 1

49.

3 1

If in a triangle ABC a = 5, b = 4 and cos (A-B) = 31/32 then the third side c is equal to
A) 4

48.

B)

B) 3 3

C) 5

If H is the orthocentre of the triangle ABC, then AH is equal to

D) 5+ 6

IIT- MATHS

A) 2R cosA
57.

B) 2R sinA

B) 1 : 3

C) 3 : 5

D) None

The cosine of the obtuse angle formed by the medians drawn from the vertices of the acute angles
of an isosceles right angled triangle is
A) 4/5

60.

B) the altitudes are in H.P.


D) None of these

In an isosceles right angled triangle, a straight line is drawn from the midpoint of one of equal
sides to the opposite angle. Then the ratio of the tangents of the two parts in two which it is
divided by the line is
A) 3 : 2

59.

C) a cotA

If in a triangle ABC, sinA, sinB, sinC are in A.P., then


A) altitudes are in A.P.
C) altitudes are in G.P.

58.

2abc
D)
cosA

B) 4/5

C) 3/5

D) 3/5

If P1, P2, P3 are the altitudes of a triangle ABC from the vertices A, B, C and D the area of the
triangle then P1 2 P 2 2 P 3 2 is equal to
A)

61.

abc

a 2 b2 c2
2

D)

abc
2

B) 1/R

C) 1/

D) r/

If P is the product of the sines of angles of a triangle and q the product of their cosines, the
tangents of the angles are roots of the equation
A) qx 2 px 3 1 q x p 0

B) px 3 qx 2 1 p x q 0

C) 1 q x 3 px 2 qx p 0

D) qx 3 px 2 1 q x p 0

If length of the sides AB, BC and AC of a triangle are 8cm, 15cm, 17 cm respectively, then length
of the angular bisector of ABC is
A)

64.

C)

cosA cos B cos C

is equal to
P1
P2
P3

A) 1/r

63.

a 2 b2 c2

If P1, P2, P3 are respectively the perpendiculars from the vertices of a triangle to the opposite
sides, then

62.

B)

120 2
cm
23

B)

60 2
cm
23

C)

30 2
cm
23

D)

30
23

In a right angle ABC, right angled at B,D and F are the points on BC such that ADB 2ACB
and AEB 3 ACB , then ratio of DE and CD will lie in the interval
1
2

A) ,1
65.

In a

1
3

B) ,1

1 2
3 3

C) ,

1 1
3 2

D) ,

ABC , B / 3 and C / 4 Let D divide BC internally in the ratio 1 : 3 then

sin BAD
equals
sin CAD
226

PROPERTIES OF TRIANGLE
A) 1/ 6
B) 1/3
66.

B) 3 3

2bc sin

A
2

B)

2bc cos

bc

A
2

bc

abc

C) 2R b C cosec A/2

D)

4
A
cos ec
bc
2

B) 3

C) 6

D) 12

B) 1130

C) 930

abc
then B =
b c2
2

D) None

B) A.P.

C) H.P.

D) None of these

If cos2A + cos2B +cos2C = 1, then the triangle is


A) isosceles

227

3 3
2

If the angles A, B and C of a triangle ABC are the sides a, b and c opposite these angles are in G.P.
then a2 , b2 , c2 are in
A) G.P.

71.

D)

In a triangle ABC, AD is the altitude from A, given b > c, C = 230 and AD=
A) 130

70.

C) 3

If in a triangle ABC, a=5, b=4 and cos(A-B) = 31/32 then the third side c is equal to
A) 2

69.

2 /3

In a triangle ABC, the length of the bisector of angle A is

A)

68.

D)

Let A 0 A1A 2A 3 A 4 A 5 be a regular hexagon inscribed in a circle of unit radius. Then the product of
the lengths of the line segments A 0 A1 , A0A 2 and A0A 4 is
A)

67.

C) 1/ 3

B) right angled

C) obtuse angled

D) acute angled

IIT- MATHS

SECTION - B
MULTIPLE ANSWER TYPE QUESTIONS
1.

If in a triangle ABC, B = 60 then


A) (a-b)2 = c2 - ab

2.

1
b cosec a
2

B) 15cm

If ABC : a = 5, b = 4, A =

B) tan-1 (9/40)
D) 2tan-1 (1/9)

E) none of these

B) tanB = b/a
C) cos C = 0
2
2
2
E) sin A + sin B + sin C = 2

These exists a triangle ABC satisfying

D) sinA + sinB =

B)

sin A sin B sin C

2
2
7

2 (sinA + cosA ) =

3 +1
3
, cosA cosB =
= sinA sinB
2
4

a c
AC
In a DABC, 2 cos 2
. Then
2
( a c2 ac
A) B = /3

B) B = C

C) A, B, C are in A.P.

D) B + C = A

In a DABC, tan C < 0. Then


A) tanA tan B < 1
C) tanA + tanB + tanC < 0

9.

D) 5 3 1 cm

If tanA, tanB are the roots of the quadratic abx2 - c2x + ab = 0, where a, b, c are the sides of a
triangle, then

C) (a+b)2 = c2 + ab and

8.

+ B for the value of angle C


2

A) tanA + tanB + tanC = 0

7.

D) OI = 2 sin cos / 2

C) 5 3 1 cm

A) tanA = a/b
D) tan A + tan B = c2/ab
6.

b cos3 / 2

C) r = 21 cos

In DABC, A = 150, b = 10 2 cm the value of a for which these will be a unique triangle meeting
theese requirement is

A) can not be evaluated


C) tan-1 (1/40)
5.

D) a2+b2+c2 = 2b2 + ac

b sin 2

B) D = 2b2 sin 2a

A) 10 2 cm
4.

C) (c-a)2 = b2 - ac

Given an isosceles triangle with equal sides of length b, base angle < /4, r the radii and O, I the
centres of the circumcircle and incircle, respectively. Then
A) R =

3.

B) (b-c)2 = a2 - bc

B) tanA tanB > 1


D) tanA + tanB + tanC > 0

If the sines of the angles A and B of a triangle ABC satisfy the equation c2x2-c(a+b)x+ab= 0, then
the triangle
228

PROPERTIES OF TRIANGLE
A) is acute - angled
C) is obtuse angled
10.

12.

B) a2 + b2 - ab < c2

3 < 0) then

C) a2+b2 > c2

D) none of these

For a triangle ABC, which of the following is true ?


A)

cos A cos B cos C

a
b
c

B)

C)

sin A sin B sin C


3

a
b
c
2R

D)

cos A cos B cos C a 2 b 2 c 2

a
b
c
2abc

sin 2A
a

sin 2B
b

sin 2C
c2

If H is the orthocentre of triangle ABC, then AH is equal to


2abc
cos A

If the angles of a triangle are in the ratio 2 : 3 : 7, then the sides opposite these angles are in the

A) 2R cosA
13.

3 x2-4x +

In a ABC tanA and tanB satisfy the inequation


A) a2+b2 + ab > c2

11.

B) is right-angled
D) satisfies sinA + cosA = (a+b)/c

B) 2R sinA

C) a cotA

D)

ratio
A)
14.

2 : 2: 3 1

B) 2 :

2 :

3 +1

C)

:1 :

3 1
2

D) 2

C) 2cos2

D) 2 sin2

If l is the medium from the vertex A to the side BC of a DABC, then


B) 4l2 = 2b2+2bc cosA
D) 4l2 = (2s-a)2-4bc sin2A/2

If in a ABC, r1 = 2r2 = 3r3, then


A) a/b = 4/5

B) a/b = 5/4

If two sides of a triangle are 12 and


third side is
A) 2 2 - 6

229

B) 2 cos2

A) 4l2 = 2b2+2c2-a2
C) 4l2 = a2+4bc cosA

19.

If sinb is the G.M. between sina and cosa, then cos2b is equal to

18.

3 1

D)

B) b sinA > a, A > /2


D) b sinA < a, A < /2, b > a

B) - 2

A) 2 sin2
17.

If cos (), cos, cos () are in H.P., then cosq sec /2 is equal to
A) -1

16.

2 :

These exists a triangle ABC satisfying the conditions


A) b sinA = a, A < /2
C) b sinA > a, A < /2

15.

C) 1 :

B)

2 6

C) a/c = 3/5

D) a/c = 5/3

8 ,the angle opposite to the shorter side is 450, then the

C)

6 2

D) none of these

IIT- MATHS

SECTION - A
SINGLE ANSWER TYPE QUESTIONS

1.

2.

3.

4.

5.

6.

7.

8.

9.

10.

11.

16.

17. 18. 19.

20.

21. 22. 23

24. 25.

26.

12. 13.

27. 28.

14. 15.

29. 30.

B,C A,C
D

A,B
D

31

32

33

34

35

36

37

38

39

40

41

42 43

44

45

AB
CD

A,B

46

47

48

49

50

51

52

53

54

55

56

57 58

59

60

61

62

63

64

65

66

67

68

69

70

71

230

PROPERTIES OF TRIANGLE

SECTION - B
MULTIPLE ANSWER TYPE QUESTIONS

1.

2.

3.

4.

CD ACD AD BD

16.

17. 18. 19.

AC

AB BC BC
CD

231

5.

6.

7.

8.

9.

10.

11.

12. 13.

14. 15.

ABC CD AC AC BD AB
DE

BC

AC ACD AD BC

IIT- MATHS

CO-ORDINATE GEOMETRY

232

CO-ORDINATE GEOMENTRY
The co-ordination of algebra and geometry is called co-ordinate geometry. Historically, co-ordinates were introduced to help geometry. And so well did they do this job, that the very identity of geometry was changed. The word geometry today generally means coordinate geometry.
In co-ordinate geometry all the properties of geometrical figures are studied with the help of
algebraic equations. Students should note that the object of coordinate geometry is to use some known
facts about a curve in order to obtain its equation and then deduce other properties of the curve from the
equation so obtained. For this purpose we require a co-ordinate system. There are various types of coordinate systems present in two dimension e.g. rectangular, oblique, polar, triangular system etc. Here we
will only discuss rectangular co-ordinate system in detail.

CARTESIAN CO-ORDINATES
Let XOX and YOY be two fixed straight lines at right angles. XOX is called axis of x and YOY
is called axis of y and O is named as origin.
From any point P a line is drawn parallel to OY. The directed line OM = x and MP = y. Here OM
is abscissa and MP is ordinate of the point P. The abscissa OM and the ordinate MP together written as
(x, y) are called co-ordinates of point P. Here (x, y) is an ordered pair of real numbers x and y, which
determine the position of point P.
Since XOX YOY, this system of representation is called rectangular (or orthogonal)
co-ordinate system.
When the axes of co-ordinates XOX and YOY are not at right angles, they are said to be oblique
axes.

Remarks :
Y
nd

st

II quadrant

I quadrant

X
O
th
IV quadrant
IIIrd quadrant

y
Lattice Point (w.r.t. co-ordinate geometry) : A point whose abscissa and ordinate both are integers.

233

IIT- MATHS

Distance between two points :


The distance between two points P(x1, y1) and Q(x2, y2 ) is given by

Q(x2, y2)

y
P
(x1,y1)

PQ =

y2

x1 x 2 2 y1 y 2 2

y1

x1
x2

SECTION FORMULA
1.

If P(x, y) divides the line joining A(x1, y1 ) & B(x2 , y2 ) in the ratio m : n, then
(i)

Internal division: x =
y=

(ii)

2.

my 2 ny1
mn

External division: x =
y=

mx 2 nx1
mn

mx 2 nx1
mn

my 2 ny1
mn

The coordinates of the mid-point of the line-segment joining (x1, y1) and (x2, y2) are
x1 x 2 y 1 y 2
,

2
2

Remarks:
If the ratio, in which a given line segment is divided, is to be determined, then sometimes, for
convenience (instead of taking the ratio m : n), we take the ratio : 1 and apply the formula for internal
division. If the value of turns out to be positive, it is an internal division otherwise it is an external
division.

Remarks:
Points P and Q are said to be harmonic conjugate of each other w.r.t. OA. Incentre and Excentre
of a triangle are harmonic conjugate of each other w.r.t. to the angle bisector on which they lie.

Centres connected with a Triangle :


(w.r.t. ABC, where A (x1, y1), B (x2, y2), C (x3, y3), BC = a, CA = b & AB = c).
Centroid :

The point of concurrency of the medians of a triangle is called the centroid of the
triangle. The centroid of a triangle divides each median in the ratio 2 : 1. The coordinates
of centroid are given by

234

CO-ORDINATE GEOMENTRY
x1 x 2 x 3 y1 y 2 y 3
,
.
G
3
3

Orthocentre : The point of concurrency of the altitudes of a triangle is called the orthocentre of
the triangle. The triangle formed by joining the feet of altitudes in a is called the
orthic triangle. Here DEF is the orthic triangle of ABC.
Incentre :

The point of concurrency of the internal bisectors of the angles of a triangle is called the
incentre of the triangle. The coordinates of the incentre are given by
ax 1 bx 2 cx 3 ay1 by 2 cy 3
,
.
I
abc
abc

Excentre :

Co-ordinate

of

excentre

opposit e

to

is

given

by

ax1 bx 2 cx 3 ay1 by 2 cy3


,
and similarly for excentres (I2 & I3) opposite to
I1
a bc
a bc

ax1 bx 2 cx 3 ay1 by 2 cy 3
,

B and C are given by I2


abc
a bc

A
c
B

b
L
C

I1

BL c
AI
bc
, also 1
LC b
I1L
a
ax1 bx 2 cx 3 ay1 by 2 cy 3
,

I3
abc
a bc

Circumcentre : The point of concurrency of the perpendicular bisectors of the sides of a triangle is
called circumcentre of the triangle.

Remarks :
1. Circumcentre O, Centroid G and Orthocentre H of a ABC are collinear. G Divides OH in
the ratio 1 : 2, i.e. OG : GH = 1 : 2
235

IIT- MATHS
2. In an isosceles triangle centroid, orthocenter, incentre and circumcentre lie on the same
line and in an equilateral triangle all these four points coincide.

Area of a triangle :
Let (x1, y1), (x2, y2) and (x3, y3) respectively be the coordinates of the vertices A, B, C of a triangle
ABC. Then the area of triangle ABC, is
1
[x (y y )+ x2 (y3 y1) + x3 (y1 y2)]
2 1 2 3

x1

y1 1

1 x2
2 x
3

y2 1
y3 1

.......(1)

.......(2)

While using formula (1) or (2), order of the points (x1, y1), (x2, y2) and (x3, y3) has not been taken
into account. If we plot the points A(x1, y1), B(x2, y2) and C(x3, y3), then the area of the triangle as obtained
by using formula (1) or (2) will be positive or negative as the point A, B, C are in anti-clockwise or
clockwise directions,

So, while finding the area of triangle ABC, we take modulus.

Remarks :
In case of polygon with vertices (x1, y1), (x2, y2), ....... (xn, yn) in order, then area of polygon is given
by

1
|(x1y2 y1x2) + (x2y3 y2x3) + .....+ (xn 1yn yn 1xn) ) + (xny1 ynx1)|
2

LOCUS
When a point moves in a plane under certain geometrical conditions, the point traces out a path.
This path of the moving point is called its locus.

Equation of locus
The equation to a locus is the relation which exists between the coordinates of any point on the
path, and which holds for no other point except those lying on the path. In other words equation to a curve
(or locus) is merely the equation connecting the x and the y coordinates of every point on the curve.

Procedure for finding the equation of the locus of a point :


(i) If we are finding the equation of the locus of a point P, assign coordinates (h, k) or (x1, y1)
to P.
(ii) Express the given conditions in terms of the known quantities to facilitate calculations.
We sometimes include some unknown quantities known as parameters.
236

CO-ORDINATE GEOMENTRY
(iii)Eliminate the parameter. So that the eliminant contains only h, k and known quantities. If
h and k coordinates of the moving point are obtained in terms of a third variable t called
the parameter, eliminate t to obtain the relation in h and k and simplify this relation.
(iv)Replace h by x, and k by y, in the eliminant. The resulting equation would be the equation
of the locus of P.

x=

0 3X
0 3Y
,y=
1 3
1 3

from which X =

4
4
x, Y = y .
3
3

Substitute these values, then the locus of P is

8
x + 4y + 4 = 0
3

2x + 3y + 3 = 0.

STRAIGHT LINE
Any equation of first degree of the form ax + by + c = 0, where a, b, c are constants always represents a straight line (at least one out of a and b is non zero)

Slope
If a straight line makes an angle in anticlockwise direction with the positive direction of x-axis,
0 < 180, 90, then the slope of the line, denoted by m is tan. i.e. m = tan`.
If A(x1, y1 ) and B(x2, y2 ), x1 x2 are any two points, then slope of the line passing through
y 2 y1

A and B is given by m = x x .
2
1
Remark :
(i)

If 900 , m does not exist and line is parallel to y - axis.

(ii)

If = 0, m = 0 and the line is parallel to x-axis.


Let m1 and m2 be slopes of two given lines.
(a) If lines are parallel, m1 = m2 and vice versa.
(b) if lines are perpendicular, m1.m2 = -1 and vice versa.

(iii)

Position of a given point relative to a given line :


The fig. Shows a point P(x1, y1) lying above a given line. If an ordinate is dropped from P to meet
the line L at N, then the x coordinate of N will be x1.
Putting x = x1 in the equation ax + by + c = 0 gives
y coordinate of

237

N=

(ax1 c)
b

IIT- MATHS

If P(x1, y1) lies above the line, then we have


y1 >

(ax1 c)
b

i.e. y1 +

i.e.

(ax1 by1 c)
>0
b

i.e.

L ( x 1 , y1 )
> 0 .......(1)
b

(ax1 c)
>0
b

Hence, if P(x1, y1) satisfies equation (1), it would mean that P lies above the line ax + by + c = 0,
and if

L ( x 1 , y1 )
< 0, it would mean that P lies below the line ax + by + c = 0.
b

Remark :
If (ax1 + by1 + c) and (ax2 + by2 + c) have same signs, it implies that (x1, y1) and (x2, y2) both lie on
the same side of the line ax + by + c = 0. If the quantities ax1 + by1 + c and ax2 + by2 + c have opposite
signs, then they lie on the opposite sides of the line.

Intercept of a straight line on the axis :


If a line AB cuts the x-axis and y-axis at A and B respectively and O be the origin then OA and OB
with proper sign are called the intercepts of the line AB on x and y axes respectively.

Standard equations of straight lines :


1 Slope-intercept form :
y = mx + c,
where m = slope of the line = tanq
c = y intercept

2 Intercept form :
x/a + y/b = 1

y
(0, b)

x y
1
a b

x intercept = a, length of x intercept = |a|


x

y intercept = b, length of y intercept = |b|

(0, 0)

(a, 0)

238

CO-ORDINATE GEOMENTRY

3 Normal form :
x cos + y sin = p, where , is the angle which the perpendicular to the line makes with the axis
of x and p is the length of the perpendicular from the origin to the line. 0 2 and p is always positive.

(0,0)

p+
ve

4.4.4 Slope point form :


Equation : y y1 = m(x x1), where
(a) One point on the straight line is (x1, y1) and
(b) The direction of the straight line i.e., the slope of the line = m

4.4.5 Two point form :


y 2 y1
Equation : y y1 = x x (x x1), where (x1, y1) and (x2, y2) are the two given points. Here
2
1
y 2 y1
m= x x .
2
1

Parametric equations of a straight line :


In figure given below let BAP be a straight line through a given point A (x1, y1), the angle of slope
being q. The positive direction of the line is in the sense BAP. (Direction of increasing ordinate is called
the positive direction of the line).
For the points P (x,y) and Q (X, Y) Shown in the figure AP is regarded as a positive vector and AQ
as a negative vector, as indicated by the arrows.
From the general definitions of cosq and sinq we have
cosq =

x x1
y y1
, sinq =
AP
AP

or x x1 = AP cosq, y y1 = AP sinq.

x x1 y y1

r
cos
sin

239

IIT- MATHS
or r (4 +

21
) + 71 = 0
5

or r =

355
. The distance between A and P is thus 355/41 units,
41

the vector AP being in the negative direction of the line.

Length of the perpendicular from a point on a line :


The length of the perpendicular from P(x1, y1) on ax + by + c = 0 is

ax1 by1 c

a 2 b2
c

The length of the perpendicular from origin on ax + by + c = 0 is

a b2

The distance between two parallel lines :


The distance between two parallel lines ax + by + c1 = 0 and ax + by + c2 = 0 is

| c1 c 2 |
a 2 b2

Reflection of a point about a line :


The image of a point (x1, y1) about the line ax + by + c = 0 is
x x1
y y1
ax by c
=
= 2 1 2 12
a
b
a b

and the foot of perpendicular from a point (x1, y1) on the line ax + by + c = 0 is
x x1
y y1
ax by c
=
= 1 2 12 .
a
b
a b

FAMILY OF LINES:
(Equation of any straight line through the point of intersecton of two given straight lines).
The equation of any straight line passing through the intersection of the two lines
ax + by + c = 0, Ax + By + C = 0
has the general form
ax + by + c + (Ax + By + C) = 0
In which can have any real value ; here, is parameter which can be evaluated specifically if
some further condition is imposed.
Hence the general equation of the family of lines through the point of intersection of two given
lines is L + L = 0 where L = 0 and L = 0 are the two given lines, and is a parameter.
Conversely, any line of the form L1 + L2 = 0 passes through a fixed point which is the point of
intersection of the lines L1 = 0 and L2 = 0. In other words if a linear expression L contains an unknown
coefficient, then the line L = 0 can not be a fixed line. Rather it represents a family of straight lines.

Remarks :
1. If L1 = 0 and L2 = 0 are parallel lines, they will meet at infinity.
240

CO-ORDINATE GEOMENTRY
2. The family of lines perpendicular to a given line ax + by + c = 0 is given by
bx - ay + k = 0, where k is a parameter.
3. The family of lines parallel to a given line ax + by + c = 0 is given by ax + by + k = 0, where
k is a parameter.

CONCURRENCY OF STRAIGHT LINES :


The condition for three lines a1x + b1y + c1 = 0, a2x + b2y + c2 = 0, a3x + b3y + c3 = 0 to be
concurrent is -

(i)

a1

b1

c1

a2
a3

b2
b3

c2
= 0.
c3

(ii) There exist three constants , m, n (not all zero the same time) such thatL1 + mL2 + nL3
= 0, where L1 = 0, L2 = 0 and L3 = 0 are the three given straight lines.
(iii) The three lines are concurrent if any one of the lines passes through the point of
intersection of the other two lines.

THE ANGLE BETWEEN TWO STRAIGHT LINES :


In fig. given below, GAH and LAM are two straight lines meeting the
Y

M
H
A

1
O

x-axis at G and L and intersecting at A. The angles of slope are 1 and 2; the corresponding
gradients are given by
Let,

m1 = tan 1 , m2 = tanq2

= 2 1

.......(1)

........(2)

Thus, f is the angle through which GA has to be rotated about G in the counter-clockwise direction
to be parallel to, and in the same sense as, LA.
From (2), tan = tan(2 1).

tan 2 tan 1
= 1 tan tan ,
2
1

m 2 m1
or, by means of (1), tan = 1 m m
1 2

.........(3)

This is the formula required ; from it we can calculate from the given-or deducible-values of the
gradients of the two given lines.
In numerical examples the value of the right-hand side of (3) may be positive or negative ; if the
value is positive, the angle is acute ; if the value is negative, the angle is obtuse..
241

IIT- MATHS
It is a convention to tell acute angle for the angle between the two lines. For this purpose tan

m 2 m1
= 1 m m , where is the acute angle.
1 2
Remarks :
1. If the lines are parallel then 2 = 1 and, by (2), = 0 that tan = 0; thus, from (3), m2 = m1,
which is otherwise obvious from (1).
1 m1m 2
2. If the lines are parallel then = 90 so that cot = 0 ; from (3), cot = m m and it
2
1

follows that, since m1 and m2 are unequal, then 1 + m1m2 = 0 or m1m2 = 1, which is the
condition that the two lines should be perpendicular.

BISECTORS OF THE ANGLES BETWEEN TWO GIVEN LINES :


Angle bisector is the locus of a point which moves in such a way so that its distance from two
intersecting lines remains same.
The equations of the two bisectors of the angles between the lines a1x + b1y + c1 = 0 and
a 1x b1 y c1

a2x + b2y + c2 = 0 are

a12 b12

a 2 x b2 y c2

a 22 b 22

If the two given lines are not perpendicular i.e. a1 a2 + b1b2 0 and not parallel i.e. a1 b2 a2b1
then one of these equations is the equation of the bisector of the acute angle between two given lines and
the other that of the obtuse angle between two given lines.

Remarks:
Whether both given lines are perpendicular or not but the angular bisectors of these lines will
always be mutually perpendicular.

The bisectors of the acute and the obtuse angles


Take one of the lines and let its slope be m1 and take one of the bisectors and let its slope be m2. If

m1 m 2
be the acute angle between them, then find tan = 1 m m
1 2

242

CO-ORDINATE GEOMENTRY
If tan > 1 then the bisector taken is the bisector of the obtuse angle and the other one will be the
bisector of the acute angle.
If 0 < tan < 1 then the bisector taken is the bisector of the acute angle and the other one will be the
bisector of the obtuse angles.
If two lines are a1 x + b1y + c1 = 0 and a2x + b2y + c2 = 0, then
a 1x b1 y c1
a12 b12

C
N

a 2x b2 y c2
A

a 22 b 22

P(x, y)

M
B

will represent the equation of the bisector of the acute or obtuse angle between the lines according
as a1a2 + b1b2 is negative or positive.

The equation of the bisector of the angle which contains a given point :
The equation of the bisector of the angle between the two lines containing the point (a,) is
a 1x b1 y c1
2
1

2
1

a b

a 2 x b2 y c2
2
2

a b

2
2

or

a 1x b1 y c1
2
1

2
1

a b

a 2 x b2 y c2
a 22 b 22

according as a1 + b1 + c1 and a2

+ b2 + c2 are of the same signs or of opposite signs.


For example the equation of the bisector of the angle containing the origin is given by
a 1x b1 y c1
2
1

2
1

a b

a 2 x b2 y c2

=+

a 22 b 22

for same sign of c1 and c2 (for opposite sign take ve sign in

place of +ve sign)

Remarks:
(i)

If c1c2 (a1a2 + b1b2) < 0, then the origin will lie in the acute angle and if c1c2 (a1a2 + b1b2) >
0, then origin will lie inthe obtuse angle.

(ii)

Equation of straight lines passing through P(x1, y1) and equally inclined with the lines
a1x + b1y + c1= 0 and a2x + b2y + c2 = 0 are those which are parallel to the bisectors between
these two lines and passing through the point P.

The equation of reflected ray :


Let L1 a1x + b1y + c1 = 0 be the incident ray in the line mirror L2 a2x + b2y + c2 = 0.
Let L3 be the reflected ray from the line L2. Clearly L2 will be one of the bisectors of the angles
between L1 and L3. Since L3 passes through A, so
L3 L1 + L2 = 0
Let (h, k) be a point on L2. Then,

243

IIT- MATHS
| a 1h b1k c1 |
2
1

2
1

a b

| a 1h b1k c1 (a 2 h b 2 k c 2 ) |
(a 1 a 2 ) 2 (b1 b 2 ) 2

Since (h, k) lies on L2, a2h + b2k + c2 = 0

a12 + a22 2 + 2a1a2 + b12 + b222 + 2b1b2 = a12 + b12

= 0 or =

2(a1a 2 b1b 2 )
a 22 b 22

But = 0 given L3 = L1.


Hence L3 L1

2(a 1a 2 b1b 2 )
L2 = 0.
a 22 b 22

Remarks :
Some times the reflected ray L3 is also called the mirror image of L1 in L2.

ROTATION OF THE AXES


(To change the direction of the axes of co-ordinates, without changing the origin, both systems of
co-ordinates being rectangular.)
Let OX, OY be given rectangular axes with respect to which the coordinates of a point P are
(x, y). Suppose that OU, OV are the two perpendicular lines obtained by rotating OX, OY respectively
through an angle a in the counter-clockwise sense. We take OU, OV as a new pair of coordinate axes, with
respect to which the coordinates of P are (x, y), then
x x cos y sin
y x sin y cos

x cos sin
y = sin cos

or,

x
y

(in matrix form)

PAIR OF STRAIGHT LINES


The general equation of degree ax2 + 2hxy + by2 + 2gx + 2fy + c = 0 represents a pair of straight
a

h g

lines if h b f 0
g

abc + 2fgh af 2 bg2 ch2 = 0 and h2 ab.


The homogeneous second degree equation ax2 + 2hxy + by2 = 0 represents a pair of straight lines
through the origin
If lines through the origin whose joint equation is ax2 + 2hxy + by2 = 0, are y = m1x and y = m2x,
then y2 (m1 + m2)xy + m1m2x2 = 0 and y2 +

2h
a
xy + x2 = 0 are identical. If is the angle between the
b
b

244

CO-ORDINATE GEOMENTRY

m1 m 2 2 4m1m 2

two lines, then tan =

1 m1m 2

2 h 2 ab
ab

The lines are perpendicular if a + b = 0 and coincident if h2 = ab.


Joint Equation of Pair of Lines Joining the Origin and the Points of Intersection of a Line and a
Curve
A

If the lines x + my + n = 0, ((n 0) i.e. the line not passing through origin) cuts the curve
ax + 2hxy + by2 + 2gx + 2fy + c = 0 at two points A and B, then the joint equation of straight lines passing
through A and B and the origin is given by homogenizing the equation of the curve by the equation of the
line i.e.
2

x my x my
ax + 2hxy + by + (2gx + 2fy)
c
0 is the equation of the lines
n n
2

OA and OB

245

IIT- MATHS

WORKED OUT ILLUSTRATIONS


ILLUSTRATION : 01
If , , are real roots of the equation x 3 3px 2 3qx 1 0 . Find the centroid of the triangle

1 1

whose vertices are , , and


Solution :
Since , , are the roots of the equation
x 3 3px 2 3qx 1 0

+ + = 3p, + + = 3q, = 1

Let A = , , B , and C ,
Let G(x,y) the centroid of ABC, then
X=

3p

p
3
3

1 1 1

3q
And y

q
3

Hence co-ordinates of the centroid of ABC are (p,q)


ILLUSTRATION : 02
The four points A (,0), B (,0), C (,0) and D( ,0 ) are such that , are the roots of equation
ax 2 2 hx b 0 and , are those of equation a' x 2 2 h' x b' = 0. Show that the sum of the
ratios in which C and D divide AB is zero if ab + ab = 2hh.
Solution :
Since , are the roots of x 2 2hx b 0
+ =

2h
b
and
a
a

. (1)

and , are the roots of a ' x 2 2h ' x b' 0


then

2h '
b'
and
a'
a'

...(2)

Let C divides AB in the ratio : 1

246

CO-ORDINATE GEOMENTRY
1.
Then
1

and let D divides AB in the ratio : 1


then

. 1.
1

but given + = 0

2 2 0

2h 2h ' 2h 2b '

0z

a'
a a ' a

or

ab + ab = 2hh

ILLUSTRATION : 03
Find all points on x + y = 4 that lie at a unit distance from the line 4x + 3y 10 = 0.
Solution :
Let x = t, then y = 4-t. Let P (t, 4 t) be an arbitrary point on the line x +y = 4
Distance of P from 4x + 3y 10 = 0 is unity

4t 3 4 t 10 |
42 32

|t + 2| = 5

t+2 5

or

t = -2 5

t = 3, -7

points are (3,1) & (-7,11)

247

IIT- MATHS
ILLUSTRATION : 04
Find a point P on the line 3x + 2y + 10 = 0 such that |PA - PB| is maximum where A is (4,2) and
B is (2,4)
Solution :
Let P be x1 , y1
And APB
2

PA PB AB

Then cos =

2 PA.PB

Since cos 1
2

PA PB AB

2 PA.PB
2

PA PB AB 2 PA.PB
2

PA PB AB

PA PB AB
| PA PB | 2 2
Maximum value of |PA - PB| is 2 2 when = 0 i.e, P lies on the line AB as well as on
the given line.
equation of AB is
y2

42
x 4
24

y 2 = -x + 4

x+y=6

and given line 3x +2y + 10 = 0

(1)
.(2)

Solving (1) and (2), we get P (-22, 28)


ILLUSTRATION : 05
Find the co-ordinates of the orthocentre of the triangle formed by the lines y = 0, (1+t)x ty +
t (1+t) = 0 and (1 +u) x uy + u (1+u) = 0 t 0 , and show that for all values of t and u, the
orthocentre lies on the line x +y = 0.
Solution :
Let equations of BC, CA and AB are y=0, (1+t)xty+t (1+t)=0 and (1+u)xuy+u (1+u) = 0
respectively.
Let (h,k) be the orthocentre then
Slope of OB x slope of AC = -1

248

CO-ORDINATE GEOMENTRY
k 0 1 t

1
hu
t

k (1+t) = -th tu

k=-

th tu k tk th tu
1 t

. (1)

and slope of OC x slope of AB = -1


k0

h t
k

1 u 1
u

u h t
k ku uh ut
1 u

(2)

Subtracting (2) from (1) , we get


K (t -u) = -h (t u) h = k = 0
locus of orthocentre is x + y = 0
Putting h = -k in (1)
Then we get k = -t u
h = tu
then orthocentre is (t,u tu)
ILLUSTRATION : 06
If m 1 and m 2 are the roots of the equation x 2

32x

3 1 0 . Show that the area of

33 11 2
c
the triangle formed by the liens y = m1x, y m 2 x and y c is
4

Solution
Since m 1 and m 2 are the roots of the equation
x2

32x

3 1 0

then m1 m 2 3 2

249

m1 m 2

3 4 4

11

3 1 0

m1 m 2 2 4m1m 2
34 34

IIT- MATHS
and co-ordinates of the vertices of the given triangle are (0,0), (c/m1,c) and (c/m2,c).
Hence the required area of triangle

0
1 c
2 m1
c
m2

0 1
c 1
c 1

1 2 1
1
c

2 m1 m 2

1 2 m 2 m1
c
2
m 1m 2

1 2
c
2

1 2
c .
2

11

3 1

33 11 2
c }

4
3 1

11 3 1

3 1

ILLUSTRATION : 07
One side of a rectangle lies on the line 4x + 7y + 5 = 0. Two of its vertices are (-3,1) and (1,1). Find
the equations of other three sides
Solution :
Since (-3,1) lies on 4x + 7y + 5 = 0
And (1,1) does not lie on 4x + 7y + 5 = 0
Equation of AD, which is to 4x +7y + 5 = 0
And passing through (-3,1) is
7x 4y + = 0

- 21 4 + = 0

= 25
Therefore, equation of AD is 7x 4y + 25 = 0
(1,1) does not lie on AD

Co-ordinate of C is (1,1)
Equation of BC which is parallel to AD and Passing through (1,1) is
7x 4y + = 0
250

CO-ORDINATE GEOMENTRY

7x 1 4 x 1 + = 0
= -3

equation of AD is 7x 4y 3 = 0
Equation of DC which is parallel to AB passing through (1,1) is
4x + 7y + = 0
4 x 1 + 7 + 1 + = 0 = -11

Therefore equation of DC is 4x + 7y 11 = 0
ILLUSTRATION : 08
A line through the variable point A (k+1, 2k) meets the lines 7x +y 16 = 0, 5x y 8 = 0 at B,C,D
respectively. Prove that AC, AB, AD are in H.P.
Solution :
Given lines are
7x y 16 = 0

(1)

5x y 8 = 0

(2)

x 5y +8 = 0

(3)

Let the equation of line passing through A (k+1, 2k) making an angle with the + v e
direction of x axis be
x k 1 y 2k

r1 , r2 , r3
cos
sin

(if AB = r1 , r2 , r3 )
(if AB = r1 , AC = r2 , AD = r3 )

B [(k+1) + cos , 2k + sin ]


C [(k+1) + cos , 2k + sin ]
D [(k+1) + cos , 2k + sin ]

Points B, C, D satisfying (1), (2) and (3) respectively


Then r1
r2 =

r3

251

91 k
7 cos sin

31 k
5 cos sin

91 k
5 sin cos

IIT- MATHS

1 1
5 cos sin 5 sin cos

r2 r3
31 k
91 k

15 cos 3 sin 5 sin cos


91 k

14 cos 2 sin
91 k

= r
1

Hence r2 , r1 , r3 are in H.P.


ILLUSTRATION : 09
The three sides of a triangle are L r x cos r y sin r p r 0 , where r = 1,2,3. Show that the
orthocentre is given by L1 cos 2 3 L 3 cos 3 1 L 3 cos 1 2
Solution :
The given lines are
L1 x cos 1 y sin 1 p1 0
L 2 x cos 2 y sin 2 p 2 0
L 3 x cos 3 y sin 3 p 3 0

Now equation of AD is
. (1)

L 2 L 3 0

(xcos 2 y sin 2 p 2 ) + (x cos 3 y sin 3 p 3 ) 0


(xcos 2 cos 3 ) + (x sin 2 sin 3 )p 2 p 3 0

cos

cos

2
3
Slope of AD = sin sin
2
3

cos

1
and slope of BC = sin
1

Since AD BC

Slope of BC x slope of AD = -1

cos 2 cos 3
cos 1

1
sin 2 sin 3
sin 1

cos 1 cos 2 cos 3 cos 1 sin 1 sin 2 sin 3 sin 1

cos 1 2 cos 3 1 0

cos1 2
cos3 1

Now from (1),


252

CO-ORDINATE GEOMENTRY
L2

cos 1 2
L3 0
cos 3 1

L 2 cos 3 1 L 3 cos 1 2

. .....(2)

Similarly, we can obtain equation of altitude BE as


L 3 cos1 2 L1 cos 2 3

..(3)

From (2) and (3), we get


L1 cos 2 3 L 2 cos 3 1 L 3 cos 1 2

ILLUSTRATION : 10
A (3,0) and B (6,0) are two fixed points and U (, ) is a variable point on the plane. AU and
BU meet the y axis at C and D respectively and AD meets OU at V. Prove that CV passes
through (2,0) for any position of U in the plane.
Solution :
The equation of BU is
Y-=

0
x
6

So that the coordinates of D are 0,

Similarly the coordinates of C are 0,

Now, the equation of AD is


x 6

y 1
3
6

and the equation of OU is x = y


Solving (1) and (2), we get
x

6
6
,y
6
6

6
6
,

Hence coordinates of V are


6 6
Then the equation of CV is
6
3

3
y
6 3 x 0
6
3
0
6

253

IIT- MATHS

3
9

x
3 63

y=

3 x
1
3 2

Which pass through the point (2,0) for all values of (, ).


ILLUSTRATION : 11
A variable line is drawn through O to cut two fixed straight lines L 1 and L 2 in R and S. A
point S is chosen on the variable line such that

mn
m
n

. Show that the locus P is a


OP
OR OS

straight line passing through the point of intersection L 1 and L 2 .


Solution :
Let the equation of the variable line through O be

x
y

and let
cos sin

OR r1 .OS r2 and OP r3

Then co-ordinates of R, S and P are


R r1 cos , r1 sin , Sr2 cos , r2 sin , P r3 cos , r3 cos .

R lies on and lies on L 1 and S lies on L 2


r1 sin c and ar2 cos br2 sin 1
r1

c
1
and r2
sin
a cos b sin

From the given condition


mn m n

r3
r1 r2

From the given condition


mn m n

r3
r1 r2

mn m n

r3
r1 r2

mn
m sin
n a cos b sin
=
r3
c

(1)

Let co-ordinates of P be (h,k) then


h = r3 cos , k r3 sin
from (1),

mn

mr3 sin
n ar3 cos br3 sin
c

mn

mk
n ah bk
c

locus of P is
254

CO-ORDINATE GEOMENTRY
n ax by

my
m n
c

y
n ax by 1 m 1 0
c

ax by 1 m y c 0
nc

L1

m
L2 0
nc
m

where

nc

L1 L 2 0

Locus of P is point of intersection of L 1 and L 2

255

IIT- MATHS

SECTION A
SINGLE ANSWER TYPE QUESTION
1.

If A and B are two points having co-ordinates (3,4) and (5,-2) respectively and P is a point such
that PA = PB and area of triangle PAB = 10 sq units then co-ordinates of P are
A) (7,4) or (13,2)

2.

B) (7,2) or (1,0)

C) (2,7) or (4,13)

D) (1,2) or (2,1)

The position of a moving point in the x y plane at time is given by (ucosa.t, u sina. t -

1 2
9t )
2

where u, a, g are constants. The locus of the moving point is


A) a circle
3.

B) an ellipse

B) x + 2y = 5

D) x + 2y = 1

C) (-2,-2)

D) (1,1)

B) circle

C) pair of st lines

D) parabola

The equations of the lines through (-1,-1) and making an angle 45 with the line x + y = 0 are
given by
B) xy + x y y2 = 0
D) xy + x +y +1 = 0

Let the algebraic sum of the perpendicular distances from the points (2,0), (0,2) and (1,1) to a
variable line is zero, then the line passing through a fixed point whose co-ordinates are
A) (1,2)

10.

C) 2x + y = 5

B) (2,2)

A) x2 xy + x y = 0
C) xy +x + y = 0
9.

B) xsin + ycos = 2a sin2


D) xsin -y cos =2a sin2

The locus of a point P which divides the line joining (1,0) and (2 cos, 2sin) internally in the
ratio 2 : 3 for all is a
A) straight line

8.

D) hyperbola

One vertex of an equilateral triangle with centroid at the origin and one side as x+y2=0 is
A) (-1, -1)

7.

C) a parabola

The equation of the line passing through the intersection of the line x 3y + 1 = 0 and 2x + 5y
9 = 0 and 0 at distance 5 from the origin is
A) 2x y = 5

6.

D) hyperbola

The equation of a line which passes through (acos3, a sin3) and perpendicular to the line xsec
+ ycosec = a
A) xcos - ysin =2a cos2
C) xsin + ycos = 2a cos2

5.

C) an ellipse

If A and B are fixed point then the locus of a point which moves in such a way that the angle APB
is a right angle is
A) a circle

4.

B) a parabola

B) (1,1)

C) (2,1)

The product of the perpendiculars from the points

D) (2,2)

5 ,0 5 ,0

to the straight line

2x cos - 3y sin = 6
A) 5

B) independent of q

C) cos2q

D) 7sin2q
256

CO-ORDINATE GEOMENTRY
11. Circum center of the triangle whose vertices are (2,-1) (3,2) and (0,3) is
A) (1,-1)
12.

B) (-1,1)

B) - 54

B) 2

C) 2

B) 0

B) 2

B) 5 units

C) 1

C) triangle not possible

B) 2x + y 7 = 0; 3x 5y + 2 = 0
D) 2x y + 5 = 0

B) equilateral

C) right angled

D) none of these

B) 3 : 4

C) 2 : 1

D) 4 : 3

B) 0

C) 4

D) 3

Let PS be the median of the triangle with vertices P(2,2) Q(6,-1) R (7,3). The equation of the line
passing through (1, -1) and parallel PS is
A) 2x 9y 7 = 0
C) 2x + 9y 11 = 0

257

5 are

The number of integer values of m for which the x coordinate of the point of intersection of the
lines 3x + 4y = 9 and y = mx + 1 is also an integer is
A) 2

23.

D) 10 units

A straight line through the origin 0 meets the parallel lines 4x + 2y = 9 and 2x + y +6 = 0 at points
P and Q respectively then the point O divides the segment PQ in the ratio
A) 1 : 2

22.

D)

The straight line x + y = 0, 3x + y 4 = 0, x + 3y 4 = 0 form a triangle which is


A) isosceles

21.

D) 1

The equation of the line through the point of intersection of the lines x 3y + 1 = 0 and

A) 3x + 2y 7 = 0, 5x 7y + 12 = 0
C) 2x + y 5 = 0

20.

D) 1

C)

2x + 5y 9 = 0 and whose distance from the origin is

19.

D) 24

If co-ordinates of the vertices B and C of a ABC are (0,0) and (5,0) respectively and co-ordinates of incentre of ABC is (3,4) then length of the side AC is
A) 25 units

18.

C) a and b both true

Number of lines passing through (3,4) and whose difference of the intercept is 2
A) 4

17.

D) 1 : 2 externally

Let equation of the BC of a ABC is 2x + 3y 1 = 0 co-ordinates of the incentre and circum


centre of ABC are (2,1) and (-1,3) respectively then value of cosB + cosC is
A) 1

16.

C) 2 : 1 externally

2
2
If the points at 1 2at 1 at 2 at 2 and (a,0) are collinear then value of t 1t 2 is

A) 1
15.

B) 1 : 1 externally

Distance between the lines 5x + 12y 1 = 0 and 10x + 24y + k = 0 is 2 then the value of K is
A) 50

14.

D) (-1,-1)

Ratio in which the join of (2,1) and (-1,2) is divided by the line x + 3y + 5 = 0
A) 1 : 1 internally

13.

C) (1,1)

B) 2x 9y 11 = 0
D) 2x + 9y + 7 = 0

The orthocentre of the triangle formed by the lines xy = 0 and x +y = 1 is

IIT- MATHS
1 1
2 2

1 1
3 3

A) ,
24.

B) ,

1 1
4 4

D) ,

C) (0,0)

The area of the triangle formed by joining the origin to the points of intersection of the line
5x 2y 3 5 and circle x2 + y2 = 10 is

A) 6
25.

B) 5

B) 1/2

C) 2

D) 3

The vertices of a triangle are A(-1,-7) , B (5,1) and C(1,4). The equation of the bisector of ABC
is
A) x 7y + 2 = 0

27.

D) 3

The area bounded by the curves x + 2|y| = 1 and x = 0


A) 1/3

26.

C) 4

B) x + 7y 2 = 0

C) x + 6y + 2 = 0

D) x 7y 2 = 0

If a, c, b are in G.P. then the line ax + by + c = 0


A) has a fixed direction
B) always passes through a fixed point
C) form a triangle with the axes whose area is constant
D) always cuts intercepts on the axes such that their sum is zero

28.

Area of the triangle formed by the lines y2 9xy +18x2 = 0 and y = 9 is


A) 27/4

29.

30.

B) 0

A) a 2 b 2 p 2 q 2

B) a 2 b 2 p 2 q 2

C) a 2 p 2 b 2 q 2

D) a 2 p 2 b 2 q 2

Two points A and B move on the xaxis and the yaxis respectively such that the distance between the two points is always the same. The locus of the middle point of AB is
B) a circle

C) a parabola

D) an ellipse

The range of values of the ordinate of a point moving on the line x=1, and always remaining in
the interior of the triangle formed by the lines y=x, the xaxis and x+y=4, is
A)(0,1)

32.

D) 27

Line L has intercepts a and b on the coordinate axes, when the axes are rotated through a given
angle; keeping the origin fixed, the same line has intercepts p and q, then

A) a straight line
31.

C) 9/3

B)(0,1)

C)(0,4)

D) None of these

Let A=(1,0) and B(2,1). The line AB turns about A through an angle /6 in the clockwise sense,
and the new position of B is B. The B has the coordinates.
3 3 3 1
A) 2 , 2

3 3 3 1

,
2
2

B)

1 3 1 3

2
2

C)

D) None of these

258

CO-ORDINATE GEOMENTRY
33.

The bisector of the acute angle formed between the lines 4x3y+7=0 and 3x4y+14=0 has the
equation
A) x+y7=0

34.

B)xy+3=0

B) (2,1)

B) =3

3 x+y=4

5
2

5 1
2 2

1 5
2 2

B) ,

C) ,

B) x2 = 9y2

D) None of these

D) None of these

C) x2 9y2 = 0

D) y2 4x2 = 0

The point P (1,1) is translated parallel to y = 2x in the first quadrant through a unit distance. The
co-ordinates of the new position of P are

,1

B) 1

,1

C)

D)

If P and P be the perpendiculars from the origin upon straight lines xsec + ysec = a and xcos
- y sin = a cos2 respectively, then the value of expression 4p2 +P2
B) 3a2

C) 2a2

D) 4a2

The line 3x + 2y = 24 meets y axis at A and x axis at B. The perpendicular bisector of AB


meets the line through (0,-1) parallel to x axis at C. Then area of D ABC is
A) 182 sq units

259

C) x+ 3 y=4

Let O be the origin and A, B be the two points having co-ordinates (0,4) and (6,0) respectively. If
a point P moves in such a way that the area of the OPA is always twice the area of POB then
P lies on

A) a2

45.

D) (a+b, b)

If P(1+t/ 2 ,2+t/) be any point on a line then the range of values of t for which the point P lies
between the parallel lines x+2y=1 and 2x+4y=15 is

A) 1

44.

D) =4

C) (0,a)

B) x+ 3 y+4=0

A) y2 = 9x2

43.

The coordinates of two consecutive vertices A and B of a regular hexagon ABCDEF are (1,0) and
(2,0) respectively. The equation of the diagonal CE is

3
2

42.

C) =4

B) (a,0)

A) ,
41.

t m
2lm

If (a,b) be an end of a diagonal of a square and the other diagonal has the equation xy=a then
another vertex of the square can be

A)
39.

1
D) tan

If the lines x - 2y - 6 = 0, 3x + y - 4 = 0 and lx + 4y + l2 = 0 are concurrent, then

A) (ab,a)
38.

D) None of these

2
2
1 t m
tan
C)
t 2 m2

B) /2

A) =2
37.

C) (0,4)

The diagonals of the parallelogram whose sides are lx+my+n=0, lx+my+ n =0, mx+ly+n=0, include an angle
A) /3

36.

D) x=2y12=0

The equations of the three sides of a triangle are x=2, y+1=0 and x+2y=4. The coordinates of the
circumcentre of the triangle are
A) (4,0)

35.

C) 3x+y11=0

B) 91 sq units

C) 48 sq units

D) 100 sq units

The line PQ whose equation is x y = 2 cuts the x axis at P and Q is (4,2). The line PQ is rotated
about P through 45 in the anticlockwise direction. The equation of the line PQ in the new posi-

IIT- MATHS
tion is
A) y = 2
46.

B) y = 2

C) x = 2

D) x = -2

I f the co-ordinates of the vertex A of a ABC is (1,2) and equation of the perpendicular bisectors

of AB and AC are 3x + 4y 1 = 0 and 4x + 3y 5 = 0 then the equation of median AD is

47.

A) 11x 10 y + 9 = 0

B) 10 x 11y + 12 = 0

C) 3x + 4y 11 = 0

D) 3x + 4y + 12 = 0

Let the equation of the side BC of ABC is x + y + 2 = 0. If co-ordinates of its orthocentre and
circum centre are (1,1) and (2,0) respectively, then radius of the circum circle of ABC is
A) 3

48.

B) 10

C) 2 2

D)

Let co-ordinates of the vertices A and B of a triangle ABC are (6,0) and (0,6) respectively and co-

21 21
ordinates of its orthocentre is , then co-ordinates of its circum center is
4 4
13 13
,
16 16

11 11
,
16 16

A)
49.

B)

2 2
,2 2
3

B) 2 3 x y 1

54.

2 and

D) 1 and

C) y 2 3 x 2

D) x y 4

B) y 2 = 0, 4x 3y = 6
D) none of these

Let P (-1,0) Q (0,0) and R (3, 3 3 ) be three pts then the equation of the bisector of the angle PQR
is
A)

53.

C)

The equation of the lines through the point (2,3) and making an intercept of length 2 units between the lines y +2x =3 and y + 2x = 5 are
A) x + 3 = 0, 3x + 4y = 12
C) x 2 = 0, 3x +4y = 18

52.

31 31
,
8 8

D)

A ray of light traveling along the line x + y = 1 is incident on the x axis and after refraction is
incident on the x-axis and after refraction it enters the other side of the x-axis by turning /6 away
from the x-axis. The equation of the line along which the refracted ray travels is
A) x + 2 3 y 1

51.

15 15
,
16 16

C)

Let L be the lines 2x + y = 2. If the axes are rotated by 45 without transforming the origin, then
the intercepts made by the line L on the new axes are respectively
A) 1 and

50.

B)

3x
x y0
2

B) x 3y 0

C)

3x y 0

D) x

3
y0
2

Let PQR be a right angled isosceles triangle, right angled at P(2,1). If the equation of the line QR
is 2x +y = 3 then the equation representing the pair of lines PQ and PR is
A) 3x 2 3y 2 8xy 20 x 10y 25 0

B) 3x 2 3y 2 8xy 20x 10 y 25 0

C) 3x 2 3y 2 8xy 10x 15y 20 0

D) 3x 2 3y 2 8xy 10x 15y 20 0

In an isosceles triangle ABC, the coordinates of the points B and C on the base BC are respec260

CO-ORDINATE GEOMENTRY
tively (2,1) and (1,2). If the equation of the line AB is y =

A) 2y = x + 3
55.

B) y = 2x

B) 4

6 2
2

60.

D) 3x + 4y = 2

B) x y = 1

C) x = 0

5
2

D) x =1

B) c2 a2 = m2

C) 2c2(1+m2) = a2

D) c2 + a2 = m2

C) y3x+9=0, 3yx=3=0

D) y3x+3=0, 3y+x+9=0

On the portion of the straight line x+y=2 which is intercepted between the axes, a square is
constructed away from the origin, with this portion as one of its side. If p denote the perpendicular distance of a side of this square from the origin, then the maximum value of p is

B) 2 2

C) 3 2

D) 4 2

Two vertices of a triangle are (3,2) and (2, 3) and its orthocentre is (6, 1). Then its third vertex
is
B) (1,6)

C) (1,6)

D) None of these

A ray of light coming from the point (1,2) is reflected at a point A on the xaxis and then passes
through the point (5,3). The coordinates of the point A is
3
,0
5

261

C) 3x + 4y = 7

B) y+3x+9=0, 3y+x3=0

A)
63

62
2

A) y 3x+9=, 3y+x3=0

A) (1,6)
62.

B) 3x + 4y =

If one of the diagonal of a square is along the line x=2y and one of its vertices is (3,0) then its
sides through this vertex are given by the equation

A)
61.

D) 5

The pair of straight lines joining the origin to the common points of x2 +y2 = a2 and y = mx +c are
perpendicular to each other if
A) 2c2 =a2 (1+m2)

59.

C) 8

If a ray traveling along the line x = 1 gets reflected from the line x + y = 1 then the equation of the
line along which the reflected ray travels is
A) y = 0

58.

D) y = x 1

There are two parallel lines, one of which has the equation 3x + 4y = 2. If the lines cut an
intercept of length 5 on the line x +y = 1 then the equation of the other line is
A) 3x 4 y

57.

1
x 1
2

Let A = (1,2) B (3,4) and let C (x,y) be a point such that (x-1) (x-3) + (y-2) (y-4) = 0. If ar D
(ABC) =1 then the maximum number of position of C in the x y plane is
A) 2

56.

C) y =

1
X then the equation of the line AC is
2

5
,0
13

B)

C) 7,0

D) None of these

In a ABC, side AB has the equation 2x+3y=29 and the side AC has the equation x+2y=16. If the
mid point of BC is (5,6) then the equation of BC is

A) 2x+y=7
64.

88
3

B)

C)

B)

4
p

69.

1
y

2
p2

B) x2 y2 = a2 b2
D) x2 + y2 = a2 b2

B) (1,-3)

C) (-1,1)

D) (3,3)

Two sides of an isosceles triangle are given by the equation 7x y + 3 = 0 and x+y3=0. If its
third side passes through the point (1,-10) then its equation are
A) x 3y 7 = 0 ; 3x + y 31 = 0

B) x 3y 31 = 0 ; 3x + y 7 = 0

C) x 3y 31 = 0 ; 3x + y + 7 = 0

D) x + 3y 31 = 0 ; 3x + y + 7 = 0

In a rhombus ABCD the diagonals AC and BD intersect at the point (3,4). If the point A is (1,2)
the diagonal BD has the equation
B) x + y 1 = 0

C) x + y + 1 = 0

D) x + y 7 = 0

If each of the points (x, 4) and (-2,y) lies on the line joining the points (2, -1) and (5,-3) then the
point P(x1y1) lies on the line
A) x = 3y

71.

D)

The image of the point (-1,3) by the line x y = 0 is

A) x - y + 1 = 0
70.

The locus of the point of intersection of lines x cos + y sin = a and xsin - ycos = b is ( is
a variable)

A) (3, -1)
68.

D) None of these

2
2
C) x y p 2

A) 2(x2 + y2) = a2 + b2
C) x2 + y2 = a2 + b2
67.

4 7

The locus of the midpoint of the position intercepted between the axes by the line
x cos + y sin = P where P is a constant is
A) x2 + y2 = 4p2

66.

C) 2xy=17

ABC is an equilateral triangle such that the vertices B and C lie on two parallel lines at a distance
6. If A lies between the parallel lines at a distance 4 from one of them then the length of a side of
the equilateral triangle is
A) 8

65.

B) x+y=1

IIT- MATHS
D) None of these

B) x = -3y

C) y = 2x + 1

D) 2x + 6y +1 = 0

The ratio in which the line 3x + 4y+2 = 0 divides the distance between 3x + 4y + 5 = 0 and
3x + 4y 5 = 0 is
A) 7 : 3

72.

C) 2 :3

D) 1 : 2

If the foot of the perpendicular form the origin to the straight line is at the point (3,-4) then the
equation of the line is
A) 3x 4y = 25

73.

B) 3 : 7

B) 3x 4y +25 = 0

C) 4x + 3y 25 = 0

D) 4x 3y +25 =0

Point Q is symmetric to P(4,-1) with respect to the bisector of the first quadrant. Then length of
PQ is
262

CO-ORDINATE GEOMENTRY
A) 5 2
B) 5
74.

B) (1,-1)

9 9
2 2

C) (1,1)

7 7
2 2

B) ,

B) 6 sq units

D) 4 sq units

B) x2 + 2x 4y + 5 = 0
D) x2 + 2x + 4y 5 = 0

B) identical

B)

C) parallel

D) none

1
10
2

C)

5
2

D) 5

a2
2

D) 3a 2

Area of the quadrilateral formed by |x| + |y| = a is


A) a 2

81.

C) 11/2 sq units

Distance between orthocentre and circumcentre of the triangle whose vertices are (3,-1) (2,1) (0,0) is
A) 10

80.

D) 1,1

If the points a 1b1 a 2 b 2 a 3b 3 are collinear then the lines aix biy 1 0 i = 1, 2, 3 are
A) concurrent

79.

11 11
,
2 2

C)

Locus of all such points which is equidistant from (1,2) and x axis is
A) x2 2x 4y + 5 = 0
C) x2 2x + 4y + 5 = 0

78.

D) (3,5)

If the Co-ordinates of the midpoints of the sides of a triangle are (1,-1) (2,3) (3,2) then area of
triangle is
A) 5 sq units

77.

In center of the triangle whose vertices are (6,0), (0,6) and (7,7) is
A) ,

76.

D)

Orthocentre of the triangle whose vertices are (1,1) (3,5) (3,0) is


A) (-1,1)

75.

C) 2 5

B) 2a 2

C)

If one vertex of an equilateral triangle of side 2 is the origin and another vertex lies on the line x
= x 3 y then the third vertex can be
A) (0,2)

82.

B)

B) y =

B) 2

mn

263

2 ,1

x at an angle 45 is

C) y =

C) 2 2

2
2

m n

D) y =1

D) 4

B) m n

C) m n

D) m n

If the vertices P,Q,R of a triangle PQR are rational points, which of the following points of the
triangle PQR is always irrational
A) Centroid

86.

D)

Area of the parallelogram formed by the lines y = mx, y = mx +1, y = nx and y = nx+1 equals
A)

85.

C) (0,2)

The area a bounded by the curves y = |x| -1 and y = - |x| +1 is


A) 1

84.

2 ,1

The line which is parallel to x-axis and crosses the curve y =


A) x =

83.

B) Incentre

C) circumcentre

D) orthocentre

The equation of the straight line passing through the point (-2,3) and making intercepts of equal

IIT- MATHS
length on the axes is
A) 2x + y + 1= 0
87.

96.

D) none of these

B) ellipse

C) hyperbola

D) circle

B) b, 2a, c are in G.P

C) b, a/2, c are in A.P.D) b, 2a,c are in G.P

B) (2,1)

C) (1,2)

D) None of these

B) 2x 1 y 1 1

C) x 1 2 y 1 1

D) 2x 1 2 y 1 1

B) aybx+2b=0

C) ax+by+2b=0

D) None of these

If a ray traveling the line x=1 gets reflected the line x+y=1 then the equation of the line along
which the reflected ray travels is
A) y=0

95.

1
2h

C)
ab
1 n

The straight line y=x2 rotates about a point where it cuts xaxis and becomes perpendicular on
the straight on the line ax+by+c=0 then its equation is
A)ax+by+2a=0

94.

n
2h

B)
ab
1 n

A line passing through P(4,2) meets the x and yaxis at A and B respectively. If O is the origin,
then locus of the center of the circumcircle DOAB is
A) x 1 y 1 2

93.

D) (a - b)2 = 4h2

L is a variable line such that the algebraic sum of the distances of the points (1,1),(2,0) and (0,2)
from the line is equal to zero. The line L will always pass through.
A) (1,1)

92.

C) a b = 2h

If bx +cy=a, where a,b,c are the same sign, be a line such that the area enclosed by the line and the
axes of reference is 1/8 unit2 then
A) b,a,c are in G. P

91.

B) a + b = -2h

Let AB be a line segment of length 4 with the point A on the line y=2x and B on the line y=x.
Then locus of middle point of all such line segment is a
A) parabola

90.

D) x +y 2 =0

The slope of one of the lines represented by ax2 + 2hxy +by2 = 0 be n times the other than
n
1 n

A)
ab
2h

89.

C) x y + 5 = 0

If one of the lines of the pair ax2 + 2hxy + by2 = 0 bisects the angle between positive direction of
the axes, then a, b, h satisfy the relation
A) a + b = 2h

88.

B) x y = 5

B) xy=1

C) x=0

D) None of these

A line passing through the point (2,2) and the axes enclose an area . The intercepts on the axes
made by the line are given by the two roots of
A) x 2 2 | | x | | 0

B) x 2 | | x 2 | | 0

C) x 2 | | x 2 | | 0

D) None of these

Let A=(1,2), B=(3,4) and let C=(x,y) be a point such that (x1)(x3)+(y2)(y4)=0. If ar(DABC)
= 1 then maximum number of positions of C in the xy plane is.
A) 2

B) 4

C) 8

D) None of these

264

CO-ORDINATE GEOMENTRY
97. The limiting position of the point of intersection of the lines 3x+4y=1 and 91+c)x+3c2y=2 as c
tends to 1 is
A) (5,4)
98.

C) (4,5)

D) None of these

If the point (a,a) falls between the lines x + y = 2 then


A) |a| = 2

99.

B) (5,4)

B) |a| = 1

C) |a| < 1

D) |a| < 1/2

If A (cos, sin) B (sin, cos) C(1,2) are vertices of a ABC then as varies the locus of its
centroid is
A) x2 + y2 2x - 4y + 1 = 0

B) 3 (x2+ y2) 2x 4y + 1 = 0

C) x2 + y2 2x 4y + 3 = 0

D) x2 + y2 +2x + 4y 3 = 0

100. If a line joining points A(2,0) and B(3,1) is rotated through A in anticlockwise direction through
an angle 15, then the equation of the line in the new position is
A)

3x y 2 3

B) 3x y 2 3

C) x 3y 2 3

D) 3x y 3

101. If a straight line L perpendicular to the line 5x y = 1 such that the axes of the D formed by the
line L and the co-ordinate axes is 5, then the equation of the line L is
A) x + 5y + 5 = 0

B) x + 5y

2 0

C) x + 5y 5 0

D) x 5y 5 2 0

102. The equations of the lines on which the perpendicular form the origin make 30 angle with x
axis and which form a triangle of area 50/ 3 with axes are
A) x +

3 y 10 = 0

B) 3x y 10 0

C) x 3y 10 0

D) 3x y 10 0

103. If the extremities of the base of an isosceles triangle are the points (2a,0) and (0,a) and the
equation of one of the sides is x = 2a then the area of the triangle is
A) 5 sq units

B) 5/2 sq units

C) 25/2 sq units

D) 2 sq units

104. If one vertex of an equilateral triangle is at (2,-1) and the base is x + y 2 = 0, then the length of
each side is
A)

3
2

B)

2
3

C)

2
3

D)

3
2

105. If co-ordinates of orthocentre and centroid of a triangle are (4,-1) and (2,1) then co-ordinates of a
point which is equidistant from the vertices of the triangle is
A) (2,2)

B) (3,2)

C) (2,3)

D) (1,2)

106. Let co-ordinates of the two fixed points A and B are (a,0) and (0,b) respectively. A variable line
meet the axes at P and Q so that BP is always perpendicular to AQ. Then locus of the point of
intersection of BP and AQ is
B) x 2 y 2 ax by 0
D) x 2 y 2 ax by 0

A) ax + by + a + b = 0
C) y 2 4a x b

107. Locus of the centres of the circles touching the line 3x 4y + 1 = 0 and 12x + 5y 1 = 0 are
A) 21x+77y 18 = 0
265

B) 99x27y+8=0

C) (A) and (b) both

D) none of these

IIT- MATHS
108. Equation of the line passing through (1,1) and give an intercept between the lines
5x+12y+7=0 and 5x + 12y 32 = 0 of length 3 unit is
A) 12x 5y 7 = 0

B) 12x+5y+7 = 0

C) 12x + 5y 7 = 0

D) 12y + 5x 7 = 0

109. Equations of the straight lines, inclined at 30 to the axis of x such that the length of its (each of
their) lines segments between the co-ordinates axes is 10 units is
A) x 3y 5 3 0
C) x 3y 5 3 0

B) x 3y 5 3 0
D) x 3y 5 3 0

110. P(3,1) Q(6,5) and R(x,y) are three points such that the angle PRQ is a right angle and the area of
PQR = 7 then the number of such points R is
A) 0

B) 1

C) 2

D) 4

111. The vertices of a triangle ABC are (1,1) (4,-2) (5,5) respectively. The equation of perpendicular
dropped from C to the internal bisector of angle A is
A) y 5 = 0

B) x 5 = 0

C) 2x + 3y 7 = 0

D) x + 5 = 0

112. Let 0 < < /2 be a fixed angle. If P (cos, sin) and Q (cos ( - ), sin ( - ) then Q is obtained
from P by
A) clockwise rotation around origin through an angle
B) anticlockwise rotation around origin through an angle
C) reflection in the line through origin with slope tan
D) reflection in the line through origin with slope tan /2
113. The angle between a pair of tangent s drawn from point P to the circle
x 2 y 2 4x 6 y 9 sin 2 cos 2 0 is 2. The equations of locus of point p is
A) x 2 y 2 4x 6 y 4 0

B) x 2 y 2 4x 6 y 9 0

C) x 2 y 2 4x 6 y 4 0

D) x 2 y 2 4x 6 y 9 0

114. In the ABC the co-ordinates of B are (0,0) AB = 2 ABC = /3 and the middle point BC has the
co-ordinates (2,0) the centroid of the triangle is

1 3
A) 2 , 2

5 1

B) 3 ,
3

4 3 1
C) 3 , 3

D) none

115. The four sides of a quadrilateral are given by the equation xy (x-2) (y-3) = 0. The equation of the
line parallel to x 4y = 0 that divides the quadrilateral in two equal areas is
A) x 4y + 5 = 0

B) x 4y 5 = 0

C) 4y = x + 1

D) 4y + 1 = 0

116. The range of values of the ordinate of a point moving on the line x = 1 always remaining in the
interior of the triangle formed by the lines y = x and the x axis, x +y =4 is
A) (0,1)

B) [0,1]

C) [0,4]

D) (0,2)

117. If a pair of lines x2 2pxy y2 = 0 and x2 2qxy y2 = 0 is such that each pair bisects the angle
between the other pair then
266

CO-ORDINATE GEOMENTRY
A) pq = -1

C) p q 0

B) pq =1

D) p q 0

118. The four straight lines given by the equations 12x 2 + 7xy 12y2 = 0 and
12x2 + 7xy 12y2 x + 7y 1 = 0 lie along the sides of a
A) square
119

B) parallelogram

C) rectangle

D) rhombus

If P (1+ / 2 , 2 / 2 ) be any point on a line then the range of values of t for which the point P
lies between the parallel lines x+2y=1 and 2x=4y=15 is
A)

4 2
5 2

3
6

B) 0

5/ 2
6

C)

4 2
0
3

D) None of these

120. The point (4,1)undergoes the following two successive transformations :


A) reflection about the line y = x
B) rotation through a distance 2 unit along the positive xaxis. Then the final coordinates of the
point are
A) (4,2)

B) (3,4)

C) (1,4)

121. In the ABC, the coordinates of B are (0,0), AB=2, ABC=

D) (7/2, 7/2)

and the middle point of BC has


3

the coordinates (2,0). The centroid of the triangle is


1

A) 2 , 2

B)

4 3 1
,
3
3

C)

D) None of these.

122. There are two parallel lines, one of which has the equation 3x+4y=2. If the lines cut an intercept
of length 5 on the line x+y=1 then the equation of the other line is
A) 3x 4 y

6 2
2

B) 3x 4 y

62
2

C)3x+4y=7

D) None of these

123. Let P=(1,1) and Q=(3,2). The point R on the xaxis such that PR+RQ is the minimum is
5
3

A) ,0

1
3

B) , o

C) (3,0)

D)None of these

124. If () be an end of a diagonal of a square and the other diagonal has the equation xy= then
another vertex of the square can be
A) (ab,a)

B)(a,0)

C) (0,a)

D) None of these

125. The point (4,1) undergoes the following three transformations successively
(I) Reflection about the line y=x

267

IIT- MATHS
(II) Transformation through a distance 2 units along the positive direction of xaxis
(III) Rotation through an angle p/4 about the origin in the anticlockwise direction.
The final position of the point is given by the coordinates
1

A)

B) 2,7 2

C)

D)

2 ,7 2

126. P is a point on either of two lines y 3 x 2 at a distance of 5 units from their point of
intersection. The coordinates of the foot of perpendicular from P on the bisector of the angle
between them are

4 5 3 or 0, 4 5 3 ,( depending on which line the point P is taken)


2
2

45 3
2

A) 0,
B) 0,

C) 0,

45 3
2

5 5 3
D) 2 , 2

127. The equation of a line through the point(1,2) whose distance from the point(3,1) has the greatest
possible value is
A) y=x

B) y=2x

C) y=2x

D) y=x

128. The point P(2,1) is shifted by 3 2 parallel to the line x+y=1, in the direction of increasing
ordinate, to reach Q. The image of Q by the line x+y=1 is.
A)(5,2)

B)(1,2)

C)(5,4)

D)(1,4)

268

CO-ORDINATE GEOMENTRY

SECTION -B
MULTIPLE ANSWER TYPE QUESTIONS
1.

The points (2, 3) (0, 2) (4, 5) and (0,t) are concyclic of the value of t is
A) 1

2.

3.

B) 1

C) 17

The point of intersection of the lines

D) 3

x y
x y
=1 and 1 lies on
a b
b a

A) x-y = 0

B) (x+y) (a+b) = 2ab

C) (lx + my) (a + b) = (1 + m) ab

D) (lxmy) (a+b) = (1-m) = ab

The equations
(b-c)x + (c-a) y + a-b= 0
(b3-c3) x + (c3-a3)y + a3-b3 = 0 will represent the same line if
A) b =c

4.

6.

269

C) x2-y2 = 2(ax+by)

B) bx = ay

B) = -3

C) = 4

B) x + y - 3 = 0

B)

D) P can be (a, b)

D) = -4

C) x - 3y - 5 = 0

D) x - 3y + 5 = 0

3 a / 2, a / 2

C) (0, -a)

D) 3 a / 2, a / 2

If the lines ax + by + c = 0, bx + cy =a = 0 and cx + ay + b = 0 are concurrent (a+b+c 0) then


A) a3 + b3 + c3 - 3abc = 0
C) a = b = c

11.

D) (-1/4, 11/4)

If one vertex of an equilateral triangle of side a lies at the origin and the other lies on the line x 3y =0, the co-ordinates of the third vertex are
A) (0, a)

10.

C) (7/2, 13/2)

Equation of a straight line passing through the point of intersection of x-y+1 = 0 and 3x+y-5 = 0
are perpendicular to one of them is
A) x + y + 3 = 0

9.

B) (3/4, -3/2)

If the lines x - 2y - 6 = 0, 3x + y -4 = 0 and x + 4y + 2 = 0 are concurrent, then


A) = 2

8.

D) a + b + c = 0

The points (k, 2-2k), (-k+1, 2k) and (-4-k, 6-2k) are collinear for
1
A) any value of k
B) k=1
C) k =
D) no value of k
2
If the point P(x, y) be equidistant from the points A(a+b, a-b) and B(a-b, a+b) then
A) ax = by

7.

C) a = b

The area of a triangle is 5. Two of its vertices are (2, 1) and (3, -2). The third vertex lies on y =
x+3. The co-ordinates of the third vertex can be
A) (-3/2, 3/2)

5.

B) c = a

B) a = b
D) a2 + b2 + c2 - bc - ca - ab = 0

If the co-ordinates of the vertices of a triangle are rational numbers then which of the following
points of the triangle will always have rational co-ordinates

A) centroid
12.

B) incentre

Let S1, S2 .... be squares such that four each n 1, length of a side of Sn equals the length of a
diagonal of Sn+1. If the length of a side of S1 is 10cmm, then for which of the following values of
n is the area of Sn less than 1 sq. cm ?
A) 7

13.

B) 8

C) 9

1
unit2 then
8

A) b, a, c, are in G.P
C) b,

15.

A) bisector of the angle including origin

B) bisector of acetic angle

C) bisector of obtuse angle

D) none of these

Two roads are represented by the equations y - x = 6 and x + y = 8. An inspection bunglow has to
be so constructed that it is at a distance of 100 from each of the roads. Possible location of the
bunglow is given by
B) (1-100 2 , 7)

C) (1, 7 + 100 2 )

D) (1, 7 - 100 2 )

Angles made with the x-axis by two lines drawn through the point (1, 2) cutting the line x + y =

A)

5
and
12
12

6 /3 from the point (1, 2) are


B)

7
11
and
12
12

C)

3
and
8
8

D) none of these

If (a, b) be an end of a diagonal of a square and the other diagonal has the equation x-y = a then
another vertex of the square can be
A) (ab, a)

18.

D) b, -2a, c are in G.P.

Consider the straight lines x + 2y + 4 = 0 and 4x + 2y - 1 = 0. The line for 6x + 6y + 7 = 0 is

4 at a distance

17.

B) b, 2a, c are in G.P.

a
, c are in A.P.z
2

A) (100 2 + 1, 7)
16.

D) 10

If bx + cy = a, where a, b, c are the same sign, be a line such that the area enclosed by the lime and
the axes of reference is

14.

C) circumcentre

IIT- MATHS
D) orthocentre

B) (a, 0)

C) (0, a)

D) (a+b, b)

The points (p+1, 1), (2p+1, 3) and (2p+2, 2p) are collinear if
A) p = -1

B) p = 1/2

C) p = 2

D) y = -

1
2

270

CO-ORDINATE GEOMENTRY

SECTION - A
SINGLE ANSWER TYPE QUESTIONS
1.

2.

3.

4.

5.

6.

7.

8.

9.

10.

11.

16.

17. 18. 19.

20.

21. 22. 23

24. 25.

26.

12. 13.

27. 28.

14. 15.

29. 30.

31

32

33

34

35

36

37

38

39

40

41

42 43

44

45

A,B B,D C

A,B

46

47

48

49

50

51

52

53

54

55

56

57 58

59

60

61

62

63

64

65

66

67

68

69

70

71

72 73

74

75

271

IIT- MATHS

76

77

78

79

80

81

82

83

84

85

86

87 88

89

90

B,D

91

92

93

94

95

96

97

98

99 100 101 102 103 104 105

106 107 108 109

121

122 123 124

110 111 112 113 114 115 116

117 118 119 120

125 126 127 128

272

CO-ORDINATE GEOMENTRY

SECTION - B
MULTIPLE ANSWER TYPE QUESTIONS

1.

2.

4.

5.

6.

10.

11.

12. 13.

14. 15.

AC

AB AB AC
CD CD

BC

BD AD BD AB AB
CD CD

AC
D

BC BD
D

AB AB
CD

16.

17. 18.

AB

BD CD

273

3.

7.

8.

9.

IIT- MATHS

THREE DIMENSIONAL GEOMETRY


Distance bet ween any two point s P x1 , y1 , z1 and Q x 2 , y 2 , z 2 in space is given by

PQ x 2 x1 y 2 y1 z 2 z1

1
2 2

___

If P x1 , y1 , z1 , Q x 2 , y 2 , z 2 are two points, the points R which divides PQ in ratio l : m is given


mx1 lx 2 my1 ly 2 mz1 lz 2
,
,

by R
lm
lm
lm

___

If the ratio l : m in positive, R lies between P and Q on PQ and if the ratio of negative, R lies on
___
___
extension of PQ when l : m is positive, we say R divides PQ internally and when the ratio is negative,
___
we say R divides PQ externally..
x1 x 2 y1 y 2 z1 z 2
___
,
,

(i) Mid point of PQ =


2
2
2
(ii) If P x1, y1 , z1 Q x 2 , y 2 , z 2 R x3 , y 3 , z 3 are vertices of a triangle, the centroid (or the
center of gravity) of DPQR is G. Then
(iii)If P x1, y1 , z1 Q x2 , y2 , z2 R

x3 , y3 , z3 and S x4 , y4 , z4

are the vertices of a

xi yi zi
tetrahedron, its center of gravity (or) centroid is given by 4 , 4 , 4

PARALLEL SHIFTING OF AXES


If the origin is changed to x1 , y1 , z1 through parallel shifting of axes [i.e. the new axes are
parallel to original axes and passing through the point x1 , y1 , z1 ] then x X+ x1 , y Y+ y1 , z = Z+ z1
where P x, y, z changes to (X, Y, Z ) in new axes.
If a line makes angles , , with X, Y, Z axes respectively, the triple cos , cos , cos is called
a triple of the direction cosines of the given line.
(i)

If

cos , cos , cos is

a t riple of direct ion cosines of a line, we get


cos cos cos 1. Note that direction cosines (D.Cs) of a line are components of a
unit vector parallel to the given to the given line and two such triplets of D.Cs exist for any
line namely cos , cos , cos , cos180 , cos180 , cos180
2

(ii)

If O is the origin and P = x1 , y1 , z1 then D.Cs of OP are

x1
y1
z1

,
,
2
2
2
2
x2 y2 z2
x1 y1 z1
x1 y12 z12
1
1
1

274

CO-ORDINATE GEOMENTRY
(iii)
Any triple of numbers proportional to the D.Cs of a line is called a triple of direction ratios
(D.Rs) of the line. i.e. If l , m, n is a triple of D.Cs of a line, then lk , mk , nk , k R -{0} is
a triple of D.Rs of the given line.
If a, b, c is a triple of D.Rs of a line, then

(iv)

a
b
c

,
,
2
2
2
a2 b2 c2
a2 b2 c2
a b c

are triple of D.Cs of the given line.

If P x1 , y1 , z1 and Q x 2 , y 2 , z 2 are two points, a triple of D.Rs of PQ is given by


x2 x1 , y 2 y1 , z2 z1

(v)

If two rays have D.Cs l1 , m1 , n1 and l 2 , m2 , n2 respectively, the angle between them is
given by

l m

cos l1l 2 m1m2 n1 n2 (or) sin

l 2 m1

The lines are perpendicular if and only if l1l 2 m1m2 n1n 2 0


l1 m1 n1
The lines are parallel if and only if l m n
2
2
2
If two rays have D.Rs a1 , b1 , c1 a 2 , b2 , c 2 respectively, the angle between them is given
by

(vi)

cos

a1a 2 b1b2 c1c 2


a12 b12 c12 a 22 b22 c 22

(or)

sin

a b

1 2

a 2 b1

a12 b12 c12 a 22 b22 c 22

The lines are perpendicular a1a 2 b1b2 c1c 2 0


a1 b1 c1
The lines are parallel a b c
2
2
2

PROJECTION

Projection of a point P x, y, z in XOY


plane is x, y,0 , in YOZ
plane is 0, y, z and in XOZ
plane is x,0, z

Projection of the line segment joining P x1 , y1 , z1 and Q x 2 , y 2 , z 2


(i)

on X - axis = x 2 x1

(ii) on Y - axis = y 2 y1
(iii) on Z - axis = z 2 z1
(iv) on any line with the D.Cs l , m, n is x 2 x1 l y 2 y1 m z 2 z1 n

PLANE
The line joining two points on a plane totally lie on the same plane.
(i)

275

The general equation of a plane is ax by cz d 0

(ii)

IIT- MATHS
If a plane cuts the X, Y, Z axes at a,0,0 0, b,0 , 0,0, c respectively, its equation is given by
x y z
1 (Intercept form) a, b, c are called the X, Y, Z - intercepts of the plane respeca b c
tively.

(iii)

If the perpendicular from (0, 0, 0) to a plane has D.Cs l , m, n and is of length p then the
equation of the plane is lx my nz p (Normal form). Hence for a given plane
|d|

ax by cz d 0 , the perpendicular distance from (0, 0,0) is


(iv)

From the above we observe that the D.Rs of any normal to the plane ax by cz d 0 are
proport ional to

a, b, c

and the D.Cs of any line normal to this plane are

a
b
c

,
,
2
2
2
a2 b2 c2
a2 b2 c2
a b c

(v)

a2 b2 c2

The equation of the plane through x1, y1 , z1 , whose normal has D.Rss a, b, c is given by
a x x1 b y y1 c z z1 0 .
Also the equation of the plane parallel to Ax By Cz D 0 through x1 , y1 , z1 is
A x x1 B y y1 C z z1 0

(vi)

The equation of the plane passing through three non collinear points A x1 , y1 , z1 , B x 2 , y 2 , z 2

x x1
x x1
and C x3, y 3, z 3 is 2
x3 x1

y y1
y 2 y1
y3 y1

z z1
z 2 z1 0
z3 z1

STRAIGHT LINE
If P x, y, z is any point in the space,
(i)

foot of the perpendicular from P to X -axis is x,0,0


and the perpendicular distance of P to X-axis is

(ii)

foot of the perpendicular from P to Y -axis is 0, y ,0


and the perpendicular distance of P to Y-axis is

(iii)

x2 z 2

foot of the perpendicular from P to Z -axis is 0,0, z


and the perpendicular distance of P to Z-axis is

(iv)

y2 z2

x2 y2

If R x, y, z is any point on the line passing through P x1 , y1 , z1 and Q x 2 , y 2 , z 2 we get that

x x1 , y y1 , z z1 and x2 x1 , y 2 y1 , z 2 z1 are two triples of D.Rs of the line PQ and


276

CO-ORDINATE GEOMENTRY
they must be proportional (i.e.)

x x1
y y1
z z1

t (say)
x 2 x1 y 2 y1 z 2 z1

Hence we get x x1 t x 2 x1 , y y1 t y 2 y1 , z z1 t z 2 z1 . Therefore any point


on the line can be taken in the form for some t R.

277

IIT- MATHS

SOLVED PROBLEM
ILLUSTRATION : 01
The perpendicular distance of (3, 4, 5) from the Z-axis is
(a)

26

(b)5

(c) 17

(d) 10

Ans : (B)
Solution:
The perpendicular distance to Z -axis =

x 2 y 2 9 16 5

ILLUSTRATION : 02
If D.Cs of two liens satisfy the relations 3l m 5n 0 and 6mn 2nl 5lm 0 , the angle
between them is
1 1
(a) Cos
6

1 1
(b) Cos
3

(c)

1 3
(d) Cos
4

Ans : (A)
Solution :
Eliminating m from the given relations we get l 2 3 ln 2n 2 0
i.e. l 2n l n 0 ;

Hence using 1st leation

i.e.

l
n
l
n
or
2 1
1 1

l
m n
l
m
n
or

hold.
2 1 1
1 2 1

The angle between these lines is given by

cos

2 2 1
4 11 1 4 1

1
6

ILLUSTRATION : 03
The foot of the perpendicular from (1,2,3) to the line joining the points (6,7,7) and (9,9,5) is
(a) (5, 3, 9)

(b) (3, 5, 9)

(c) (3, 9, 5)

(d) (3, 9, 9)

Ans : (B)
Solution :
Any point on the line joining the given points can be taken as 6 3t, 7 2t, 7 - 2t If it
is required foot of the perpendicular of (1, 2, 3) we get 3(5 3t) 2(5 2t) - 2(4 - 2t) 0 i.e, t = -1

278

CO-ORDINATE GEOMENTRY
ILLUSTRATION : 04
If A (1, 2, 3), B(6, 7, 8) C(1,2, 5) and D (3, 0, 4) are given points, then the projection of

AB on CD is

(a) 1/3

(b) 4/3

(c) 25/3

(d) 5/3

Ans : (D)
Solution :

D.Rs of CD are (2, -2, -1) or (-2, 2, 1)

2 2 1

D.Cs of CD are 3 , 3 , 3

2
2
1
2 2 1 5
___

Projection of AB on CD = 6 1 7 2 8 3 = 5
3
3
3
3

3
ILLUSTRATION : 05
The equation of the plane passing through the three points are (-2, -2, 2), (1, 1, 1) and (1, -1, 2) is
(a) x 3 y 6 z 8 0

(b) x 3 y 6 z 14 0

(c) x 3 y 6 z 4 0

(d) x 3 y 6 z 20 0

Ans : (A)
Solution :
The required plane equation is

x 1 y 1 z 1 x 1 y 1 z 1
3
3
1
3
3
1
= 0; i.e. x 1 3 2 y 10 3 z 16 0
0
2
1
0
2
1
i.e. x 3 y 6 z 8 0
ILLUSTRATION : 06
If (2, 3, -1) is the foot of the perpendicular from (4, 2, 1) to a plane, then equation of the plane is
(a) 2 x y 2 z 3 0

(b) 2 x y 2 z 9 0

(c) 2 x y 2 z 5 0

(d) 2 x y 2 z 1 0

Ans : (D)
Solution :
The line joining the given points is normal to the plane.
279

IIT- MATHS
D.Rs of normal are (2, -1, 2) and (2, 3, -1) lies in the plane.
The equation of the plane is 2 x y 2 z 4 3 2
ILLUSTRATION : 07
The equation of the plane parallel to x y 2 z 3 0 through (1, 2, 4) is
(a) x y 2 z 11 0 (b) x y 2 z 6 0 (c) x y 2 z 11 0 (d) x y 2 z 8 0
Ans : (C)
Solution :
Plane parallel to the given plane can be taken in the form x y 2 z k 0 . This pass through
(1,2, 4).

k 1 2 8 11

ILLUSTRATION : 08
The point which is equidistant from A(3, 4, -1) and B(1, -2, 5) on Y-axis is
(a) (0,1, 0)

1
(b) 0, ,0
3

1
5

(c) 0, ,0 (d) 0, ,0
3
3

Ans : (C)
Solution :
___

The plane that perpendicularly bisects AB is 2 x 2 6 y 1 6 z 2 0


(i.e.) x 3 y 3z 1 0
1

This cuts Y-axis at 0, ,0


3

ILLUSTRATION : 09
The plane 2 x 2 y 3z 14 0 and the line joining (1, 2, 4) and (3, 3, 0) intersect at
(a) (5, 2, 0)

(b) (5, 4, -4)

(c) (-3, -1, -6)

(d) (10, -15, 12)

Ans : (B)
Solution
Any point on the given line is 1 2t , 2 t , 4 4t
This lies in the given plane; i.e. 21 2t 22 t 34 4t 14 0
(i.e.,) 4t 2t 12t 2 4 12 14 0
(i.e.) 14t 28 0 t = 2
Intersection point is (5, 4, -4)

280

CO-ORDINATE GEOMENTRY
ILLUSTRATION : 10
The equation of the plane through (2, -3, 1) which is perpendicular to line joining the points (3,4,
-1) and (2, -1, 5) is
(a) x 5 y 6 z 19 0
Ans : (A)

(b) x 5 y 6 z 7 0 (c) x 5 y 6 z 23

Solution:
DRs of line joining (3, 4, -1) (2, -1, 5) are 1, 5, -6
The required plane is x 5 y 6 z 2 15 6 19

281

(d) x 5 y 6 z 11 0

IIT- MATHS

SECTION - A
SINGLE ANSWER TYPE QUESTIONS
1.

The projection of the line PQ joining the points P(3,4,5) Q(4,6,3) on x-axis is
a) 1

2.

a b c

2 2 2

c) 21

d) 7/9

b) 5

c) 5 2

d) 2

b) 7 : 5

c) 9 : 11

d) 11: 9

b) 4x+y-3z-26=0

c) 2x-4y+3z+23=0

d) 3x+5y-2z+12=0

b) (-1,0,-7)

c) (1,0,-7)

d) (7,0,1)

b) -4

c) 4

d) 0

a b c
2 2 2

b) , ,

a b c
,
2 2 2

c) ,

a b c
, ,
2 2 2

d)

The foot of the perpendicular drawn from the point A (1,2,1) to the line joining B(1,4,6) and
C(5,4,4) is
a) (-3,-4,5)

12.

b) 7/6

The co-ordinates of a point equidistant from the four points O(0,0,0) A(a,0,0) B(0,b,0) and C(0,0,c)
is
a) , ,

11.

9 3 3
,
2 2 2

d) ,

The points A(1,-1,1) B(2,a,5) and C(5,-13,11) are collinear then a =


a) 4

10.

9 7 3
, ,

2 2 4

c)

The point of intersection of the line through (-2,3,4) (1,2,3) with the xoz plane
a) (1,0,7)

9.

3 7 7
2 2 2

b) , ,

The equation of the plane passing through (2,3,-5) and perpendicular to the planes x+2y+2z-8=0
and 3x+3y+2z+5=0 is
a) 2x-3y+2z+15=0

8.

d) (0, 0, 6)

The ratio in which the plane 2x3y+5z2=0 divides the line segment joining (1,2,3) (2,1,2) is
a) 3 : 5

7.

c) (1/2, 1/3,1/4)

If the projections of a line segment on the axes are 3,4,5 then the length of the line segment is
a) 12

6.

b) (11,-16,2)

The projection of the join of the two points (2,5,6) (3,2,7) on the line whose D.rs are (6,-3,6) is
a) 7/3

5.

d) 1

If the orthocentre and circumcentre of a triangle are (3, 5, 2), (6, 2, 5) then its centroid is
a) (3, 3, 4)

4.

c) 2

The harmonic conjugate of A (2, 3, 4) with respect to B (3, 2, 2), C (6, 17, 4) is
a) (18/5, 5,4/5)

3.

b) 3

b) (3,-4,5)

c) (-3,4,-5)

d) (3,4,5)

The angle between the points passing through the points (8,2,0) (4,6,-7) and (-3,1,2) (-9,-2,4) is

282

CO-ORDINATE GEOMENTRY
2

13.

14.

1
b) Cos

a)

l1 l 2 m1 m 2 n 1 n 2
,
,
2
2
2

b) l1 + l2, m1 + m2 , n1 + n2

c)

l1 l 2 m 1 m 2 n 1 n 2
,
,
3
3
3

d)

b)

1
2

c)

66

b) x2+y26y+ 25 = 0
d) x2 + y2 6y 8z + 25 = 0

b) (1,2,2) (2,1,2)
d) (1,2,-2), (2,-1,-2)

b) 45o

c) 60o

b) 2x-3y+3z+5= 0

283

d) 2x-3y-3z+5 = 0

b) 2x + 4y 3z 39 = 0
d) 2x + 4y 3z = 0

The equation of the plane lying mid way between 2x + 3y 6z + 1 = 0 and 2x+3y-6z+7 = 0 is
b) 2x-3y+6z 4 =0
d) 2x+3y+6z-8 = 0

A plane passing through the fixed point (a,b,c) cuts the coordinate axes at A,B,C. Then the locus
of the centroid of the triangle is
a)

22.

c) 12x+6y-5z = 1

If (2, 4, 3) is foot of perpendicular drawn from origin to plane, then the equation if the plane is

a) 2x+3y-6z+4 = 0
c) 2x+3y-6z+8 = 0
21.

d) 90o

Equation of the plane passing through the points (1,1,0) (1,2,1) and (-2,2,-1) is

a) 2x + 4y 3z 29 = 0
c) 2x + 4y + 3z + 29 = 0
20.

66

If the d.cs l,m,n of two lines are connected by the relations l+m+n = 0, l2 m2 + n2 = 0 then the
angle between the lines is

a) 2x+3y-3z = 5
19.

d)

If the d.cs l,m,n of two lines are connected by the relations 2l+2m-n = 0 and mn+nl+mn =0 then
the d.rs of the two lines are

a) 30o
18.

17
2

The equation of the locus of the point which moves in such a way that the sum of its distances
from (2,3,4) and (-2,3,4) is 4 is

a) (1,-2,-2) (2,-1,2)
c) (1,-2,2) (2,1,-2)
17.

l1 l 2 m 1 m 2 n 1 n 2
,
,
2
2
2

The distance between orthocentre and circumcentre of the triangle formed by (1,2,3)(3,-1,5)(4, 0, 3) is

a) y2+z26x8z+25=0
c) x2 + y2 + z2 25 = 0
16.

d) Cos
63

1
c) Cos

If (l1, m1, n1), (l2, m2, n2) are d.cs of two lines inclined at an angle 1200, then the d.cs of the line
bisecting the angle between them are

a) 0
15.

63

63

a) Cos
63

x y z
=3
a b c

b) x y z 3

c) ax+by+cz = 3

d) x2 + y2 + z2 = a2+b2+c2

A variable plane is a at a constant distance 3p from the origin and meets the axes in A, B and C.
The locus of the centroid of the triangle ABC is

IIT- MATHS
1

16

a) x 2 y 2 z 2 p 2 b) x 2 y 2 z 2 p 2 c) x 2 y 2 z 2 p 2 d) x 2 y 2 z 2 p 2
23.

The end points of a diagonal of a rectangular parallelopiped with faces parallel to the coordinate
planes are (2,3,5) and (5,7,10). The lengths of its edges are
a) 5,4,3

24.

30.

b) (1,1,5)

b) (-1/2, 2, 0)

c) (3, 5, 2)

b) 450

c) 600

x 1 y 1 z 10

is
2
3
8k

a) (3, -4, -2)

c)(1,-1,-10)

d) (1,-1,-5)

d) (1/2, -2, 0)

d) (3, 5, 2)

b)(5, -8,-4)

d) None of these

d)(2,-3,8)

If the foot of the perpendicular from the origin to a plane is (a,b,c) the equation of the plane is
x y z
3
a b c

b) ax by cz 3
d) ax by cz a b c

The angle between the lines whose direction cosines are given by the equations l2+m2n2 =0,
l+m+n = 0 is
a) /6

b) /4

c) /3

d)/2

The volume of the tetrahedron included between the plane 3x+4y-5z-60=0 and the coordinate
planes is
a) 60

33.

c) (-1/2, -2, 0)

b) (3, 5, 2)

c) ax by cz a 2 b 2 c 2

32.

c) (1,1,-5)

The reflection of the point P(1,0,0) in the line

a)

31.

d) None

A line makes an angle of 600 with each of x and y-axis, the angle which it makes with the z -axis
is
a) 300

29.

c) 13

The image of (1, 3, 4) in the plane 2x y +z + 3 = 0 is


a) (3, 5, 2)

28.

b) 2 13

The circumcentre of the triangle formed by the points A(1,1,0) B(1,2,1) and C(0,-5,1) is
a) (1/2, 2, 0)

27.

d) 3,4,5

A point lying on the line joining the points (-3,5,4) and (1,-1,5) has its x-coordinate as 1 then the
point is
a) (1,-1,5)

26.

c) 3,5,4

The extremities of a diagonal of a rectangular parallelopiped whose faces are parallel to reference planes are (-2,4,6) and (3,16,19). The length of the base diagonal is
a) 13

25.

b) 4,5,3

b) 600

c) 720

d) none of these

The plane passing through the point (-2,-2,2) and containing the line joining the points (1,1,1)
and (1,-1,2) makes intercepts on the coordinates axes the sum of the whose lengths is
a) 3

b) 4

c) 6

d) 12
284

CO-ORDINATE GEOMENTRY
34. A line segment has length 63 and direction ratios are (3, -2,6). If the line makes an obtuse angle
with x-axis, the components of the line vector are
a) 27, -18,54
35.

The lines

b) x 2 y 2 z 2 2

c) x y z 1

d) x y z 2

b) 3:2

c) 4:5

d) -7 :8

b) r 2 x ry z 3r 2

c) x ry r 2 z 3

d) r 2 x ry z 3

b) 19/2

c)-22/3

d) 26/3

Equation of the plane passing through the origin and containing the lines whose direction ratios
are 1, -2, 2 and 2, 3, -1 is
a) x 2 y 2z 0

41.

d) k = 3 or -3

Algebraic sum of the intercepts made by the plane x+3y-4z+6 =0 on the axes is
a) -13/2

40.

c) k = 0 or -3

A plane meets the coordinates axes in A,B,C such that the centriod of the triangle is the point (1,
r, r2), the equation of the plane is
a) x ry r 2 z 3r 2

39.

b)k =1 or -1

The ratio in which the yz plane divides the line joining the points (-2,4,7) and (3,-5,8) is
a) 2 :3

38.

d)27,-18,-54

A point moves so that the sum of the squares of its distances from the six faces of a cube given by
x = 1 , y = 1 , z= 1 is 10 units. The locus of the point is
a) x 2 y 2 z 2 1

37.

c) -27,18,-54

x 2 y 3 z 4
k 1 y 4 z 5

and
are coplanar if
1
1
k
k
2
1

a) k = 0 or -1
36.

b) -27,18,54

b) 2x 3y z 0

c) x 5 y 3z 0

d) 4x 5y 7 z 0

If a line makes angles , , , with four diagonals of a cube, then cos 2 cos 2 cos 2 cos 2
is equal to
a) 1/3

42.

285

b)3

c)6

d)None of these

b) xyz 8k 3

c) x y z 6k

d) x 3 y 3 z 3 64k 3

A plane meets the co-ordinate axes in A, B, C such that the centroid of the triangle ABC is the
point (a, b, c). Then the equation of the plane is
a)

45.

d) 8/3

A variable plane makes with coordinate planes a tetrahedron of constant volume 64k3. The locus
of the centroid of the tetrahedron is
a) xyz 6k 3

44.

c) 4/3

The area of the triangle with vertices A(3,4,-1) B(2,2,1) and C(3,4,-3) is
a) 4 5

43.

b) 2/3

x y z
3
a b c

b)

a b c
3
x y z

c) ax + by + cz = 3

d) none of these

The equation of the plane through the points (1, 0, -1) and (3, 2, 2) and parallel to the line

IIT- MATHS

x 1 y 1 z 1

is
1
2
3
a) 4x+y+2z = 6
46.

b) 3/2

65

c) 65 / 3

c) 3

d) 1

The centre of the circle given by r ( i 2 j 2k ) = 15 and r ( j 2k ) 4 is


b) (-1, 3, 4)

c) (1, -3, 4)

d) (1, 3, -4)

The d.cs of a line which makes equal angles with the axes is
1
1 1
,
b) ,
3
3 3

a) 1, 1, 1
1

,
,

c)
2
2 2

50.

d) none of these

b) 4

a) (1, 3, 4)
49.

d) none of these

If a, b, g are angles which a line makes with the axis then the value of sin2a + sin2b + sin2g is equal
to
a) 2

48.

c) 4x-y-2z = 6

The area of the triangle whose vertices are (0, 0, 0), (3, 4, 7) and (5, 2, 6) is
a) 3/ 74

47.

b) 4x-y+2z = 6

d) None of these

When a right handed rectangular cartesian system OXYZ is rotated about the Z-axis through an
angle p/4 in the counter - clockwise direction it is found that a vector a has the components
2 3 , 3 2 and 4. The components of a in the OXYZ coordinate system are
a) 5, -1, 4

51.

b) 5, -1, 4 2

d) none of these

b) 1

c) -1

d) 0

The coordinates of the foot of the perpendicular from the point A (1, 8, 4) to the line joining
B(0, -1, 3) and C(2, -3, -1) is
a) (-5/3, 2/3, 19/3)

b) (5/3, 2/3, 19/3)

c) (5/3, -2/3, 19/3)

d) none

The locus of a point which moves so that the difference of the squares of its distance from two
given points is constant is a
a) straight line

55.

The value of x for which the points A (1,0,3), B(-1,3,4) C(1,2,1) and D(x,2,5) are coplanar is
a) 2

54.

b) x y z = 1

c) 1 x 1 y 1 z =1

53.

d) -1, 5, 4

The coplanar points A, B, C, D are (2-x, 2, 2), (2, 2-y, 2), (2, 2, 2-z) and (1, 1, 1) respectively, then
a) x + y + z = 1

52.

c) -1, -5, 4 2

b) plane

c) sphere

P (1,1,1) and Q () are two points in the space such that PQ =

d) none

27 , the value of can be


286

CO-ORDINATE GEOMENTRY
a) -4
b) 2
56.

The lines

b) 14 / 3

b) -1

The lines

d) -2

c) -69

d) 28

X 1 Y 1 Z 2 X 1 Y Z 1

,

are
1
2
1
2
1
4
b) intersecting lines

c) perpendicular lines

d) none of these

The projections of a line on the axes are 9, 12 and 8. The length of the line is
a) 7

b) 17

c) 21

d) 25

A variable plane makes with the coordinate planes, a tetrahedron of constant volume 64K3. Then
the locus of the centroid of tetrahedron is the surface.
a) xyz = 6k2

64.

c) 2

b) 73

a) parallel lines

63.

d) 5/ 3

The plane 4x+7y+4z+81=0 is rotated through a right angle about its line of intersection with the
plane 5x + 3y + 10z = 25. The equation of the plane in its new position is x-4y+6z=k, where k is
a) 106

62.

c) 16/ 3

AP BQ CR DS

equals.
PB QC RD SA

a) 1

61.

d) none

P, Q, R, S are four coplanar points on the sides AB, BC, CD, DA of a skew quadrilateral. The
product

60.

b) intersect at (4, 0, -1) c) intersect at (1, 1, -1)

The distance of the point A (-2, 3, 1) from the line PQ through P (-3, 5, 2) which make equal
angles with the axes is
a) 2/ 3

59.

b) x2 + y2 + z2 + x + y + z = 0
d) x2 + y2 + z2 - x - y - z - 2 = 0

x 1 y 1 z 1
x 4 y 0 z 1

and
3
1
0
2
0
3

a) do not intersect
58.

d) none of these

The equation of a sphere which passes through (1, 0, 0) (0, 1, 0) and (0, 0, 1) and whose centre
lies on the curve 4 x y = 1 is
a) x2 + y2 + z2 - x - y - z = 0
c) x2 + y2 + z2 + x + y + z = 0

57.

c) -2

b) xy + yz + zx = 6k2

The plane cont aining t he t wo lines

c) x2 + y2 + z2 = 8k2

d) none

X 3 Y 2 Z 1
X 2 Y 3 Z 1

and

is
1
4
5
1
4
5

11x+my+nz = 28 where
a) m = -1, n = 3
65.

c) m = -1, n = -3

d) m = 1, n = 3

A variable plane passes through the fixed point (a,b, c) and meet the axes at A,B,C. The locus of
the point of intersection of the planes through A,B,C and parallel to the co-ordinate plane is
a)

287

b) m = 1, n = -3

a b c
2
x y z

b)

a b c
1
x y z

c)

a b c
1
x y z

d) x y z 2

66.

IIT- MATHS
A line passes through the points (6, -7, -1) and (2, -3, 1). The direction cosines of the line so
directed that the angle made by it with positive direction of X-axis is acute, are
a)

67.

2 2 1
,
,
3 3 3

The point in which the line


a) (7, -8, 26)

68.

2 2 1
, ,
3 3 3

c)

2 2 1
,
,
3 3 3

d)

2 2 1
, ,
3 3 3

X 2 Y 1 Z 2

meets the plane x-2y+z = 20 is


3
4
12

b) (8, 7, 26)

c) (7, 8, 26)

d) none

If P(3, 2, -4) , Q (5, 4, -6) and R(9, 8, -10) are collinear then R divides PQ in the ratio
a) 3 : 2 internally

69.

b)

b) 3 : 2 externally

c) 2 : 1 internally

The equation of the plane perpendicular to the line

d) 2 : 1 externally

X 1 Y 2 Z 1

and passing through


1
1
2

the point (2, 3, 1) is


a) r.i j 2k 1
70.

1 2 2
, ,
3 3 3

b)

d)

1 2 2
,
,
3 3 3

c) x+y+z = 1

d) none

b) r2x + ry + z = 3r2

c) x + ry + r2z = 3

d) r2x + ry + z = 3

c) 7 : 8

d) 1 : 1

A line segment has length 63 and direction ratios are (3, -2, 6). If the line makes an obtuse angle
with X-axis, the components of the line vector are

The lines
a) k = -1

76.

1 2 2
,
,
3 3 3

X 1 Y 3 Z 2

and the point (0, 7, -7) is


3
2
1

b) x+y+z = 2

b) 4 : 5

a) 27, -18, 54
75.

c)

The ratio in which the plane 2x - 1 = 0 divide the line joinig (-2, 4, 7) and (3, -5, 8) is
a) 2 : 3

74.

d) none

A plane meets the coordinate axes in A, B, C such that the centroid of the triangle is the point (1,
r, r2) the equation of the plane is
a) x + ry + r2z = 3r2

73.

1 2 2
, ,
3 3 3

The equation of plane containing the line


a) x+y+z = 0

72.

c) r.i j 2k 7

A mirror and a source of light are situated at the origin O and at a point on OX respectively. A ray
of light from the source strikes the mirror and is reflected. If the DRs of the normal to the plane
are 1, -1, 1 then DCs of the reflected ray are
a)

71.

b) r.i j 2 k 1

b) -27, 18, -54

c) -27, 18, -54

d) 27, -18, -54

X 2 Y3 Z4
X 1 Y 4 Z 5

and
are coplanar if
1
1
k
k
2
1
b) k = +3

c) k = 1

d) k = 0

If P1, P2, P3 denote the distances of the plane 2x - 3y + 4z + 2 = 0 from the planes
2x-3y+4z+6 = 0, 4x-6y+8z+3 = 0 and 2x - 3y+4z-6 = 0 respectively then
288

CO-ORDINATE GEOMENTRY
a) P1 + 8P2 - P3 = 0
b) P3 = 16P2
77.

b) y-axis

b) 4 planes

r r r

b) x , y , z

The line

85.

86.

289

x y z
, ,
r r r

d) none of these

b) sin2 + sin2 + sin2 = 1


d) cos2+cos2+sin2=1

b) (-3, 5, 2)

c) (3, -5, 2)

d) (3, 5, -2)

X 2 Y 1 Z 1

intersects the curve xy = c2, z = 0 if c =


3
2
1
b) 1/3

c)

d) none

The number of spheres of radius r touching the coordinate axis is


a) 4

84.

d) 6 planes

The image of the point P(1, 3, 4) in the plane 2x - y + z + 3 = 0 is

a) 1
83.

c) 5 planes

If be the angles which a line makes with the coordinate axes, then

a) (3, 5, 2)
82.

d) yz- plane

c)

a) sin2 + cos2 + sin2 = 1


c) cos2+cos2+cos2 = 1
81.

c) z-axis

If (x, y, z) be the coordinates of a point P and OP = r then the direction cosines of OP are
a) rx, ry, rz

80.

d) (A) , (B) , (C)

Tetrahedron is bounded by
a) 3 planes

79.

2a

Graph of the equation y2 + z2 = 0 in three dimensional space is


a) x-axis

78.

c) P1 + 2P2 + 3P3 =

b) 6

c) 8

d) none

If 1 , m1, n1 and 2 , m2, n2 are DCs of the two lines inclined to each other at an angle q, then the
DCs of the internal bisector of the angle between these lines are
a)

1 2 m1 m 2 n 1 n 2
,
,
2 sin / 2 2 sin / 2 2 sin / 2

b)

1 2 m1 m 2 n 1 n 2
,
,
2 cos / 2 2 cos / 2 2 cos / 2

c)

1 2 m1 m 2 n 1 n 2
,
,
2 sin / 2 2 sin / 2 2 sin / 2

d)

1 2 m1 m 2 n 1 n 2
,
,
2 cos / 2 2 cos / 2 2 cos / 2

If 1 , m1, n1 and 2 , m2, n2 are DCs of the two lines inclined to each other at an angle q then the
DCs of the external bisector of the angle between the lines are
a)

1 2 m1 m 2 n 1 n 2
,
,
2 sin / 2 2 sin / 2 2 sin / 2

b)

1 2 m1 m 2 n 1 n 2
,
,
2 cos / 2 2 cos / 2 2 cos / 2

c)

1 2 m1 m 2 n 1 n 2
,
,
2 sin / 2 2 sin / 2 2 sin / 2

d)

1 2 m1 m 2 n 1 n 2
,
,
2 cos / 2 2 cos / 2 2 cos / 2

1 3 3

, , then other end is


If one end of a diameter of sphere x2 + y2+z2 - 2x - 2y - 2z + 2 = 0 is 1
2 2 2

IIT- MATHS

a) 1

1 1
, ,
2 2 2

87.

88.

b) 1

1 3 3

,
,
2 2 2

a)

x 1 y 2 z 3

1
1
1

c)

x 1 y 2 z 3

2
3
1

b)

x 1 y 2 z 3

1
1
1

a b c

2 abc

b) | a x b b x c c x a |

|axbbxccxa |

d) none

A (3, 2, 0), B(5, 3, 2) and C(-9, 6, -3) are the vertices of a triangle ABC. If the centroid of ABC
meets BC at D, then coordinates of D are
19 57 17
, ,
8 16 16

19 57 17
, ,
8 16 16

b)

19 57 17
,
,
8 16 16

c)

b) 2x - y = 5

c) 2x + z = 5

The points A (0, 0, 0), B(2, 0, 0), C(1,


a) rhombus

b) square

c) a regular tetrahedron

d) none

If P is the length of perpendicular from the origin on to the plane where intercepts on the axes are
a, b, c then
d) none

A line makes angle a, b, g, d with the four diagonals of a cube, then the value of cos2+cos2+
cos2 + cos2 is equal to
a) 3/4

94.

d) 2x - z = 5

1 2 2
,
1,
are the vertices of a
,
0)
and
D
3
3 3

a) a 1 b 1 c 1 p 1 b) a 2 b 2 c 2 p 2 c) a + b + c = p
93.

d) none

The Cartesian equation of the plane r ( 1 )i ( 2 ) ( 3 2 2 )k is


a) 2x + y = 5

92.

d) none

The length of the perpendicular from the origin to the plane passing through three non-collinear
points a , b, c is

a)

91.

d) none

c) a b c

90.

1 1
,
2 2 2

Equation of a line passing through (-1, 2, -3) and perpendicular to the plane 2x+3y+z + 5 = 0 is

a)

89.

c) 1

b) 1/4

The coordinates of a point on the line

c) 4/3

d) 2/3

x 1 y 1

=z at a distance 4 14 from the point


2
3

(1, -1, 0) are


a) 9,13,4
c) (-7, 11, -4)
95.

b) - 8 14 +1, 12 14 -1, -4
d) 8+1, -12-1, 4

The position vectors of points A and B are and respectively. The equation of a plane is = 0. The

290

CO-ORDINATE GEOMENTRY
points A and B
a) lie on the plane
c) are on opposite sides of the plane
96.

291

b) are on the same sides of the plane


d) none of the above

The extremities of a diameter of a sphere lie on positive y and positive z-axis at distance 2 and 4
from the origin respectively, then
a) sphere passes through origin

b) centre of the sphere is (0, 1, 2)

c) radius of the sphere is

d) all the above

IIT- MATHS

SECTION - B
MULTIPLE ANSWER TYPE QUESTIONS
1.

The line whose vector equations are


r = 2i 3j + 7k + (2i + pj + 5k)
and

r = i + 2j + 3k + (3i pj + pk)

are perpendicular for all values of l and m if p =


a) 1
2.

b) 2

b) 2

The lines

d) 5

b) (2, 3, -1)

c) (3, 1, 1)

d0 (4, 0, 7)

x 2 y3 z 4
x 1 y 4 z 5

and
are coplanar if
1
1
k
k
2
1

a) k = 0
5.

c) 5

The plane passing through the origin and containing the line whose direction cosines are proportional to 1, -2, 2 and 2, 3, -1 passes through the point
a) (1, -2, 2)

4.

d) 6

A plane meets the coordinate axes in A, B, C such that the centroid of the triangle ABC is the
point (1, r, r2). The plane passes through the point (4, -8, 15) if r is equal to
a) 3

3.

c) 5

b) k = -1

c) k = 2

d) k = -3

An equation of the line passing through 3i 5j + 7k and perpendicular to the plane 3x4y+5z= 8
is
a)

x 3 y5 z 7

3
4
5

b)

c) r = 3i 5j + 7k + l (3i 4j + 5k)

x 3 y 4 z 5

3
5
7

d) r = 3i 4j 5k + m (3i + 5j + 7k)

( are parameter)
6.

The coordinates of a point on the line

x 1 y 1

= z at a distance 4 14 from the point (1,-1,


2
3

0) are

7.

a) (9, -13, 4)

b) 8 14 1, 12 14 1, 4 14

c) (-7, 11, -4)

d) 8 14 1, 12 14 1, 4 14

If a plane is at a distance 3/2 from the origin O, and meets the axes in A, B and C, the coordinates
of the centroid can be
1
a) 1, , 1
2

8.

b) , 1, 1
2

1
c) 1, 1,

d) (1, 1, 1)

If a plane passes through a fixed point (2, 3, 4) and meets the axes of reference in A, B and C, the
point of intersection of planes through A, B, C parallel to the coordinate planes can be

292

CO-ORDINATE GEOMENTRY
a) (6, 9, 12)
9.

Equation of a plane through the line


a) 4y 3z + 1 = 0

10.

c) (1, 1, -1)

d) (2, 3, -4)

x 1 y 2 z 3

and parallel to a coordinate axis is


2
3
4

b) 2x z + 1 = 0

c) 3x 2y + 1 = 0

d) 2x + 3y + 1 = 0

The line joining the points (2, -3, 1) and (3, -4, -5) cuts a coordinate plane at the point.
a) (0, -1, 13)

11.

b) (4, 12, 16)

b) (0, 0, 1)

c) (-1, 0, 19)

d) (8, -9, 0)

If l, m, n are the direction cosines of the line of shortest distance between the lines

x 3 y 15 z 9
x 1 y 1 z 9

and
then
2
7
5
2
1
3
a) 3l 25m + 9n = 0
c) l = m = n = 1/ 3
12.

The foot of the perpendicular from the origin to the join of A(-9, 4, 5) and B (11, 0, -1) lies on the
plane
a) 2x + y + z = 6

13.

b) x y + z = 1

c) x + y + z = 1

d) x y z = 1

If a plane p passes through the point (1, 2, 3), direction cosines of the normal to p are l, m, n; and
it contains the line joining the origin to the point (1, 1, 1), then
a) l + 2m + 3n = 0

293

b) 2l 7m + 5n = 0
d) 2l + m 3n = 0

b) l + m + n = 0

c) l + m n = 0

d) l m + 2n = 0

IIT- MATHS

COMPREHENSION TYPE QUESTIONS


PASSAGE 1:
A (-2, 2, 3) and B (13, -3, 13)
L is a line through A
1.

A point P moves in the space such that 3PA = 2PB, then the locus of P is
a) x2 + y2 + z2 + 28x 12y + 10z 247 = 0
c) x2 + y2 + z2 + 28x 12y 10 z + 247 = 0

2.

Coordinates of the point P which divides the join of A and B in the ratio 2 : 3 internally are
a) (33/5, -2/5, 9)

3.

4.

b) x2 + y2 + z2 28x + 12y + 10z 247 = 0


d) x2 + y2 + z2 - 28x + 12y 10 z + 247 = 0

b) (4, 0, 7)

c) (32/5, -12/5, 17/5)

d) (20, 0, 35)

Equation of a line L, perpendicular to the line AB is


a)

x 2 y 2 z3

15
5
10

b)

x 2 y 2 z3

3
13
2

c)

x 2 y 2 z3

3
13
2

d)

x 2 y 2 z3

15
5
10

Direction ratios of the normal to the plane passing through the origin and the points A and B are
a) 15, -5, 10

b) 11, -1, 16

c) 3, 13, 2

d) 7, 13, -4

PASSAGE 2:
a = 6i + 7j + 7k, b = 3i + 2j 2k, P(1, 2, 3).
5.

The position vector of L, the foot of the perpendicular from P on the line r = a + lb is
a) 6i + 7j + 7k

6.

c) 3i + 5j + 9k

d) 9i + 9j + 5k

The image of the point P in the line r = a + b is


a) (11, 12, 11)

7.

b) 3i + 2j 2k

b) (5, 2, -7)

c) (5, 8, 15)

d) (17, 16, 7)

If A is the point with position vector a the Area of the PLA in sq. units is equal to
a) 3 6

b) 7 17 / 2

c) 17

d) 7/2

PASSAGE 3:
P(2,3, -4), b = 2i j + 2k
8.

Vector equation of a plane passing through the point P perpendicular to the vector b is
a) r.(2i j + 2k) = -7 b) r. (2i j + 2k) = 7 c) r.(2i + 3j 4k) = -7 d) r . (2i + 3j 4k) = 7

9.

Cartesian equation of a plane passing through the point with position vector b and perpendicular to the vector OP , O being the origin is
a) 2x y + 2z + 7 = 0 b) 2x y + 2z 7 = 0 c) 2x + 3y 4z + 7 = 0 d) 2x + 3y 4z 7 = 0

10.

Sum of the lengths of the intercepts made by the plane on the coordinate axes is
294

CO-ORDINATE GEOMENTRY
a) 14
b) 91/12

c) 9/7

d) 5/7

PASSAGE 4:
L:

x 1 y 1 z 1

2
3
4

p1: x + 2y + 3z = 14, p2: 2x - y + 3z = 27

If the line L meets the plane p1 in the point P,and the coordinates of P are (), then
is equal to

11.

a) 3
12.

b) 14

c) 28

d) 29

The line through P perpendicular to the plane p1 passes through the point
a) (1, 1, 1)

b) (0, 1, 0)

c) (0, 0, 0)

d) (0, 0, 1)

If the line through P perpendicular to 1 meets the plane 2 in the point Q, then the coordinates of
the mid-point of PQ are

13.

a) (1, 2,3)

b) (3, 6, 9)

c) (2, 3, 4)

d) (2, 4, 6)

MATCHING TYPE QUESTIONS


1.
i)

x 2 y 7 z 5

3
4
2

a) Perpendicular to the plane 3x + 4y + 2z = 1

ii)

x 1 y 2 z 7

3
4
2

b) Passes through (2, 7, -5)

iii)

x 5 y 2 z 2

c) direction cosines are 2/ 30 5/ 30 1/ 30


1
3
4

iv)

x 1 y 1 z 1

2
5
1

2. ax + by + cz + d = 0,

d) lies in the plane 7x y z = 35

x y z

m
n

i) lines is perpendicular the plane

a) if al + bm + cn = 0

ii) line is parallel to the plane ifb) if al + bm + cn = 0 and aa + bb + cg + d = 0


iii) line lies in the plane c) if a/l = b/m = c/n

3. L :

295

x 2 y3 z4

is a line them
3
4
5

i) Point on the line at a distance 10 2 from (2, 3, 4)

a) (-1, -1, -1)

ii) Point on the line common to the plane x + y + z + 3 = 0

b) (2, 3, 4)

iii) Point on the line at a distance

c) (8, 11, 14)

29 from the origin

IIT- MATHS

MATCHING TYPE QUESTIONS


1.
i)

x 2 y 7 z 5

3
4
2

a) Perpendicular to the plane 3x + 4y + 2z = 1

ii)

x 1 y 2 z 7

3
4
2

b) Passes through (2, 7, -5)

iii)

x 5 y 2 z 2

c) direction cosines are 2/ 30 5/ 30 1/ 30


1
3
4

iv)

x 1 y 1 z 1

2
5
1

2. ax + by + cz + d = 0,

d) lies in the plane 7x y z = 35

x y z

m
n

i) lines is perpendicular the plane

a) if al + bm + cn = 0

ii) line is parallel to the plane ifb) if al + bm + cn = 0 and aa + bb + cg + d = 0


iii) line lies in the plane c) if a/l = b/m = c/n

3. L :

x 2 y3 z4

is a line them
3
4
5

i) Point on the line at a distance 10 2 from (2, 3, 4)

a) (-1, -1, -1)

ii) Point on the line common to the plane x + y + z + 3 = 0

b) (2, 3, 4)

iii) Point on the line at a distance

c) (8, 11, 14)

29 from the origin

296

CO-ORDINATE GEOMENTRY

NUMERICAL SUBJECTIVE TYPE QUESTIONS


1.

A plane meets the coordinate axes in A, B, C such that the centroid of the DABC is the point
(12,15,16). The sum of the squares of the intercepts made by the plane on the coordinate axes is

2.

If q is the angle between the line

x 1 y 1 z 2

and the plane 2x+y3z+4=0, then 64 cosec2 q


3
2
4

is equal to
3.

If d is the distance between the point (-1, -5, -10) and the point of intersection of the line
x 2 y 1 z 2

with the plane x y + z = 5, then 3d3 is equal to.


3
4
12

4.

If Q is the foot of the perpendicular fro the point P(4,-5,3) on the line

x 5 y 2 z6

then 100
3
4
5

(PQ)2 is equal to
5.

A plane passes through (1, 2, 2) and is perpendicular to two planes 2x 2y + z = 0 and x y + 2z


= 4. Square of the distance of the plane from the point (52, 53, 57) is.

6.

P, Q, R, S are the points (1, 2, -2), (8, 10, 11), (1, 2, 3) and (3, 5, 7) respectively. If s denotes the
projection of PQ on RS then 29s2 + 29 is equal to.

7.

The lines

x 4 y 17 z 11
x 15 y 9 z 8

and
intersect at the point P, then square of the
15
9
8
4
17
11

distance of P from the origin is.


8.

If the position vector of the point of intersection of the line r=(i+2j+3k)+l (2i+j+2k) and the
planer
r.(2i 6j + 3k) + 5 = 0 is
ai + bj + ck, then (50a + 70b + 75c)2 is equal to.

9.

If d is the shortest distance between the lines r = 3i + 5j + 7k + l (i + 2j + k) and r = -i j k m (7i


6j + k) then 125 d2 is equal to.

10.

If the foot of the perpendicular from the origin on a plane is (11, 11, 11), then the sum of the
square s of the intercepts made by the plane on the coordinate axes is equal to.

11.

If the point of intersection of the line


r = (i + 2j + 3k) + l (2i + j + 2k) and the plane
r.(2i 6j + 3k) + 5 = 0 lies on the plane
r. (i + 75j + 60k) - a = 0, then 19 a + 17 is equal to.

12.

If the line

x 1 y 1 z 1

intersect the curve 6x2+5y2=1, z=0; then 10n220n+k=0,where the


15
16
n

value of k is.

297

IIT- MATHS

SECTION - A
SINGLE ANSWER TYPE QUESTIONS

1.

2.

3.

4.

5.

6.

7.

8.

9.

10.

11.

16.

17. 18. 19.

20.

21. 22. 23

24. 25.

26.

12. 13.

27. 28.

14. 15.

29. 30.

31

32

33

34

35

36

37

38

39

40

41

42 43

44

45

46

47

48

49

50

51

52

53

54

55

56

57 58

59

60

61

62

63

64

65

66

67

68

69

70

71

72 73

74

75

298

CO-ORDINATE GEOMENTRY

76

77

78

79

80

81

82

83

84

85

86

87 88

89

90

91

92

93

94

95

96

SECTION - B
MULTIPLE ANSWER TYPE QUESTIONS
1.

2.

3.

4.

A,D B,C A,B A,D


C

5.

6.

7.

8.

9.

10.

11.

12. 13.

A,C A,C A,B A,B A,B A,C B,C A,B A,B


C C,D C
D
C

COMPREHENSON TYPE QUESTIONS

1.

2.

3.

4.

5.

6.

7.

8.

9.

10.

11.

MATCHING TYPE QUESTIONS


1.

(i) (b), (ii) (a), (iii) (d), (iv) (c)

2.

(i) (c), (ii) (a), (iii) (b)

3.
299

(i) (c), (ii) (a), (iii) (b)

12. 13.

IIT- MATHS

NUMERICAL SUBJECTIVE TYPE QUESTIONS

1.

5625

5.

5202

9.

2116

2.

1624

6.

8129

10.

3267

3.

6591

7.

1398

11.

2563

4.

1828

8.

7225

12.

2630

300

CO-ORDINATE GEOMENTRY

THREE DIMENSIONAL GEOMENTRY


We know that the position of a point in a plane can be determined if the coordinates
(x, y) of the point with refersence to two mutually perpendicular lines called x and y axes are known. But
all points of space donot lie in a plane and so in order to locate a point in space two coordinate axes are
insufficient. In order to locate a point in space we need threecoordinate axes.
The position of a point in space can be determined with reference to three mutually perpendicular
lines called x, y and z axes.
In case of two dimensional geometry two mutually perpendicular lines are taken and they divide the
plane (xyplane) in four parts. The four parts are called the quadrants. The sign of coordinates of the
points in the four parts are (+ , +), (, +), (, ) and (+, )
In case of three dimensional geometry we take three mutually perpendicular lines which divide the
space in eight parts called octants.
The sign of coordinates of the points in the 8 parts in which the space is divided are
(+, +, +) (, +, +) (+, , +), (+, +, ) (, , +),
(, +, ), (+, , ), (, , ).

O
X
Z

CO-ORDINATES OF A POINT ON AXES


1.

Coordinates of a point P on xaxis will be of the form (a, 0, 0) where a is the distance of
foot of perpendicular from P to xaxis from the origin with suitable sign. a is positive or
negative according as it lies on positive or negative direction of xaxis.

2.

Coordinates of a point on yaxis is of the form (0, b, 0).

3.

Coordinates of a point on zaxis is of the form (0, 0, g).

4.

Projection of a Line segment on a Plane: Let AB be a line segment, and L and M be the feet of
normals from A and B to a given plane. Then the line segment LM is called the projection of
the line segment AB on the plane.

DISTANCE FORMULA
Let P (x1, y1, z1) and Q (x2, y2, z2)
Then PQ =
301

(x1 x 2 )2 (y1 y 2 ) 2 (z1 z 2 ) 2

IIT- MATHS
We draw PL xy plane.
Then L (x1, y1, 0) and M (x2, y2, 0)
PL = z1, QM = z2
From DPHQ
Let PQ =

PH 2 QH 2 =

2
2
2
LM 2 QH 2 = {(x1 x 2 ) (y1 y 2 ) } (z 2 z1 )

{ LM2 = (x1 x2)2 + (y1 y2)2]


=

(x1 x 2 )2 (y1 y 2 ) 2 (z1 z 2 ) 2 .

SECTION FORMULAE
I.

Section Formula for Internal Division


Let P (x1, y1, z1) and Q (x2, y2, z2)
Let R divide the line segment PQ internally in the ratio m : n

mx 2 nx1 my 2 ny1 nz 2 nz1


,
,
Then R

mn
mn
mn

II.

Section Formula for External Division


Let P (x1, y1, z1), Q (x2, y2, z2)
Let R divide the line segment PQ externally in the ratio m : n, then

PR m

RQ n

Let R (x, y, z) From P, Q, R draw PL, QM and RN perpendiculars to xyplane


Again draw PH RN and QK RN
RH PR

From similar DPHR and DQKR


RK QR

z z1 m PR m

nz nz1 = mz mz2
z z 2 n QR n

(m n) z = nz2 nz1
z=

nz 2 nz1
mn

Similarly by drawing perpendiculars from P, Q, R to yzplane we can show that


x=

mx 2 nx1
mn

Again by drawing perpendiculars from P, Q, R to zx plane we can show that

302

CO-ORDINATE GEOMENTRY
my 2 ny1
y=
mn
mx 2 nx1 ny 2 ny1 mz 2 nz1
,
,
Thus R

mn
mn
mn

ANGLE BETWEEN TWO LINES


Let AB and CD be two given lines having direction cosines l1 , m1, n1 and l2, m2, n2 respectively and
q be the angle between them. Let O be the origin. Through O we draw OP parallel to AB and OQ parallel
to CD.
Let P (x1, y1, z1), Q (x2, y2, z2) and OP = r1, OQ = r2
Since direction cosines of AB are l1, m1, n1 and OP || AB
Therefore d.cs of OP are l1, m1, n1. Similarly d.cs of OQ will be l2, m2, n2

x1
y1
z1
x2
y2
z2
Now l1 = r , m1 r , n1 r and l2 = r , m 2 r , n 2 r
1
1
1
2
2
2
From PQO,
r12 r22 [(x1 x 2 )2 (y1 y 2 )2 (z1 z 2 )2 ]
OP 2 OQ 2 PQ 2
cosq =
=
2.r1 r2
2.OP.OQ
r12 r22 [(x12 y12 z12 ) (x 22 y 22 z 22 ) 2x1x 2 2y1y 2 2z1z 2 ]
=
2r1r2

x1 x 2 y1 y 2 z1 z 2
r12 r22 (r12 r22 2x1x 2 2y1y 2 2z 1z 2 )
=
= r .r r .r r .r
2r1r2
1
2
1
2
1
2
cos = l1l2 + m1m2 + n1n2) (1)
or

= cos1 (l1l2 + m1m2 + n1n2)

ANGLE BETWEEN TWO LINES IN TERMS OF DIRECTION RATIOS


Let a1, b1, c1 and a2, b2, c2 be the direction ratios of two lines AB and D respectively.
Direction cosines of the two lines are given by
l1

a1
a12 b12 c12

, m1

a2

and l2 =

2
2

2
2

a b c

2
2

b1
a12 b12 c12

, m2

c1

, n1

a12 b12 c12

b2
2
2

2
2

a b c

2
2

, n2

If be the angle between the two lines, then


cos = l1 l2 + m1m2 + n1n2

303

c2
2
2

a b 22 c22

IIT- MATHS
a1a 2 b1b 2 c1c 2

2
1

a b12 c12 a 22 b 22 c 22

The two lines will be perpendicular iff or, a1a2 + b1b2 + c1c2 = 0
sin =

(l1m 2 l2 m1 )2 (m1m 2 m 2 n1 ) 2 (n1l2 n 2l2 ) 2

The two lines will be parallel iff = 0 or sin = 0


or,

(a1b2 a2b1)2 + (b1c2 b2c1)2 + (c1a2 c2a1)2 = 0

or,

(a2b2 a2b1)2 = 0, (b1c2 b2c1)2 = 0, (c1a2 c2a1)2 = 0

or,

a1 b1 c1
sin

tanq
=
=
a 2 b2 c2
cos

(a1b 2 a 2 b1 ) 2 (b1c 2 b 2c1 ) 2 (c1a 2 c 2a1 ) 2


a1a 2 b1b 2 c1c 2

PROJECTION OF THE LINE SEGMENT JOINING TWO POINTS ON A LINE


Projection of the line segment joining (x1, y1, z1) and (x2, y2, z2) on the line having direction cosines l, m, n.
= l(x2 x1) + m(y2 y1) + n (z2 z1)
The length of projection of the line segment joining (x1, y1, z1) and (x2, y2, z2) on the line having
direction cosines l, m, n.
= |l(x2 x1) + m(y2 y1) + n(z2 z1)|

GENERAL EQUATION OF A PLANE


General equation of the first degree in x, y, z always represents a plane.
Let the general equation of second degree in x, y and z be
ax + by + cz + d = 0

(1)

where at least one of a, b, c is nonzero.


Locus (1) will be a plane if the line joining any two points on the surface fully lies on the surface.
Let A(x1, y1, z1) and B(x2, y2, z2) be any two points on locus (1), then
ax1 + by1 + cz1 + d = 0 (1)
and ax2 + by2 + cz2 + d = 0

(2)

m.(2) + n.(1)
a(mx2 + nx1) + b(my2 + ny1) + c(mz2 + nz1) + (m + n) d = 0

mx 2 nx1 my 2 ny1 mz 2 nz1


or, a
b
c
+d=0
mn mn mn

(3)

mx 2 nx1 my 2 ny1 mz 2 nz1


Let P () where denotes
,

, it is clear from
mn mn mn

(3) that P() lies on locus (1). But P is the point dividing the line segment joining A and B in the ratio

304

CO-ORDINATE GEOMENTRY
m : n and m and n are arbitrary. Hence all points on the line joining A and B lies on the locus (1). Thus
locus (1) is a plane.
Note 1: Equation of the xyplane
Let the equation of the xyplane be
ax + by + cz + d = 0 (1)
Since O(0, 0, 0) lies in the xyplane

d=0

Since (1, 0, 0) lies on xaxis, therefore it also lies on the xyplane

a=0

Again (0, 1, 0) lies on yaxis, therefore it also lies on xyplane

b=0

Hence from (1), equation of the plane becomes


cz = 0 or, z = 0
Thus equation of xyplane is z = 0

Equation of a plane in Intercept Form


Equation of the plane which cuts intercepts a, b, c on x, y and zaxis respectively is
x y z
1
a b c

Equation of a plane in Normal Form


Equation of the plane upon which the length of perpendicular from origin is p and l, m, n be the
direction cosines of the normal to it is
lx + my + nz = p

Equation of a plane parallel to a given plane:


General equation of a plane parallel to the plane ax + by + cz + d = 0 is ax + by + cz + k
= 0. Where k is a constant.

Angle between two planes:


Angle between two planes is equal to the angle between their normals
Angle q between two planes
a1x + b1y + c1z + d1 = 0 and a2x + b2y + c2z + d2 = 0 is given by

305

IIT- MATHS
a1a 2 b1b 2 c1c 2

cos =

2
1

a b12 c12 a 22 b 22 c 22

here a1, b1, c1 are the direction ratios of the normal to the first plane and a2, b2 c2 are the direction
ratios of the normals to the second plane.

Distance of a Point From a plane


Length of perpendicular from point (, , ) to the plane ax + by + cz + d = 0 is given
by
P=

a b c d
a 2 b2 c2

Equation of the planes bisecting the angle between two planes


Equation of the planes bisecting the angle between the planes
a1x + b1y + c1z + d1 = 0 and a2x + b2y + c2z + d2 = 0 is
a1x b1y c1z d1

2
1

2
1

2
1

a 2 x b2 y c2z d 2
a 22 b 22 c 22

a b c

Bisector of the angle between two planes containing the origin


Let the equation of the two planes be
a1x + b1y ++ c1z + d1 = 0 (1)
and a2x + b2y + c2z + d2 = 0

(2)

where d1 and d2 are positive then


equation of the bisector of the angle between planes (1) and (2) containing the origin is
a1x b1y c1z d1
2
1

2
1

2
1

a 2 x b2 y c2z d 2

a b c

a 22 b 22 c 22

Bisector of the acute and obtuse angle between two planes


Let the equation of the two planes be
a1x + b1y + c1z + d1 = 0

(1)

and a2x + b2y + c2z + d2 = 0

(2)

where d1, d2 > 0


(i) If a1a2 + b1b2 + c1c2 > 0, then origin lies in the obtuse angle between two planes and the
equation of bisector of the acute angle between two planes is

306

CO-ORDINATE GEOMENTRY
a1x b1y c1z d1
a x b2 y c2z d 2
2
2
2
2
a1 b1 c1
a 22 b 22 c22
(ii) If a1a2 + b1b2 + c1c2 < 0, then origin lies in the acute angle between two planes and the
equation of bisector of the acute angle between two planes is
a1x b1y c1z d1
2
1

2
1

2
1

a b c

a 2 x b2 y c2z d 2
a 22 b 22 c 22

Otherwise, the bisector of the acute angle makes with either of the planes an angle which is less
then 45 and the bisector of the obtuse angle makes with either of them an angle which is greater than 45.
This gives a test for determining which angle, acute or obtuse, each bisecting plane bisects.

307

IIT- MATHS

THE STRAIGHT LINES


Introduction:
We know that
1. one and only one line can be drawn through a given direction.
2. One and only one line can be drawn through two given points.
3. Intersection of two non parallel planes is a unique line.
Thus a straight line in space will be determined uniquely if
(i)

it passes through a fixed point and is parallel to a fixed line

(ii)

it passes through two fixed points.

(iii)

it is the intersection of two given non parallel planes.

Equation of the line passing through point (x1, y1, z1) and having direction rations (or
direction cosines) l, m, n is
x x1
y y1 z z1

=
l
m
n

Equation of the line joining points (x1, y1, z1) and (x2 , y2, z2) is

x x1
y y1 z z1

x1 x 2 y1 y 2 z1 z 2
The Equation of a Line (General Form)
Intersection of two planes is a straight line, therefore in general a line is obtained by the
intersection of two planes.
Equation of a line as the intersection of two planes
Equation of the line, which is the intersection of two planes
a1x + b1y + c1z + d1 = 0 and a2x + b2y + c2z + d2 = 0 is
a1x + b1y + c1z + d1 = 0

(1)

and a2x + b2y + c2z + d2 = 0

(2)

Equation (1) and (2) taken together is the equation of line of intersection of planes (1) and (2)
in asymmetrical form.
Angle between a line and a plane.
Angle between a line and plane is the complement of the angle between the line and the
normal to the plane.
Let the given line PQ be

308

CO-ORDINATE GEOMENTRY
x x1 y yi z z1

(1)
l
m
n
and the given plane be
N

Q
90

ax + by + cz + d = 0

(2)

Let q be the angle between the line and the plane, then the angle between the line and normal
PN to the plane is 90 .
Direction ratios of PN are a, b, c and direction ratios of line PQ are l, m, n
al bm cn
Now cos(90 )

a 2 b 2 c2 l 2 m 2 n 2
al bm cn

or sin =

a 2 b2 c2 l 2 m2 n 2

General Equation of the Plane containing a line


Equation of any plane containing line
x x1 y y1 z z1

is
l
m
n

A(x x1) + B(y y1) + C(z z1) = 0


Where Al + Bm + Cn = 0
Equation of the Plane containing two given lines
Equation of the plane containing lines

x x1 y y1 z z1
x x 2 y y2 z z 2

and

l1
m1
n1
l2
m2
n 2 is

or,

x x2

y y2

z z2

l1
l2

m1
m2

n1
n2

To Find the shortest distance between two lines

309

x x1

y y1

z z1

l1
l2

m1
m2

n2
n2

=0

IIT- MATHS

x x1 y y1 z z1
x x 2 y y2 z z 2

and
l1
m1
n1
l2
m2
n2
Given lines are

x x1 y y1 z z1

(1)
l1
m1
n1
x x 2 y y2 z z 2

And CD : l
m
n 2 (2)
2
2
AB :

A(x1, y1, z1)

90
90

(x2, y2, z2)

Let l, m, n be the direction ratios of the shortest distance LM.


Since LM AB and LM CD
ll1 + mm1 + nn1 = 0

(3)

andll2 + mm2 + nn2 = 0

(4)

From (3) & (4),

l
m
n

m1n 2 m 2 n1 n1l2 n 2l1 l1m 2 l2 m1 =

1
(m1n 2 m 2 n1 )2

From the above equation l, m & n can be found out and the shortest distance between the lines is
projection of line joining the point (x1, y1, z1) & (x2, y2, z2) on the line with the direction cosines l, m & n.

Image of a Point in a Plane


To find the image of the point P(a, b, g) in the plane
ax + by + cz + d = 0
Given plane is
ax + by + cz + d = 0 (1)
P

Q
P (a, b, g)
310

CO-ORDINATE GEOMENTRY
Let Q(x1 ,y1 , z1) be the image of point P in the plane (1)
Let PQ meet plane (1) at L.
Direction ratios of normal to plane (1) are a, b, c
Since PQ plane (1), therefore direction ratios of PQ are a, b, c
equation of lien PQ is

x y z

= r (say)
a
b
c

Coordinates of any point on line PQ may be taken as


(ar + a, br + b, cr + g)
Let Q (ar + a, br + b, cr + g)
Since L is the middle point of PQ

ar
br
cr

L , ,
2
2
2

Since L lies on plane (1)

ar
br
cr

a b c d = 0
2
2
2

(a2 + b2 + c2)

r
= (aa + bb + cg + d)
2

Thus Q (a + ar, b + br, g + cr)


Where r =

311

2(a b c d)
a 2 b2 c2

r=

2(a b c d)
a 2 b2 c2

You might also like